Wikipedia:Reference desk/Science: Difference between revisions

From Wikipedia, the free encyclopedia
Content deleted Content added
→‎Medroxyprogesterone versus medroxyprogesterone acetate: correction - not sure about MP's use in humans at all
Line 877: Line 877:


I'm trying to figure out the difference between [[medroxyprogesterone]] versus [[medroxyprogesterone acetate]]. Anyone know? Anyone have any sources that can be used to distinguish the two? [[User:WLU|WLU]] <small>[[User talk:WLU|(t)]] [[Special:Contributions/WLU|(c)]] Wikipedia's rules:</small>[[WP:SIMPLE|<sup><span style='color:#FFA500'>simple</span></sup>]]/[[WP:POL|<sub><span style='color:#008080'>complex</span></sub>]] 01:57, 3 July 2010 (UTC)
I'm trying to figure out the difference between [[medroxyprogesterone]] versus [[medroxyprogesterone acetate]]. Anyone know? Anyone have any sources that can be used to distinguish the two? [[User:WLU|WLU]] <small>[[User talk:WLU|(t)]] [[Special:Contributions/WLU|(c)]] Wikipedia's rules:</small>[[WP:SIMPLE|<sup><span style='color:#FFA500'>simple</span></sup>]]/[[WP:POL|<sub><span style='color:#008080'>complex</span></sub>]] 01:57, 3 July 2010 (UTC)
:Other than the obvious difference in chemical structure, what are you looking for? My sense is that these are equivalent biologically, i.e. the acetate is readily hydrolyzed to the parent compound. [http://www.merck.com/mmpe/lexicomp/medroxyprogesterone.html The Merck Manual entry] indicates that the former is "supplied as the acetate". The [http://www.nlm.nih.gov/medlineplus/druginfo/meds/a682470.html article in MedlinePlus] uses the terms interchangeably. -- [[User:Scray|Scray]] ([[User talk:Scray|talk]]) 02:59, 3 July 2010 (UTC)
:Other than the obvious difference in chemical structure, what are you looking for? It appears that the 17-acetate is the only one used medically - both of the references in the MP article actually relate to MPA. [http://www.merck.com/mmpe/lexicomp/medroxyprogesterone.html The Merck Manual entry] indicates that the former is "supplied as the acetate". The [http://www.nlm.nih.gov/medlineplus/druginfo/meds/a682470.html article in MedlinePlus] uses the terms interchangeably. Many references discuss plasma levels of MPA, so my initial impression that the acetate is readily hydrolyzed is probably false. -- [[User:Scray|Scray]] ([[User talk:Scray|talk]]) 02:59, 3 July 2010 (UTC)

Revision as of 03:10, 3 July 2010

Welcome to the science section
of the Wikipedia reference desk.
Select a section:
Want a faster answer?

Main page: Help searching Wikipedia

   

How can I get my question answered?

  • Select the section of the desk that best fits the general topic of your question (see the navigation column to the right).
  • Post your question to only one section, providing a short header that gives the topic of your question.
  • Type '~~~~' (that is, four tilde characters) at the end – this signs and dates your contribution so we know who wrote what and when.
  • Don't post personal contact information – it will be removed. Any answers will be provided here.
  • Please be as specific as possible, and include all relevant context – the usefulness of answers may depend on the context.
  • Note:
    • We don't answer (and may remove) questions that require medical diagnosis or legal advice.
    • We don't answer requests for opinions, predictions or debate.
    • We don't do your homework for you, though we'll help you past the stuck point.
    • We don't conduct original research or provide a free source of ideas, but we'll help you find information you need.



How do I answer a question?

Main page: Wikipedia:Reference desk/Guidelines

  • The best answers address the question directly, and back up facts with wikilinks and links to sources. Do not edit others' comments and do not give any medical or legal advice.
See also:



June 28

Listed ingredients

So I was drinking my Lipton Lemon Ice Tea (LLIT), and thinking, "damn, this tastes like bad water", and I realized (perhaps a bit late) that drinks, obviously containing huge amounts of water, rarely (never?) actually list water as part of their ingredients, at least where I live.

Is there any reason companies would wish to avoid printing water on their ingredient labels (other than the fact that it can always be assumed, and thus is just a waste of printing space)?
Why are the relative amounts of individual ingredients on packaging never (properly) listed? Is this a "printing space" issue as well? i.e. listing them in order weight eliminates a lot of information that may not be needed. I would have liked to know if LLIT was only 95% water, or like 99.9% water, mind you.
For the consumer, it seems that the more comprehensive the ingredients are, the better. Is there any other reason why companies seem only to publicize the absolute legal minimum when it comes to the ingredients of consumables? Thanks in advance. 210.165.30.169 (talk) 01:03, 28 June 2010 (UTC)[reply]
In the UK, drinks that contain added water do include water in their list of ingredients. --Tango (talk) 01:37, 28 June 2010 (UTC)[reply]
In the US, if you look at, say, a bottle of Diet Coke, the first ingredient is listed as "Carbonated filtered water", so I am not familiar with any type of product that is allowed to omit water from the ingredients list. As for why companies only do the legal minimum, one possible reason is that from the company's point of view, their recipe may be considered a crucial trade secret and they don't want to tell all their competitors how to clone their product. Comet Tuttle (talk) 02:50, 28 June 2010 (UTC)[reply]
Yeah I see labels that say "Flavor" or "Seasoning" instead of listing the ingredients. --Chemicalinterest (talk) 10:50, 28 June 2010 (UTC)[reply]
Also in the US, and I am under the impression that very few products put water in the label. I certainly only see it rarely. Falconusp t c 04:58, 28 June 2010 (UTC)[reply]
Can you give an example of a product that contains added water (not just water contained in another ingredient) and doesn't include it in the list of ingredients? --Tango (talk) 15:53, 28 June 2010 (UTC)[reply]
Sometimes they put "aqua" instead of the word water, and talking about "bad" those tea leaves were probably decomposed slightly before making up the drink. Graeme Bartlett (talk) 05:16, 28 June 2010 (UTC)[reply]
As for why companies are not interested in detailed ingredient lists, it's in part because of trade secrets, as well as the fact that most of them don't get a lot out of telling the consumer that a bunch of scary-sounding (but probably innocuous) chemicals are added to their food in order to give it the flavor, texture, and color they desire. When questions of labeling have come up—say, in regards to BGH—the companies not in favor of labeling usually say, "hey, the FDA says it's safe—otherwise we couldn't sell it—and so why do we need to rile people up about something that won't hurt them?" I find it a pretty uncompelling argument—"people are too dumb to understand ingredients lists, so let's just keep them in the dark on it"—but there you have it. --Mr.98 (talk) 16:19, 28 June 2010 (UTC)[reply]
Well, when you consider how many people get freaked out over dihydrogen monoxide in consumer products, it's pretty clear that many people are too dumb to understand ingredient lists and thus are better off kept in the dark on it. FWiW 67.170.215.166 (talk) 07:47, 30 June 2010 (UTC)[reply]

Sigma additivity in quantum mechanics

Is sigma additivity true for probabilities in quantum mechanics? Sigma additivity is one of the axioms of probability, what I would like to know is if it is an arbitrarily chosen axiom or if there is any scientific basis behind it (and the only application of probability in its purest form in science is in quantum mechanics). Also, do the other axioms of probability hold in quantum mechanics? ––115.178.29.142 (talk) 03:21, 28 June 2010 (UTC)[reply]

Yes, the probabilities in quantum mechanics are probabilities. If the definition of probability didn't apply, it wouldn't be called a probability. (I'm having difficulty seeing the motivation for this question.) Looie496 (talk) 04:25, 28 June 2010 (UTC)[reply]
I'm afraid I think that answer is a little reductive. Sigma-additivity is part of the mathematical theory of probability. Whether it has any direct physical counterpart is another matter entirely.
To elaborate: Sigma-additivity says that if you have a countably infinite (or, less interestingly, finite) number of mutually exclusive possibilities, the probability that one of the events happens is exactly the sum of all the probabilities of an individual event happening.
This property is important to make the mathematical theory work smoothly. What, if any, physical reality it corresponds to, is less clear — are there even an infinite number of mutually exclusive events to consider, in the physical world? There may be, but then again there may not.
You don't have to go to anything as exotic as sigma-additivity of measures to see this distinction. An example much nearer to home is provided by the real numbers. Mathematically, the reals are an inherently infinitary notion; each real number contains infinitely much information, all wrapped up in a neat little package. Does, say, the distance between two electrons truly encode infinitely much information? Perhaps, but I think the jury is still out (in fact the jury may well never come back). --Trovatore (talk) 08:08, 28 June 2010 (UTC)[reply]
So basically it's unknown whether or not sigma additivity is true in physics? Fundamentally what I want to know is if the axioms of probability are actually "true" in real life, so to speak. Axioms are meant to be self-evident, especially in mathematics, is this true for the axioms of probability such as sigma additivity?––Original Poster

Note that probabilities in quantum mechanics are the square of the norm of the wavefunction. The (complex) wavefunction itself is not a probability and does not obey the axioms of probability, even though certain terminology, such as "probability amplitude" might suggest otherwise. 157.193.175.207 (talk) 08:06, 28 June 2010 (UTC)[reply]

Note that sigma additivity of probabilities applies to sets of pairwise disjoint (i.e. mutually exclusive) events. It can appear to fail in quantum mechanics because events that we intuitively expect to be mutually exclusive are not necessarily so. For example, in a double-slit experiment, suppose event A is "particle goes through slit 1 and hits point X on screen", and event B is "particle goes through slit 2 and hits point X on screen". In classical physics we expect the particle to follow a single path from source to screen (even though we do not observe this path) and so we expect A and B to be mutually exclusive; this leads us to expect Pr("particle hits point X on sreen") to be Pr(A)+Pr(B). In quantum mechanics, Pr("particle hits point X on sreen") is not Pr(A)+Pr(B) - wavefunctions add, but probabilities do not. This is not because sigma additivity fails, but because A and B are no longer mutually exclusive - the particle can, in effect, go through both slits (see path integral formulation). Gandalf61 (talk) 08:49, 28 June 2010 (UTC)[reply]
obviously the probability amplitudes do not obey the axioms of probability because the probability amplitudes are not actually probabilities. However, do the probabilities derived from the probability amplitudes obey the axioms of probability? Basically what I want to know, as I stated above, is whether or not the probability axioms are actually true in nature or if they were just arbitrarily chosen. ––Original Poster
Ignoring quantum mechanics, have you tried rolling dice, or something else simple where you can test for yourself whether the axiom has a physically real basis, or are you already past that stage? What exactly are you asking - is this question specifically about quantum mechanical probabilities, or more general?
"However, do the probabilities derived from the probability amplitudes obey the axioms of probability" yes.87.102.11.74 (talk) 12:13, 28 June 2010 (UTC)[reply]
The result obtained from rolling dice is a deterministic system dependent on how you roll the dice. That's why a chose quantum mechanics ––OP
That's a false dilemma. The axioms of probability are not "just arbitrarily chosen" - they are carfeully chosen to provide a consistent mathematical framework that implements our intuitive concept of odds and probability. However, we cannot measure probabilities directly - we can only estimate them from the results of multiple trials. So it is probably impossible to say whether they are "actually true in nature". How would you design an experiment to test whether probabilities "in nature" do or do not conform to the axioms of probability ? Gandalf61 (talk) 12:11, 28 June 2010 (UTC)[reply]
How are the axioms of probability derived? Or are they self-evident, as many axioms in mathematics are? And if they are self-evident, doesn't that mean that they must be true in quantum mechanics?––OP
I think you may have misunderstood the term "self-evident". When we say that axioms are taken to be self-evident, this means that we do not have to prove that an instance of a mathematical structure, such as a probability, conforms to the axioms of that structure, because the axioms are taken to be part of the object's definition. This avoids the danger of infinite regress in mathematical proofs - each chain of reasoning in a proof eventually reaches an axiom, which we can take to be true by definition. "Self-evident" does not means "obvious" - the axioms for many mathematical structures are far from obvious. Neither does it mean "true in reality" - a mathematical structure may or may not be a good model of some part of reality, but this does not affect its axioms. Gandalf61 (talk) 14:00, 28 June 2010 (UTC)[reply]
Hmm, no, I don't really agree with that; that's a bit close to the sort of formalism that reduces mathematics to a meaningless game. It is important whether axioms are true in reality, for cases (like the natural numbers, or sets considered as elements of the von Neumann hierarchy), where the objects they are talking about are well-specified. And I think self-evident axioms do have to be obvious once correctly understood.
The point is that not all axioms are self-evident. There are different sorts of axioms. One sort is the self-evident kind; once you understand what it means, it is intuitively clear that it's true. The axiom of choice is an example of this.
Then there are axioms that are not intuitively clear at all, but for which evidence accumulates that they are actually true. Large cardinal axioms are in this category.
Then there are axioms for which there is no particular evidence, or even evidence against them, but which are convenient in certain contexts (e.g. the axiom of constructibility, Martin's axiom, although some might argue that Woodin's work provides evidence for the latter).
But the "axioms of probability" being discussed here are in still a fourth category, that of "definitions in disguise". When you speak of the "axioms" of a group, what you really mean is the properties that a structure must satisfy in order to be considered a group. Similarly, the axioms of probability are the properties that a function from your event space to the reals must satisfy, in order to be considered a probability measure. --Trovatore (talk) 19:02, 28 June 2010 (UTC)[reply]
Despite your poor opinion of it, formalism is a logical and consistent position and a mainstream branch of the philosophy of mathematics. Your taxonomy of self-evident axioms, axioms that require evidence, convenient axioms and axioms that are "definitions in disguise" seems highly arbitrary to me. To a formalist, all axioms are "definitions in disguise", and the idea of collecting evidence "for" or "against" them is meaningless. Gandalf61 (talk) 21:30, 28 June 2010 (UTC)[reply]
Formalism is understandably very popular among mathematicians who would rather not think about foundational philosophy at all, but it has severe limitations once you start to take it seriously. This is probably not the place to discuss them at length; let me just say that formalism has no satisfying explanation for the apparent coherence of the overall mathematical picture.
I disagree that "to a formalist, all axioms are 'definitions in disguise'" — actually you have to be a realist to some degree to make sense of that notion of axiom. Otherwise, what are you defining? I would say that formalists would more easily place all axioms in the "convenient" category. --Trovatore (talk) 21:50, 28 June 2010 (UTC)[reply]
The question "What are you defining ?" only makes sense in a realist framework. For a formalist, axioms define what they define. A formalist does not expect the structures of mathematics to have any "existence" or reference outside of themselves. If they do happen to have some contingent correspondence to some aspect of the real world, then that is a fortuitous coincidence. I don't see how or why a formalist would categorise axioms as "convenient" or "inconvenient". Gandalf61 (talk) 07:14, 29 June 2010 (UTC)[reply]
So you're saying that the axioms define — themselves? If all you have is the axioms, what is the point of speaking of them as "defining" anything? --Trovatore (talk) 07:26, 29 June 2010 (UTC)[reply]
No, I said the axioms define what they define. The group axioms define a group, and a group is a mathematical object for which the group axioms are true - nothing more, nothing less. For a formalist, all mathematical structures are like that; there is no need or expectation of some real world referent against which the axioms can be compared. Gandalf61 (talk) 12:15, 29 June 2010 (UTC)[reply]
But if there are in fact no mathematical objects, as a formalist would hold, then how can you make sense of the claim that a group is a mathematical object for which the group axioms are true? --Trovatore (talk) 18:27, 29 June 2010 (UTC)[reply]
We clearly have somewhat different conceptions of formalism. No point continuing this discussion - I am done here. Gandalf61 (talk) 09:04, 30 June 2010 (UTC)[reply]
LOL, all I was asking was if the axioms of probability were true in quantum mechanics (to which I still don't have a definitive answer...) Actually it appears that I do have a definitive answer from 87.102.11.74, but he didn't give any reason to support his claim. Does classical probability apply in quantum mechanics? ?––115.178.29.142 (talk) 22:15, 28 June 2010 (UTC)[reply]

{outdent} It's not clear to me that anyone can possibly answer your question. It's a little like saying, we measure mass as a real number, so do masses have the Archimedean property? I can't imagine how you would design an experiment to test that. --Trovatore (talk) 22:19, 28 June 2010 (UTC)[reply]

In that case, what's the logic behind using the axioms we do? And furthermore, is it OK to use classical probability in quantum mechanics?––115.178.29.142 (talk) 00:30, 29 June 2010 (UTC)[reply]
You might want to take a gander at The Unreasonable Effectiveness of Mathematics in the Natural Sciences. I don't think you're going to get any definitive answer here. This is stuff people will be arguing about for a long time. I would note in passing that there's all sorts of stuff in QM of which people could (and do) ask whether/why it's "OK". --Trovatore (talk) 00:39, 29 June 2010 (UTC)[reply]
The probability of finding particle A at position y at time x is 1/5. The probability of finding particle B at position z and time w is 1/10. What is the probability of finding particle A at position y at time x AND particle B at position z at time w?––115.178.29.142 (talk) 01:00, 29 June 2010 (UTC)[reply]
No idea - you haven't given us any information on whether the events are independent or correlated. I carry my umbrella on 1 day in 5 and it rains on 1 day in 10. What is the probability that I carry my umbrella on a day when it is raining ? Gandalf61 (talk) 07:14, 29 June 2010 (UTC)[reply]

Signal processing

Could anyone recommend a good textbook on circuits and signal processing? I would like something to read over the summer. 173.179.59.66 (talk) 07:58, 28 June 2010 (UTC)[reply]

Are you looking for an introductory circuits text, or a text on circuits for signal processing? This will significantly change what will count as a "good book" for you. Proakis and Manilakis, Digital Signal Processing, is the "standard" DSP reference. It's very mathematical and expects a solid understanding of discrete mathematics before your start. It also exclusively focuses on digital signal processing. This is the book if you already have a solid background, but it will be totally incomprehensible if you aren't mathematically inclined. (It will also "assume" that you understand how to map a z-domain algorithm back to a digital circuit - an easy task, but one that isn't explained in this book). I also recommend Physical Audio Signal Processing. This book focuses on modeling physical acoustic behaviors with signal-processing approximations, and then implementing those as simple algorithms in software or digital hardware. It also makes for "light reading" although it will jump to extremely mathematical treatments for one or two sections at a stretch. The fields of "circuits" and "signal processing" are extremely broad - and the best books for an electrical engineer would be totally incomprehensible to a non-engineer. If you've never had exposure to even basic circuits, you should start with an introductory text on electronics before you start to worry about signal processing. Fundamentals of Electric Circuits by Alexander and Sadiku is a good one, but if you've even had a cursory circuit training, the first half of this book will insult your intelligence. In any event, the book does cover methods all the way up to frequency analysis, resonance, and I think even does s-domain circuit solutions. If you want to proceed down the all-analog route, you may find a book on analog control theory or RF signal conditioning a "must" - signal processing entails a totally different skillset in the analog domain. For this, you will need Analog Circuits, by Gray and Meyer, or Planar Microwave Engineering. (You can buy this online or in any bookstore... but these are very advanced circuit theory books). Can you specify your baseline knowledge/background a bit more? Nimur (talk) 15:07, 28 June 2010 (UTC)[reply]
There is a course next semester that I'm enrolled to take called signal-processing (they don't have a textbook listed yet). Here's the course description: "Experimental research depends strongly on electrical and electronics instruments. Today, signals from various probes are most of the time transformed into some kind of electric signal follow by some kind of digitization. This course will review some of the concepts that are encountered in the treatment of such electric signals in order to optimize the quality of the measurements. Some of the main Course Topics that will be discussed are:

• dc circuits and networks • Linear circuit elements: R, L, C • Sinusoidal signals: phasors and complex algebra • Filters: High-pass, Low-pass and Resonance • Power, rectification and noise • Fourier methods"

I'm not sure which of the textbooks above fits with the course. And thanks for the swift and detailed response! 173.179.59.66 (talk) 00:28, 29 June 2010 (UTC)[reply]
Based on what you've described, it sounds like Alexander and Sadiku is the book you want. The others might be fun to look at but I think you'll need to work your way up to them. If you find that the Fundamentals book moves too slow, you can really skip or skim several of the first chapters. The other books I mentioned will probably be too advanced if you still do not know the basics of Fourier transforms and linear networks; but in time you'll have the foundations. Electronic engineering is very dependent on a solid understanding of the basics. Nimur (talk) 01:17, 29 June 2010 (UTC)[reply]
Thanks man, I really appreciate your help. 173.179.59.66 (talk) 02:31, 29 June 2010 (UTC)[reply]

Quantam Handwaveing

What is meaning of Quantam Handwaving ?  Jon Ascton  (talk) 08:41, 28 June 2010 (UTC)[reply]

See handwaving. Some context would be helpful, but it probably means a plausible but informal argument based on the principles of quantum mechanics. Gandalf61 (talk) 12:01, 28 June 2010 (UTC)[reply]
Maybe, maybe not. I've observed that putting "quantum" in front of anything can change its commonsense meaning quite considerably if done so by scientists (e.g. the meanings of quantum teleportation or quantum computer are not obvious from their names).
In any case, Googling for the phrase seems to show it cropping up somewhat informally in ID/Creationism/Big Bang debates, probably in reference to the theory that? quantum thermal fluctuations serves as a "first cause" in the Big Bang, which seems to be seen as a form of handwaving by ID/Creationist types (that is, you are appealing to quantum mechanics in a vague way to get the answer you want, but it is not very concrete feeling). --Mr.98 (talk) 15:59, 28 June 2010 (UTC)[reply]

Wind chill and perspiration

I am trying to understand the effect of wind chill on cooling the human body and its impact on perspiration. Anecdotally I find that when there is a strong breeze when exercising, I seem to sweat much less. I am not sure if they is because I am sweating as much as usual, but that the breeze is helping it to evaporate faster, or whether my body is being cooled by the wind chill, so I need to sweat less, or a combination of both. The article on wind chill mentions that it causing cooling, but doesn't actually go into the details of why that happens. My guess, is that the moving air is having the effect of providing a greater number of "cool" air molecules to absorb heat from the body, much like why a liquid cools objects faster than air. On that basis I would expect that, in fact, the wind chill is cooling the body so that less sweating is required. However, I have also been told that swimmers are advised to still drink a lot because they do actually still sweat. If my theory about the wind chill is correct, it would seem to suggest that water would cool even more and so reduce sweating even more. Any input would be appreciated. Thanks HappyHopper777 (talk) 11:31, 28 June 2010 (UTC)[reply]

I think both effects are significant, and the balance will depend on humidity and airflow over the skin. A person will perceive that they are sweating more under high humidity or in still air, because the sweat accumulates on the body, but people still lose heat through sweating in high winds and low humidity, even when there is no noticeable sweat on the skin. Dbfirs 12:35, 28 June 2010 (UTC)[reply]
It takes energy to convert liquid water into water vapor - that energy comes from the warmth of your skin - so when sweat evaporates, it cools you down. With less humid air, that happens more easily - and in 100% humidity, it doesn't happen at all. When there is no wind - or when you are wearing lots of clothing that traps air - the temperature of the air close to your skin goes up (because it's being heated by your body) and humidity of that air also goes up because it's picking up water vapor from your evaporating sweat. When that layer of air isn't moving at all, you start to feel really hot because sweat evaporation has stopped and the air is warm. Add a little wind - and/or remove clothing to allow some convection (hot air rises) - and the air next to your skin is replaced by fresh air from further away. Now you have cooler AND drier air and your sweat can do it's job. The situation with swimmers is a little different. Air is a really good insulator - so heat passes only slowly through it. Water conducts heat away quickly - so even though your sweat can't evaporate, the water is conducting the heat away very efficiently and you don't overheat so long as the water is cooler than you are. SteveBaker (talk) 12:44, 28 June 2010 (UTC)[reply]
Thanks Steve. That fits with my undestanding I think. That water is a good conductor, compared to air, because there are a lot more molecules available to absorb the heat from the object being cooled. Fast moving air seems to have a similar effect in that, for a given amount of time, it provides a greater number of "cooler" molecules (i.e. molecules with less energy). However, because of the cooling effect of the fast air or water, I would have thought that there would be less need to sweat (because the body could transfer heat more easily to the fast air and to the water). I need to digest what you said above a bit more. Thanks HappyHopper777 (talk) 13:15, 28 June 2010 (UTC)[reply]

Pitot tube constant

For an 's' type pitot tube, the manufacturer gives the pitot tube constant as 0.8. Does this mean my actual velocity will be 0.8 times indicated velocity? Since the pitot tube is a straightforward application of Bernoulli's theorem, what causes this constant? Thanks —Preceding unsigned comment added by 125.17.148.2 (talk) 11:38, 28 June 2010 (UTC)[reply]

You are correct in believing a pitot tube is a straightforward application of Bernoulli's theorem. The only error suffered by pitot tubes is alignment error when the axis of the tube is so far out of alignment with the oncoming flow that the pressure in the tube is less than stagnation pressure. I don't know what it means to say a pitot tube has a constant of 0.8. It certainly doesn't mean that your indicated airspeed at sea level is 0.8 (or even 1.25) times true airspeed. For type certificated aircraft there is a requirement that the airspeed indicating system must be calibrated in flight and the error may not exceed three percent or five knots, whichever is greater. (See FAR 23.1323(b))
There is an error in temperature probes used on aircraft because a probe causes the airstream to come to a stop (stagnate) and that raises the temperature of the air in the vicinity of the probe, leading to an error. However, temperature probes are not called 's' type pitot tubes so that doesn't explain the 0.8. If 's' stands for supersonic the 0.8 may be related to the fact that when the aircraft is flying at supersonic speed the pitot tube is operating behind a shock wave so the airspeed sensed by the pitot-static system needs to be processed before giving a meaningful indication of true airspeed. Is your 's' type pitot tube intended for a supersonic aircraft? Dolphin (t) 23:03, 28 June 2010 (UTC)[reply]

No there is nothing so technical about 's' type, it is just more suited for particle laden air than conventional 'l' type tubes. I agree with the alignment problem which may occur but that is something which is to be avoided while measuring and is not an intrinsic part of the tube. Certain sites give some sort of constant less than unity which must be multiplied to the velocity but I dont quite understand the logic behind that. Any sort of non-alignment issues can only give a constant greater than 1. Now the manufacturer does not seem too technically aware apart from the fact that there is 'some constant 0.8'. 0.8 incidentally is roughly what must be multiplied to the centre velocity to give average velocity in duct. But I dont think he is talking about that (and neither am I). So basically what is a pitot tube constant? Thanks —Preceding unsigned comment added by 122.175.68.41 (talk) 16:23, 29 June 2010 (UTC)[reply]

Contents of Cold Pack

Do cold packs only contain ammonium nitrate? I had one and reacted it with hydrochloric acid and it formed a precipitate of ammonium chloride, which is much less soluble than the nitrate. Nitric acid should be formed, but it didn't react with copper to form copper(II) nitrate. It turned brown when it was heated and formed a white precipitate. --Chemicalinterest (talk) 13:14, 28 June 2010 (UTC)[reply]

Are you talking about the gels that you pre-freeze before using, or the endothermic ones where you break&shake a room-temperature bag? Our cold pack article gives some possible materials for each one. DMacks (talk) 15:35, 28 June 2010 (UTC)[reply]
Break and shake. It contained a saturated solution of the chemical with a packet of water. --Chemicalinterest (talk) 16:38, 28 June 2010 (UTC)[reply]
At a guess, I would say that it's hydroxyethyl cellulose. It would be in there to increase the viscosity of the resulting solution, so that the cold pack sit more securely on whatever you're trying to cool. To see why that might be helpful, fill and tie-seal a smallish plastic bag with tap water and then try to balance it on your ankle... Physchim62 (talk) 21:03, 28 June 2010 (UTC)[reply]
Could be ammonium chloride rather than ammonium nitrate. You say you expect you started with an already (near-)saturated solution? You don't say how strong your HCl solution was, but it's easy to get HCl much more concentrated than NH4Cl, so when you mix them you boost the overall Cl concentration a bunch and NH4Cl might precipitate. Physchim62 has an interesting point that it might be some gelling agent rather than the actual "dissolves to cool" material. The ice packs I've used really do seem almost water-liquidy. But I'm not sure why adding (presumably) aqueous HCl to a water-solution of hydroxyethyl cellulose would cause it to precipitate, or if an aqueous acid solution of hydroxyethyl cellulose would turn brown and form a white precipitate when heated. Do you have access to silver nitrate? It's the classic test for chloride. Our qualitative inorganic analysis article has many of the classic near-definitive tests fir determining what ions you have (though some require chemicals you may not have access to). DMacks (talk) 14:22, 29 June 2010 (UTC)[reply]
My HCl was concentrated, about 12M. It was viscous, the solution was. the precipitate did seem to have similar solubility to NaCl, so it was probably NH4Cl. The hydroxyethyl cellulose may have pyrolyzed when heated in the superconcentrated solution.
I have no silver nitrate; I wanted to buy it but it was too expensive, $12.95 for a couple grams.
So it doesn't seem like it has ammonium nitrate. It did react with household bleach sodium hypochlorite though (more criteria). --Chemicalinterest (talk) 15:16, 29 June 2010 (UTC)[reply]
Whats the decomp temp of hy eth cellulose? --Chemicalinterest (talk) 15:21, 29 June 2010 (UTC)[reply]
The melting point (not decomp temp) is 140 Celsius. There's no decomposition temperature given, and in any case the stuff would hydrolyse rather than pyrolyse when heated in solution. FWIW 67.170.215.166 (talk) 01:07, 30 June 2010 (UTC)[reply]

documentary

I'm looking for good tv documentaries on ants, specifically the Black garden ant. I've already watched Life in the Undergrowth but if just skipped about from different species without giving any really detailed info. I want a documentary that follows the entire life cycle of the black garden ant. Thanks 82.43.90.93 (talk) 14:10, 28 June 2010 (UTC)[reply]

Do OLED displays degrade with UV exposure?

Do the organic compounds in OLED displays degrade with UV exposure? Should devices with such displays be kept out of direct sunlight? --70.167.58.6 (talk) 15:00, 28 June 2010 (UTC)[reply]

They do, but only in the really old OLED displays. The early OLEDs were made of PPV, which is easily oxidized by oxygen in the air, especially when catalyzed by UV rays. Most modern OLEDs, though, are made of other materials such as polyfluorenes, indium alloys, etc., which are stable under UV radiation. So if your device is really old, then it's best to keep it out of direct sunlight (and be careful not to scratch the protective coating on the screen); but if it's a new device, then it doesn't really matter. FWiW 67.170.215.166 (talk) 08:10, 30 June 2010 (UTC)[reply]

Most toxic element

What's the most toxic element in its pure form? Note that I said toxic rather than harmful, so radioactive elements like uranium and plutonium don't count. --76.77.139.243 (talk) 15:03, 28 June 2010 (UTC)[reply]

Can I infer that what you're really trying to say is most chemically toxic element? Because plutonium, for example, is quite toxic (in aerosolized form), and the fact that the toxicity derives from its radioactivity does not change that (see toxicity). Toxicity does not distinguish between mechanism, but we can, for the sake of argument, ignore toxicity related to radiation, if that is what you are asking about. --Mr.98 (talk) 15:12, 28 June 2010 (UTC)[reply]
Well, discounting radioactives (polonium would easily win if you include radioactives), and going on U.S. permissible exposure limits (PELs) for the element (discounting compounds), the answer is beryllium. Physchim62 (talk) 15:14, 28 June 2010 (UTC)[reply]
Yes, I was going to suggest beryllium, judging from Median lethal dose. Of course, toxicity is tricky—as our beryllium poisoning article points out, there are a whole set of different toxicities and effects depending on the route of exposure. But it's pretty nasty stuff, worse than arsenic and other "traditional" elemental poisons. --Mr.98 (talk) 15:17, 28 June 2010 (UTC)[reply]
PELs (and LD50s) are incomplete descriptors of toxcity, but it's noteworthy that the PEL for beryllium is 100-times lower than the PEL for elemental fluorine; that is, you're allowed to have 100-times more fluorine gas than beryllium dust in the air of a U.S. workplace. Physchim62 (talk) 15:44, 28 June 2010 (UTC)[reply]
There is also berylliosis which discusses the lung disease caused by beryllium. None of the articles explain why it is so toxic however, this paper says it inhibits enzymes that contain magnesium + calcium ions. It can also function as a hapten leading to apoptosis of macrophages in the lungs. 86.7.19.159 (talk) 16:12, 28 June 2010 (UTC)[reply]
See also the last time we talked about this, and other previous on the science and other ref-desks. Just type "most toxic element" into the ref-desk search-box at the beginning of the page. DMacks (talk) 15:39, 28 June 2010 (UTC)[reply]
Yes I remember asking a similar question a while back. One thing that the previous question brought up is that some elements are harmful if they are inhaled, but not all that harmful if they are eaten, since they are not absorbed into the body. So your question on which element is most toxic is going to depend on the manner of exposure. Googlemeister (talk) 16:19, 28 June 2010 (UTC)[reply]

High level nuclear waste

I thought all high level waste can be recycled and reused? But according to High level waste, it doesn't mention recycling anywhere in the article. Just disposal. 148.168.127.10 (talk) 15:20, 28 June 2010 (UTC)[reply]

Actually it does mention it quite a bit, but perhaps in a term with which you are not familiar: nuclear reprocessing. --Mr.98 (talk) 15:42, 28 June 2010 (UTC)[reply]
High-level waste is the highly radioactive waste material resulting from the reprocessing of spent nuclear fuel so the stuff is already recycled. Only the material which is not good for a nuclear reactor any more is disposed. There are highly radioactive elements not usable for fission especially the lighter elements produced by fission --Stone (talk) 15:53, 28 June 2010 (UTC)[reply]
The terminology seems to be: once you get it out of a reactor, it is spent nuclear fuel (SNF). If you reprocess that, the result is high-level waste (HLW). Now what's confusing is that our article on radioactive waste does not differentiate between the two even though they are quite different from an economic and a physical standpoint. I suspect this is because we are going by US definitions and the US is kind of muddy about such things, since it does not reprocess except for military purposes (and hasn't done that for awhile, I don't think), so we treat them as being basically the same from a waste perspective. In France, though, the distinction would be important, as they do civilian reprocessing. --Mr.98 (talk) 16:14, 28 June 2010 (UTC)[reply]

wormholes

I would like to win the nobel prize by building two wormholes, one leading to a very hot place (e.g. surface of the sun), one leading to a very cold place (lots are available), and use the heat difference to gain, in human terms, "free" energy. Also insofar as the wormholes are unstable and require energy, some of this energy could be invested in keeping them stable through some means. The key thing is that the wormholes don't have to be very big, now do they? An infitessimal point of high heat is enough to drive an engine, isn't it? Now, I would like to know how to make a wormhole, as I've looked on the Internet and no manufacturer sells such a solution, at any price. It will be very hard for me to find investors if I cannot even quote the price of what I am trying to acquire! Therefore, I would like to get a price estimate for building one very small, but stable wormhole. If it is in the billions of dollars, rather than a few hundred thousand or million, can someone explain why I have to pay so much, and can I somehow get around those billions by doing some of the work myself or in-house? In all, I would like to approach the subject much as an investment project in building a new power plant, however my real personal interest is in the Nobel prize. Thank you kindly for any help you may have to offer toward my goals. Very truly yours,
Philius Botsch —Preceding unsigned comment added by 84.153.206.127 (talk) 16:25, 28 June 2010 (UTC)[reply]

It cannot be done for any amount of money with current technology. Even if you had all the money of every intelligent race in the entire universe, you couldn't purchase a wormhole machine because they do not exist. -- kainaw 16:29, 28 June 2010 (UTC)[reply]
I did not mean in the lower-class sense of purchasing something on the shelves, on display, or in the catalogues, I meant in the bespoke sense of paying for the work that will lead to the realization. Of course, in the former sense, if currently no mechanical watch includes a hydrometer, then even a hundred billion dollars will not buy you one off of any shelf or out of any display on Earth or anywhere in the Universe. To a prince, of course, far less than a billion is needed if he is to have a mechanical watch with a hydrometer. It is in this latter sense that I ask how much it will cost. 84.153.206.127 (talk) 16:37, 28 June 2010 (UTC)[reply]
I believe Kainaw understood the intent of your question. There is no reason to believe that a way of creating and maintaining a stable wormhole can be developed for any amount of money. This is more like "building a watch that makes time run backwards" than "building a watch with a hygrometer." -- Coneslayer (talk) 16:44, 28 June 2010 (UTC)[reply]
I was under the impression that it was fully possible under the standard model of physics, ie you do not have to change physics if you are to have a wormhole. What are the mechanisms being proposed that would allow for the very wormhole standard physicists (such as Hawking, Greene, etc) talk about, and, engaging in a bit of blue-sky thinking, what are - let me put it this way - what sum has been paid for comparable achievements (physically allowed, however nowhere realized upon inception, and with uncertain prospects) that have in fact been brought to fruition? (quantum computing, etc, etc, etc). That might give me a baseline to base my calculations on. 84.153.206.127 (talk) 16:56, 28 June 2010 (UTC)[reply]
You are under a false impression. As I say below, you need to add exotic matter to the standard model if you are going to get stable wormholes and there is no reason to believe exotic matter is possible (and plenty of reason to believe it isn't - for example, wormholes can mess with causality in ways that a lot of scientists suspect is impossible). --Tango (talk) 17:02, 28 June 2010 (UTC)[reply]
Thank you for dispelling my false impression. In your estimation, in that model which is the standard model with the addition of exotic matter (a model that granted might not describe our universe) what mechanisms could cause a wormhole? (if you know) or, if you don't know, what methods are good candidates to potentially cause wormholes. Thank you kindly for your input. 84.153.206.127 (talk) 17:12, 28 June 2010 (UTC)[reply]
If you want a Nobel Prize, you're going to have to earn it. Generating free energy in the way you describe would probably be worth a Nobel Prize, but the key part is creating the wormholes. Once you can create wormholes, using them to make a heat engine is trivial by comparison. Current theories suggest that useful wormholes (ones that remain stable for long enough for something to pass through, including energy) can only exist if you have exotic matter, in particular matter is negative mass. At the moment, we have no evidence that such matter is even possible. We have no idea how to create it. If you can solve that problem, you'll probably get a Nobel Prize. --Tango (talk) 16:42, 28 June 2010 (UTC)[reply]
Thank you for your detailed response. When you say "we have no idea how to create [it]" where "it" might not be possible, can you tell me which candidate methods are in the minds of serious, credible physicists who would even write the words "useful wormholes" in succession? No idea = no candidate methods at all, if I list everything I can think of that is physically possible to do with the objects of the universe, they would reply to each one "not worth a thought"? 84.153.206.127 (talk) 17:10, 28 June 2010 (UTC)[reply]
Assuming you're not just trolling, which I suspect to be the case, then no such method exists to create a wormhole, because there is very good evidence they don't/can't exist. No amount of money will buy you something which is impossible. Further more, if you want the Nobel Prize, it's usually a prerequisite that you develop the technology and theory yourself or as a group, not asking others to do the work for you and then claim it as your own. Lastly, I suspect this sort of theory would obtain several Nobel prizes. One of discovering the existence of wormholes, another for creating one, another for using the system to generate free energy. Good luck to you, sir. Regards, --—Cyclonenim | Chat  16:57, 28 June 2010 (UTC)[reply]
Thank you for your detailed response. I believe you are asking me to make a jump from your justified reasoning "because there is very good evidence they don't/can't exist" for the statement "no such method exists to create a wormhole" to the unjustified statement "there is no candidate method to create a wormhole" on the same grounds. Non sequitur. Clearly, there is something that impels physicists to talk seriously of wormholes: in their blue-sky thinking, what mechanisms could even be candidates for creating these? Kind regards, PB 84.153.206.127 (talk) 17:10, 28 June 2010 (UTC)[reply]
Some physicists believe that wormholes can exist. Some do not. Some believe that they do exist. Of those who believe that can and do exist, they mostly all believe that wormholes only exists for less then a nanosecond and are extremely tiny. To get the science-fiction version of a wormhole, you need tons and tons of energy. You can't use real energy. You have to use "negative energy". This is the energy that is the opposite of real energy. You pump negative energy into the wormhole to blow it up like a balloon. As you'd expect, negative energy doesn't react nicely with real energy, so we are basically discussing an attempt to keep a violent explosion much more powerful than a supernova under control. Further, there is no proof that negative energy exists. There is no way to detect it or create it. There is no way to detect a wormhole or predict where one will be. There is no way to inject energy that you cannot create into a wormhole that you cannot find. There is no way to control the wormhole if you had the energy you can get and the wormhole you can't find. In other words, this is all theory. It is not reality. Your best bet is to fund the creation of a time machine to jump millions of years into the future and bring back their technology since we are much closer to time travel than wormhole creation and use. -- kainaw 17:19, 28 June 2010 (UTC)[reply]
Thank you, your response has finally brought home to me just what I have to do. 84.153.206.127 (talk) 17:34, 28 June 2010 (UTC)[reply]
Just so we're clear, what you "have to do" is give up. At present, this cannot be done, and unless you are the most brilliant scientist in existence (and sorry, but I do not get that impression), then you will not succeed in this monumental, probably impossible-at-present, task. Regards, --—Cyclonenim | Chat  17:55, 28 June 2010 (UTC)[reply]
Seriously, I am glad you guys were not talking to Gallileo or Newton when they were doing science. If the guy wants to try to discover and then create wormholes, I say go for it. Just understand that the odds of success are low (but not zero). Googlemeister (talk) 18:25, 28 June 2010 (UTC)[reply]
Galileo and Newton were both experts on the existing science that they ended up overthrowing (actually did Newton really disagree with the established position on anything?). The OP has much to learn about existing science before he can hope to create new science. --Tango (talk) 18:30, 28 June 2010 (UTC)[reply]
I talked with God - it took 45 minutes of prayer - and you guys are absolutely right. He told me that wormholes are an impossibility. Everything several of you said above is exactly right, and so I must abandon this avenue of both basic research and development. Thank you for bringing me to the point where I seriously questioned my premises and invested the time to get a definitive answer. I hope you will be as helpful the next time I have plans on an equal scale. Very sincerely yours, Philius Botsch. 84.153.206.127 (talk) 18:43, 28 June 2010 (UTC)[reply]
Thanks for the trolling. I really hope I'm not the only person who can see that. Regards, --—Cyclonenim | Chat  19:06, 28 June 2010 (UTC)[reply]
You are not alone, at the very least it would be good if people understood that volunteers on this desk do not do it out of a desire to satisfy someone elses every whim, or do 99.9% of their work for them. But hey! who knows?Sf5xeplus (talk) 21:23, 28 June 2010 (UTC)[reply]
Re: Newton: postulating gravity as an "occult force" was considered pretty scientifically and philosophically controversial at the time. The issue, basically, was that Newton proposed gravity as a force that acted at a distance for reasons he did not understand. This was considered kind of shady by people who considered themselves to be serious thinkers. It also didn't help that Newton basically thought that the entire universe would fall apart pretty quickly if God wasn't actively holding it together constantly. In the long run, though, Newton's laws work pretty dang well, even if you don't know what the big G "really means" in physical terms. Even today—well after Einstein gave us a better explanation of what is going on with gravitational force—we are still trying to come up with a wholly satisfying explanation (i.e. quantum gravity). So yeah... Newton was controversial, definitely disagreed with the established position on a lot of things (and not all of which he published on). --Mr.98 (talk) 19:51, 28 June 2010 (UTC)[reply]
What makes the above question amusing but not very encouraging is 1. the desire is to do something that is assumed will be so scientifically revolutionary that it would warrant a Nobel Prize, 2. the question asker happily admits he has no idea how this could be done, and 3. the question asker would like us to supply the means of doing it. This is not a serious endeavor. If we knew how to do it, and thought it would work, why wouldn't we do it and win the Nobel ourselves? We aren't running a charity here. Or any other scientist for that matter, including those who have spent their entire lives dreaming about the physics of wormholes. I'm not saying it can't be done, but you aren't going to do it the way you're trying to do it. If there is a Nobel to be won, it won't be won by asking us to do the work for you! --Mr.98 (talk) 19:54, 28 June 2010 (UTC)[reply]
Actually, we are running a charity here... --Tango (talk) 20:16, 28 June 2010 (UTC)[reply]
The goal of Wikipedia is to provide information. We provided it. What the OP does with it is his/her business. Some of our OPs are serious and legitimately curious individuals who make good use of the free service we provide. Some OPs are giggly adolescents who don't realize the amazing learning tool that they have been given free access to. That's fine; we provide high-quality responses anyway. The world has ways of sorting individuals for us, we don't have to waste our time doing that. All we do is provide scientific references. OP: read Physics, if you actually have even a slight inclination or interest. Nimur (talk) 21:37, 28 June 2010 (UTC)[reply]
If you have a large pile of money - you could spend it on paying a bunch of physicists to try to build you a wormhole - but that is an unbelievably risky thing. We don't know that wormholes can exist - if they can, we don't know whether or where any can be found naturally, we don't know how to create them, even if we had one, it would probably be smaller than an atom - and the only way to make it bigger might be to somehow (we don't know how) feed it with 'negative energy' - which may also not exist, not be manufacturable, etc, etc. Safe to say, the probability of getting any return on an investment is microscopically tiny. You have (maybe) a one in a trillion chance of getting a wormhole by spending (let's say) a trillion dollars in research. The other 999,999,999,999 times, you blow all the money and have nothing but a lot of REALLY interesting (but useless) research papers to show for your money.
You can make money far more reliably by (for example) investing in electric car design or fancy new solar panel designs. Even a relatively modest (millions of dollars - but not billions or trillions) would produce a really good probability of making a decent profit on your investment. The problem here isn't about how clever your initial idea is - it's about risk. SteveBaker (talk) 23:37, 28 June 2010 (UTC)[reply]

Battery chemistry labs

Other than UT Austin where are the best battery chemistry laboratories, who has been running them, and what are they known for? 208.54.14.26 (talk) 16:27, 28 June 2010 (UTC)[reply]

The Cui lab at Stanford University got a lot of press a few years ago for some nano-wire lithium ion battery technology. They are a materials-engineering and chemistry research group. You can read brief introductions to their resesarch here and a list of scholarly peer-reviewed papers here. Nimur (talk) 21:42, 28 June 2010 (UTC)[reply]

Photonic computers

Why aren't photonic computers commercially available? They're much smaller and faster than electronic computers, so there would seem to be demand for them since people continue to buy new, faster electronic computers. --76.77.139.243 (talk) 16:42, 28 June 2010 (UTC)[reply]

I trust you've read optical computing? The last paragraph in 'Misconceptions, challenges and prospects' discusses why it is not in widespread use just yet. Regards, --—Cyclonenim | Chat  16:48, 28 June 2010 (UTC)[reply]
They use more power, but aren't they still faster despite that fact? --76.77.139.243 (talk) 16:58, 28 June 2010 (UTC)[reply]
It's not just about power and speed. Electronic computers combine multiple signals to perform complex computational tasks. Photons don't interact with each other (in this case) and so you can't combine signals easily. Regards, --—Cyclonenim | Chat  17:53, 28 June 2010 (UTC)[reply]
I don't think there is sufficient motivation to develop the technology yet because we haven't reached the limits of what we can do with electronic computers. There are at least a few more generations of electronic technology to go before we need to worry about developing photonic computers and the development cost of better electronics is much cheaper than developing photonics from scratch. --Tango (talk) 17:05, 28 June 2010 (UTC)[reply]
Our optical computing article is full of glaring errors and scientific inaccuracies. I'll try to rework it over the next night. For example, "Light, however, creates insignificant amounts of heat, regardless of how much is used." This is totally incorrect. Radiative heating occurs at all frequencies of electromagnetic radiation. Other discussion about interference, photonic logic, and so on, are all in need of attention. I would not recommend this article as a source of information until it has gone through significant editing with a reliable source. I'm hitting the library tonight to get a book on optical computing. The article does not seem to convey that light is just a different frequency of electromagnetic waves; some different physical phenomena are more common, and different materials have more desirable properties; but nothing is fundamentally different between optics and lower-frequency electromagnetic waves. Nimur (talk) 20:09, 28 June 2010 (UTC)[reply]
But electronic computers use electric current, not electromagnetic waves. Rmhermen (talk) 20:43, 28 June 2010 (UTC)[reply]
That's debatable. Modern computers use electromagnetic signaling - current is a parasitic effect. If we could design transistors that switched without any transient current, many of our power-density problems would be solved. E.g. zero-current switching. We don't use the current for anything - it is like "friction" in an engine, and only serves to dissipate energy. The work of calculating is done by switching signal-levels. At the fundamental level, information theory dictates that we must dissipate some tiny quantity of energy in order to store information. (This is sort of the "2nd law of thermodynamics" as applied to information computation, [1]). But the overwhelming majority of the electric current in a modern VLSI system is there because our transistors are "leaky." See the Power section and the logic section of our CMOS article, for example. Nimur (talk) 21:06, 28 June 2010 (UTC)[reply]
So Nimur wants to debate with Rmhermen... zero-current switching is an emi reduction technique that has nothing to do with computing. It is not only information theory and leakage that dictate some current flow in switching signal-levels. Any real conductor has capacitance and a flow of current is needed to change the voltage (= binary logic level 0 or 1) on it. Cuddlyable3 (talk) 12:18, 29 June 2010 (UTC)[reply]
I do not want a debate. You can read how information is conveyed in CMOS logic at the CMOS article. I can recommend several good books in addition. Digital Design: Principles and Practices ([2]) has an entire chapter dedicated to introducing you to the electrical behaviors and signal pathways in digital circuits. CMOS RFIC will introduce you to a more quantitative analysis of the characteristics of a modern VLSI circuit's electrical properties. Yes, any capacitor requires a current in order to charge. And it must by necessity dissipate that current as a resistive loss, or else it oscillates and information is not stored on it. Right now, the state of the art is such that the actual current is much higher than it theoretically could be, due to additional parasitic losses and leaks in the device. (In a photonics circuit, that would be analogous to "dielectric losses" - which are even worse than the thermal losses at VLSI scales). Any technique that improves CMOS switching current will reduce dynamic power dissipation in a digital circuit, whether the signal is propagated by light-frequency or RF. At present, though, there is no "CMOS" for optical frequencies because there are no practical optical transistors. Instead, photonic computer research usually focuses on other mechanisms to store information, such as in the form of frequency modulation, using nonlinear materials to perform frequency mixing. These techniques are at present not sufficiently developed to build a VLSI system. That is the primary reason we do not have optical computers - they can not be built in a way that their device counts compare to RFICs. Nimur (talk) 16:31, 29 June 2010 (UTC)[reply]
Are we talking about fibre optic cables to transmit computer signals instead of using electricity? It would require electricity to produce the light photons. ~AH1(TCU) 22:12, 2 July 2010 (UTC)[reply]

Least dangerous compound

I noticed that even seemingly harmless compounds, such as sodium bicarbonate, don't have all 0 for their NFPA 704; is there anything which is ranked 0 in all three categories? --76.77.139.243 (talk) 17:09, 28 June 2010 (UTC)[reply]

Lotsa stuff. See for yourself. --Ouro (blah blah) 17:51, 28 June 2010 (UTC)[reply]
Water? --Chemicalinterest (talk) 22:00, 28 June 2010 (UTC)[reply]
I was surprised myself! --Ouro (blah blah) 05:19, 29 June 2010 (UTC)[reply]
Am I missing something? The above ref lists water from "Mallinckrodt-Baker" as having a 0 for all three Nil Einne (talk) 09:40, 29 June 2010 (UTC)[reply]
Our article on Properties of water had it down as 0-0-1. Checking the criterion for R=1, "Normally stable, but can become unstable at elevated temperatures and pressures", water is obviously R=0. I've changed the infobox. Physchim62 (talk) 09:57, 29 June 2010 (UTC)[reply]
Ah okay I checked the link to water and couldn't find anything about the NFPA from a cursory glance/search so gave up although I wondered if there was an article which covered the chemistry in more detail. From the earlier ref, it doesn't list any NFPA for "Water Deionized and Bacteria Filtered" from CMS. I've been thinking that may be part of the reason is some feel imparting a NFPA rating on water particularly for Instability/Reactivity is questionable since the definitions are partially based on the reactivity with water Nil Einne (talk) 11:50, 29 June 2010 (UTC)[reply]
However, it would not be entirely accurate to say that water is harmless. See water intoxication, dihydrogen monoxide hoax and flood. ~AH1(TCU) 22:09, 2 July 2010 (UTC)[reply]
Also drowning, storm surge, and tsunami. Am I forgetting anything? 67.170.215.166 (talk) 00:29, 3 July 2010 (UTC)[reply]

Applications of Wormholes

The last question got me curious about wormholes, and what they could potentially be used for. Assuming they exist, and they work, how many ways can they be used? The possibilities are enticing. Like what was mentioned before, they can be used as a heat engine, faster than light travel, time travel, weapons - I would assume sending a bomb through a wormhole would be pretty powerful, but you could also just open one mouth near a target on earth, then open the other in the vacuum of space, and you have a small black hole that sucks everything into it. 148.168.127.10 (talk) 17:46, 28 June 2010 (UTC)[reply]

By the word "applications", I assume you mean "worthwhile applications". Keep in mind that IF wormholes exist and IF we can find one and IF we had technology to keep one open so it could be used, it would take massive amounts of technology and energy to do so. So, if you wanted to send a bomb through one to blow up someone's city, you'd first have to smuggle in all the technology to hold it open on the city's end. Similarly, to open it to the vacuum of space, you'd have to smuggle in all the technology. It is the equivalent of shooting someone with a rifle by asking them to aim their end of the barrel while you pull the trigger on the other end. Further, if you had the technology in place to hold open a wormhole, you would have the technology in place to simply destroy whatever it is you want to destroy. -- kainaw 18:00, 28 June 2010 (UTC)[reply]
(ec) I'm not sure they would work well as weapons. I doubt you could just "open one mouth near a target". You would probably have to drag one mouth from your lab to the target, which makes it difficult to use as a weapon (you could just take a big bomb instead). Faster than light travel is the obvious application, although once again you would probably have to drag one mouth there at slower than light speeds, so the first journey to your destination would have to be by the slow route. Time travel is also a possibility, although it probably couldn't be used to travel back to before you created the wormhole. The heat engine idea would probably work - it's sort of like putting a solar panel right next to the sun, but you don't need to transfer the electricity back to Earth. Faster than light information transfer is probably another option - no more lag on satellite feeds. (You will notice that every idea I've mentioned has "probably" in it - the existence of stable wormholes requires some change in our understanding of physics and there is no way of knowing what else would change with it.) --Tango (talk) 18:05, 28 June 2010 (UTC)[reply]
you've smuggled your response to my heat engine idea down here, but I'm on the case. Just so you know, my proposal is for there to be TWO wormholes, and so you don't have to transport the electricity back to earth, because the heat exchange happens right in the Philius Botsch Power Center on Earth. In the power engine, you have both the other end of the "hot" wormhole (with its other mouth on the surface of the sun, or as far away from a sun as you require, e.g. in orbit around a sun at a certain (perhaps low) distance) and the other end of the "cold" wormhole (with its other mouth on some cold gaseous planet, for example). Then you just run the engine off of the heat difference. No interplanetary power transportation required! Very sincerely yours, Philius Botsch 84.153.206.127 (talk) 18:40, 28 June 2010 (UTC)[reply]
Yes, I understood the idea and, if you can get the wormholes, it should work. There is no real need for the "cold" wormhole - the Earth is cold enough. --Tango (talk) 19:10, 28 June 2010 (UTC)[reply]
with that mentality, you should work for an oil company! The reason for the cold wormhole is to stave off at least some of the global warming associated with the scheme. Kindly, T. Philius Botsch84.153.206.127 (talk) —Preceding undated comment added 19:15, 28 June 2010 (UTC).[reply]
You can use it for time travel. If you have one wormhole sitting there, and another in orbit, time dilation will cause them to slowly get out of sync. After a while, you can bring them close together, and put an optic fiber through it. If you send a signal through the fiber to the younger end, it will have come out of the older end a tiny fraction of a second. If you did it right, it will then go back to the other wormhole in a tinier fraction of a second, sending the signal slightly back in time. If you stick an amplifier along the optic fiber, you can send the signal arbitrarily far back. If you want to use it for energy, put one wormhole above the other, and drop something through it. — DanielLC 04:38, 30 June 2010 (UTC)[reply]
As a fan of the Command & Conquer series, I seriously wonder if the chronosphere (what, no article?) makes use of wormholes? 67.170.215.166 (talk) 08:20, 30 June 2010 (UTC)[reply]

asteroid mining

If hypothetically, there was an asteroid that was made of solid platinum, 99.9% pure, what is the largest ΔV for the break even point to send the platinum back using current technology?

For simplification, we can assume that we don't need to crash chunks of metal into earth, we have a nice space station in geosynchronous orbit. Also, despite bringing tons of platinum back, the price is not going to crash, and the mining equipment is already on site. Googlemeister (talk) 18:38, 28 June 2010 (UTC)[reply]

Assuming you throw out economics ("the price is not going to crash"), you can spend whatever you want to get that much platinum on hand and you'll still be able to buy half the planet. Say you get this asteroid at 1km across -- there are 1-2 million asteroids that size or larger in the main belt. That's 4 quadrillion cubic centimeters of platinum. 90 trillion kg of platinum. At $1500/oz, that's $4.5 quintillion USD of market value. By way of contrast, world GDP is $60 trillion USD. So spend whatever you like. No meaningful answer is possible. — Lomn 19:01, 28 June 2010 (UTC)[reply]
If you brought that amount back, the price of platinum would crash to that of for example water. 92.24.188.76 (talk) 19:27, 28 June 2010 (UTC)[reply]
I disagree that no meaningful answer is possible. For example, to lift things into low earth orbit costs somewhere around $10,000/lbm. that weight of platinum (if at $2,000/toz in reality is is like $1600/toz) is worth around $29,000, so obviously LEO is feasible, but the ΔV to get to mars would mean you can not put something on mars for $10,000/lb, it will cost more, so at what distance does that price equal that of platinum? Googlemeister (talk) 19:35, 28 June 2010 (UTC)[reply]
That seems to be a completely different question. The platinum asteroid is already in orbit; it needs negligible delta V to reach Earth. I don't see where a launch from Earth enters into the picture. — Lomn 20:37, 28 June 2010 (UTC)[reply]
(Edit Conflicts) No, similar question and not negligible, because as a first-order approximation*, if it takes X delta V to get something from the Earth's surface to LEO (or GEO, or Mars, or wherever), it will take the same delta V to get it from LEO (or wherever) to the surface: delta V is the difference in velocity between two different orbits (mathematically, everything is in an orbit) and therefore equals the change needed to get from one to the other, and is a scalar quantity independent of which direction the change is, so it will take the same amount of energy (hence, roughly, cost) to move a tonne of platinum from the Asteroid Belt to Earth as it would to move a tonne (of anything) from Earth to the Asteroid Belt. The article Delta-v budget may be helpful.
Since you've stipulated a return only to a GEO station (which presumably has some pressing need for vast quantities of platinum) you'll save the considerable delta V between GEO and Earth's surface (remember, there is a good deal of delta V even between LEO and GEO, and as Robert Heinlein famously said, LEO is "halfway to anywhere"), and need only consider that between the asteroid and GEO: this will depend heavily on the asteroids's initial orbit, and can be minimised by a carefully timed Hohmann transfer orbit plus additional necessary manouevres to allow for whatever degree of non-coplanarity is involved.
(*I say "first order approximation" because this ignores some cost of moving some of the fuel necessary for subsequent manoeuvres (see Tsiolkovsky rocket equation), the possible use of aerobraking, etc, but for this initial analysis it's close enough.) 87.81.230.195 (talk) 21:46, 28 June 2010 (UTC)[reply]
It takes a lot of thrust for a sustained time to get an object from the Earth's surface into a low Earth orbit. It clearly does not take a comparable sustained thrust to get it out of low Earth orbit. If there is an atmosphere at the destination, spacecraft do not "back down on the rocket" like in old space opera sci-fi films of the 1930's. (Granted, they did that on the Moon where aerobraking was impossible). All that is required is a brief and relatively weak retro-rocket burn for something in low Earth orbit to decrease the velocity by 1% or so. The object (space capsule or shuttle) then drops enough closer to the surface that air resistance slows it, while the ablative heat shield (on older Apollo capsules and such) or the insulating ceramic tiles (on the shuttle) heat up. A cargo mined from an asteroid would need to be sent on a carefully chosen approach to Earth for aerobraking to work. If the angle of approach is too direct, it will mostly burn up like a meteor. If it is too shallow, it will skip off on another trip through the solar system. If the angle is just right, and it has an ablative heat shield large enough relative to its mass, it will fall in a descending path partway around the Earth and land relative unscathed in the desert where it can be recovered. The Apollo missions returning from the Moon got to the ground this way. For a cargo of metal from an asteroid, a hard landing which does not hit someone or their property and does not bury it too deep in the ground would be a windfall. A parachute might not be necessary (at least not a huge one required for a gentle landing) if the payload is a hunk of metal. We have a very long history of using pass=bys of various planets to aim a spacecraft toward some other planet, and of achieving precise approach paths. No space pilot on board would be necessary or desirable. Since the cargo is starting a long way off, relatively small thrusters could make the midcourse corrections needed for a precise reentry trajectory in relatively short bursts. Edison (talk) 18:02, 29 June 2010 (UTC)[reply]
Which is why I specified a first order approximation that omitted considerations like aerobraking (by implication, between LEO and Earth, which, as the OP had already specified a return only to LEO, was not directly relevant to his question). The OP seemed to think there would be negligable delta V between an asteroidal orbit and LEO, which I suggest is incorrect and which I was trying to refute. 87.81.230.195 (talk) 21:13, 29 June 2010 (UTC)[reply]
Where are you getting that the asteroid is in orbit around earth??? Obviously if that was the case, then the question would not make sense. If the asteroid was in orbit around the sun at the same distance as mars, though the ΔV is going to be higher. At what ΔV does this become uneconomical? Googlemeister (talk) 20:55, 28 June 2010 (UTC)[reply]
I should have been more clear: the asteroid is in solar orbit. Orbital mechanics allow for absurdly efficient transfer orbits. Given the vast amounts of handwaving already present in your assumptions, you cannot find a breakeven point. On the other hand, given reasonable assumptions, you also can't find a breakeven point. It's nicely symmetric that way. — Lomn 21:10, 28 June 2010 (UTC)[reply]
You know, if you are going to be obtuse, I would rather you not answer. Googlemeister (talk) 21:20, 28 June 2010 (UTC)[reply]
The assumption of fixed price is seriously flawed. There is no need for 90 trillion kilograms of platinum. What would we do with it? Who would pay for it? How could you expect to sell all of it at current prices? That would be 10,000 kilograms of platinum for every member of the planet, with 10 trillion kilograms left over. How could you expect a reasonable market to exist for this quantity of metal? If you want to justify space travel economically, you can't throw economics out the window. Nimur (talk) 21:02, 28 June 2010 (UTC)[reply]
Obviously a fixed price is not accurate, but then the question would not be answerable because you could not tell me how much the price would be impacted by adding 50 tons, or 100 tons or 500 tons of platinum to the market could you. And I never said the asteroid was 9e16 kg. Googlemeister (talk) 21:05, 28 June 2010 (UTC)[reply]
You're right. Specific numbers are tangential, since this is all hypothetical anyway. But the point is, space travel is rarely justified in the same way as trade down here on earth. If bananas are cheaper on another continent, you can go buy them there and ship them here. But space travel involves going someplace where earth economics are meaningless - things only have "value" if they contribute to some objective. So, if your objective is "get lots of platinum", then you have to demonstrate that the best way to do that is space mining. It's pretty unlikely that any resource is easier to get in outer space than down here on earth - so mining for commodity metal is not a very good objective. Most of human space exploration has been justified in terms of expanding our understanding of the universe, not in terms of economic benefit (though some politicians justify space exploration because of the peripheral technological and economic development it does create here on earth). Nimur (talk) 21:28, 28 June 2010 (UTC)[reply]
Find a large chunk of the platinum and than build a very simple catapult on it an throw 1 meter size balls on trajectory which will collide with the earth. Some 50% will evaporate, but that is not a big problem. Cheap an easy. Calculate 10 more likely 50 Ariane 5 launches each 250 mio Euro and another 10000 mio Euro for the program to bring people there and back (mining is not a job for robots). This makes a lot of platinum you have to get back.--Stone (talk) 21:27, 28 June 2010 (UTC)[reply]
The question isn't "How much should I spend to bring back an entire platinum asteroid?" It's "If I'm harvesting some small portion of the asteroid, how much fuel can I afford per ounce of platinum? And is that enough to bring that ounce of platinum home?" APL (talk) 02:00, 29 June 2010 (UTC)[reply]
What complicates this stuff is that some materials which are essentially worthless on earth (air, water, dirt) are phenomenally valuable when you put them someplace else. A ton of air can be had for free down here on earth - but just 200 miles away in low-earth-orbit, at $10k per pound of launch costs, that same ton of air is worth $20 million. Just taking an icy asteroid and smacking it into the equator of the moon or Mars - or getting it into any kind of stable earth orbit would produce a resource that would make a spectacular difference to the future of humans in space. In that sense, the mere fact of where you place this huge pile of metal is more important than what it's made of. Platinum is a bit of a pain to work with, and it's kinda heavy. Aluminium, water, oxygen or even reasonably fertile dirt would probably be worth more in those kinds of quantities because while platinum's value is due to it's rarity (which you're about to destroy), the value of other materials is due to the amount of fancy technology and rocket fuel you're saving by not shipping it up from Earth - and that's something whose value would be much harder to erode. SteveBaker (talk) 23:22, 28 June 2010 (UTC)[reply]

The article Asteroid mining might help. Cuddlyable3 (talk) 11:56, 29 June 2010 (UTC)[reply]

Getting rid of freezer odour

A couple of weeks ago I had the unfortunate experience of finding out my freezer had accidentally become unplugged (thanks to the stupidly short mains lead) and the contents had been in there rotting for at least a week. I just about managed to bag and bin the contents without being physically sick (the smell was so bad it would have offended the devil himself) and have cleaned the thing - thoroughly - several times, with bleach, disinfectant and various other such products but to no avail. The smell was on my hands for days and even white spirit (which is usually pungent enough to mask any other smells) only brought a temporary respite.

I then put the freezer outside with the door off and left it there for 2 weeks (no rain luckily) and there is still a conspicuously unpleasant odour, although a shadow of its former self. Is there any way I can exorcise this satanic stench permanently or is it best to get rid of the freezer? —Preceding unsigned comment added by 94.197.153.113 (talk) 19:00, 28 June 2010 (UTC)[reply]

You're probably going to have to partially take apart the freezer. There might be some rot hidden behind some of the plastic panels. Once you remove all the bulk rot material you'll want to force the bacteria to finish the job - i.e. get the rot going at high speed, make the freezer warm and humid, but aerobic (i.e. let oxygen in). Also, once you turn it on the cold temperature will probably stop the smell. And next time (hopefully not :) buy a gas mask in a hardware store. Ariel. (talk) 19:11, 28 June 2010 (UTC)[reply]
(ec)My freezer also failed while it was loaded with fish. I filled a window cleaner spray with undiluted chlorine bleach, sprayed every internal surface including the seals and latch, then shut the door for a day. After what had gone before, the lingering slight chlorine smell was a blessed relief. Cuddlyable3 (talk) 19:13, 28 June 2010 (UTC)[reply]
I agree that you need to let the bacteria finish the job they started. They are breaking the remaining food residue down and making the stink in the process - but eventually, they'll run out of stuff to decompose - and then you're done. The more stuff you can remove yourself, the faster that'll happen. Pay particular attention to small crevices, screw heads and places like that. The large, flat, smooth surfaces are easy to clean. When something similar happened to me, I found that leaving it outside (opened) and in sunlight helped.
Oh, and at the risk of sounding obvious, may I suggest getting an extension cord so that the same problem doesn't occur in the future? After all, you said that the freezer got unplugged because of a "stupidly short mains lead" -- sounds to me like an extension cord would fix that for good. 67.170.215.166 (talk) 00:59, 29 June 2010 (UTC)[reply]

On swallowing food items larger than one's own head...

Just saw this on YouTube. How long do you reckon it takes to digest that? How is there even room in the digestive system to hold and move that through? --Kurt Shaped Box (talk) 19:38, 28 June 2010 (UTC)[reply]

"this", btw, is a kookaburra eating a rat. --Tagishsimon (talk) 21:43, 28 June 2010 (UTC)[reply]
Don't know much about kookaburras, but I know a different species which can do that, too. --Ouro (blah blah) 05:33, 29 June 2010 (UTC)[reply]

NFPA 704

Why does sodium bicarbonate have a 1 in toxicity? People eat it all the time with no ill effects. --75.25.103.109 (talk) 19:59, 28 June 2010 (UTC)[reply]

I think that the solid can cause ill effects by reaction with gastric acid. --Chemicalinterest (talk) 20:03, 28 June 2010 (UTC)[reply]
Looking at the International Chemical Safety Card here, it could be because the powder is irritating to the eyes. Physchim62 (talk) 20:20, 28 June 2010 (UTC)[reply]
I don't know if this is related and it's been a while and I can't find any sources but I believe there are or were some regulations for labs which also oddly enough end up covering sodium chloride in NZ which impose some requirements or restrictions if you store it in large quantities that relate to the fact as with many things it is obviously toxic if you eat too much and while the level is fairly high (I think I've heard 500g), it's not considered high enough that you don't have to worry about it. This has unsurprisingly sometimes the subject of ridicule since someone in a fish and chips shop has no such requirements surrounding how they store their sodium chloride. Also from NFPA 704 "Exposure would cause irritation with only minor residual injury (e.g., acetone)" it doesn't sound like eating is the only consideration here as Physhim mention you don't generally want it on your eyes and for that matter open wounds or whatever. Nil Einne (talk) 11:40, 29 June 2010 (UTC)[reply]

evolutionary history of apoptosis and vertebrate immune systems

What was the precursor to the vertebrate immune system and its diversity? What did the very first white blood cells look like? (They seem a bit renegade...did they evolve from cells that had a little of a rebellious streak?) Also what were the predecessors to apoptosis? John Riemann Soong (talk) 21:38, 28 June 2010 (UTC)[reply]

Lot of questions - I'll try my best! Firstly, have you read Immune_system#Other_mechanisms? I'm not sure about the evolution of white blood cells, this might help though or other papers here. This discusses the evolution of apoptosis. Interestingly there are some analogies between plant and animal immune responses - systemin activates MAPKs in turn releasing jasmonic acid from the membrane, a similar system causes the production of prostaglandins in the mammalian inflammatory response (see this). Although systemin is only found in the Solanaceae, similar peptides have been found in Arabidopsis. 86.7.19.159 (talk) 23:37, 28 June 2010 (UTC)[reply]

anywhere I can buy movement protein?

I'm also using more professional sources, but I thought I'd try for a quick answer here...does anyone know if tobacco mosaic virus movement protein will work on garlic or onion cells (or similar edible rootlike tissues) and where I can buy it? John Riemann Soong (talk) 21:41, 28 June 2010 (UTC)[reply]

This paper discusses alfalfa mosaic virus movement proteins moving through the plasmodesmata of onion epidermis cells. I take it that this collection in Wageningen would have it, it would cost $75, or more likely be free. 86.7.19.159 (talk) 22:46, 28 June 2010 (UTC)[reply]
Onion cells ... exactly what we are using. (It's why I brought my potato peeler and various random vegetables to work today... I love this project.) THANKS. John Riemann Soong (talk) 22:49, 28 June 2010 (UTC)[reply]
No worries, this might be a better option, now that I've noticed you're in the US. 86.7.19.159 (talk) 23:41, 28 June 2010 (UTC)[reply]
Uhh can't seem to find a specific entry or price. My supervisor recommended me some large protein company, but I can't seem to remember the name. John Riemann Soong (talk) 16:17, 29 June 2010 (UTC)[reply]

Ancient lead

This story says that nuclear physicists are stoked that a cargo of lead ingots was found on the floor of the Mediterranean after having lain there for 2000 years, because almost all of the lead-210 has decayed by now, so they're going to use it to shield the "CUORE" neutrino experiment (our redlink article from the "Cuore" disambig page is Cryogenic Underground Observatory for Rare Events) and are happy to have some lead for this purpose that isn't going to emit any radioactivity. Why is this lead particularly different from any random amount of lead ore that was mined yesterday? Is the problem that metallurgists are unable to get "pure" lead of some stable isotope, and some heavier elements are always in the lead and eventually decay into lead-210? Comet Tuttle (talk) 23:16, 28 June 2010 (UTC)[reply]

Yes, that is the problem. It is very difficult to separate different isotopes of the same element because you have to use physics rather than chemistry. People 2000 years ago were able to purify their ore into nearly pure lead and all the radioactive isotopes eventually decayed away. We can also get pure lead, but it will contain lots of isotopes, and we can't easily remove the radioactive ones (which wouldn't have been lead 2000 years ago, which is why people then could remove it). --Tango (talk) 23:39, 28 June 2010 (UTC)[reply]
From this fact sheet from the Argonne National Laboratory, the culprit appears to be radon-222, which is a decay product of uranium-238. Rn-222 has a long enough half-life to spread small amounts of lead-210 (and hence polonium-210) quite widely in the environment, so I would imagine there would be a particular seeding of deposits of lead ore (often associated with uranium-containing minerals). Physchim62 (talk) 23:44, 28 June 2010 (UTC)[reply]
I've just noticed that the reader comments to the Physics World article give the same suggested answer (radon-222). Physchim62 (talk) 23:52, 28 June 2010 (UTC)[reply]
Thank you! Comet Tuttle (talk) 05:27, 29 June 2010 (UTC)[reply]
Ref Desk in the past has discussed how scientists also like to use steel from pre-1945 battleships as shielding, since after the first atomic bomb detonations, refined iron somehow contains works less well as shielding. It is surprising that in an era before modern chemistry or metallurgy they were able to refine "pure" lead, when all they had was empirical rules of thumb and superstition as guides, with no real way to chemically analyze the result. Edison (talk) 17:35, 29 June 2010 (UTC)[reply]
I have the same idea about how savages, such as chefs at four-star restaurants, can cook at all, let alone well, given that most of them don't have even a grade-school, or nineteenth-century understanding of the chemistry involved. 92.230.66.154 (talk) 20:54, 29 June 2010 (UTC)[reply]
Irrelevant. Do the cooks there do a lot of lead refining? Remind me to stay out of that restaurant.Edison (talk) 19:48, 30 June 2010 (UTC)[reply]
Galena is relatively easy to smelt compared with most metal ores. Unlike most sulfide ores, you can get the sulfide itself to act as the reducing agent under the right conditions. As for purity, it would be interesting to see what sort of purity they managed; but they would have been aiming for particular metallugical properties, not chemical purity as we understand it today. Physchim62 (talk) 22:36, 29 June 2010 (UTC)[reply]


June 29

Polaroid film powered radio...

As a boy (perhaps 20 years ago), a relative gave me a small blue radio that was powered by Polaroid film. You popped a cartridge in and it powered right up. Aside from the in-hindsight ridiculous wastefulness (and likely, terrible expense) of the device, I find myself now wondering how exactly it drew power from the film. Has anyone ever seen one of these? Any ideas? 218.25.32.210 (talk) 00:59, 29 June 2010 (UTC)[reply]

As our Polaroid film article describes, such film cartridges often contained a thin battery. -- Scray (talk) 03:09, 29 June 2010 (UTC)[reply]
I had a Polaroid instant camera when I was a kid. It had a flash but no batteries in the camera body. The batteries were in the film cartridge and would power the flash if needed. Dismas|(talk) 10:44, 29 June 2010 (UTC)[reply]
The polaroid film battery by virtue of its slim form can be a component of a Letter bomb. Cuddlyable3 (talk) 11:41, 29 June 2010 (UTC)[reply]
The Polaroid Corporation introduced the Polaroid SX-70 in 1972. The camera was automated and highly motorized, so it was not necessary to pull out the film pack for each picture as in earlier Polaroid cameras. There was a fresh "Polapulse" six volt battery included in each filmpack, so there was not the "60 second disappointment" of weak batteries preventing the consumer from using up a film pack at about a dollar a picture (big bucks in 1972). The [3] was a flat lightweight zinc--chloride thin-film battery in a thin insulated pack, with two exposed metallic contact areas. It had pretty low internal resistance (so the motors would run smoothly). It was developed by Rayovac, and measured 3.5 by 2.75 inches, by 1/8 inch thick. (8.9 cm x 7 cm x 0.3 cm.) It was easily removed from the spent film pack after the 10 or so exposures were used up. It usually had lots of energy left in it. It could be connected via clip leads or wires taped to the metallic contacts to power a 6 volt flashlight or other device for quite a while. It possibly deserves its own article, since it had significant coverage in many books and magazine artilcles. Alkaline batteries today probably fill the niche it once, but it had a very high peak current capability of 26 amperes, decreasing to a still impressive 5 amperes after 30 seconds and 2.5 amperes after 60 seconds. and seemed able to supply high current for a long time without pooping out, from a very low volume and weight battery. It used conventional LeClanche carbon zinc technology, so it had that handicap to overcome compared to alkaline. I would be interested to see what a similar flatpack with alkaline battery or rechargeable lithium ion chemistry could do for applications where weight and size are at a premium. Metal cylinder batteries like AA or AAA are not always a good fit in designs for miniature devices. Edison (talk) 17:26, 29 June 2010 (UTC)[reply]
Take a look at the battery in an iPhone, Edison. --203.22.236.14 (talk) 09:38, 30 June 2010 (UTC)[reply]
There is no info in the article about its size or weight, or its current capability, except to note that it is not easily replaceable and has disappointing performance. Edison (talk) 19:46, 30 June 2010 (UTC)[reply]

Double slit prob

Someone performs the double slit experiment. The probability that the electron passes through the first slit is 1/4. The probability that the electron passes through the second slit is 1/5. What is the probability that the electron passes through either slit? 1/4 + 1/5 = 9/20?––115.178.29.142 (talk) 01:26, 29 June 2010 (UTC)[reply]

if they are independent, then with 3/4 probability it will miss the firstslit. Also, it has to miss the second slit, which it does with 4/5th probability. If you want to roll two sixes independently, your chances are 1/6 * 1/6, if you want to miss both slits it is 3/4 * 4/5 = 12/20 = 3/5. So you don't go through either 3/5 of the time, then the rest of the time, 2/5 of the time, you must go through either. 92.224.207.131 (talk) 07:39, 29 June 2010 (UTC)[reply]

The issue that the original math misses is that of that 1/4 of the time when it does pass through the first slit, it also passes through the second slit 1/5 of that time. Isn't the whole point of the dual-slit experiment to demonstrate that an electron has wave-like qualities and go through both slits? These are two overlapping/nonexclusive sets of event probabilities (the electron can go through first and/or second), not additive. Say I have 20 scoops of ice cream, of which 1/4 (5) are chocolate and 1/5 (4) have peanuts. That's not 4 of the ones that aren't chocolate--by the strict probabilities, there is one scoop that is chocolate-with-peanuts. So there are 8 not 9 that are [chocolate and/or peanuts]. DMacks (talk) 14:04, 29 June 2010 (UTC)[reply]

The question is insufficiently precise. What circumstance do the 1/4 and 1/5 measure? If they measure the probability of going through slit A given that slit B is covered, and vice versa, then the question will make sense, but it is still not answerable in general. The interpretation of the double slit experiment only makes sense if 100% of the time that the electron appears on the far side of the screen it went through both slits and their wave functions interfered. In general, the probability that the electron is observed on the far side of the screen at all depends on that interference which occurs after the slits (which is just how weird quantum mechanics is). So the short answer is that the probabilities are not additive, and one needs further details about the configuration in order to determine the probability of observing the electron on the far side of the screen. Dragons flight (talk) 18:25, 29 June 2010 (UTC)[reply]

The inteference term should vanish upon integration. By the time the electron is at the screen, we can write the wavefunction foirmally as:
|psi> = |psi1> + |psi2>
where |psi1> and |psi2> are the (unnormalized) states correspond to the electron moving through hole 1 and 2, respectively. Then we have an interference term for the probability as a function of the position on the screen:
P(x) = |<x|psi>|^2 = |<x|psi1>|^2 + |<x|psi2>|^2 +
2 Re[<psi1|x><x|psi2>]
The last term is the inteference term. However when we integrate over x, tis vanishes because |psi1> and |psi2>, being different eigenstates of the observable that measures through which hole the electron went, are orthogonal. Count Iblis (talk) 19:06, 29 June 2010 (UTC)[reply]

Einstein twin paradox confusion

What is wrong with my logic here?

  1. An astronaut checks her watch and then leaves Earth at some huge percentage of c.
  2. Rather than go out in a straight line, she orbits the Earth, always keeping it in sight.
  3. After her watch advances a few hours, she returns to Earth and lands.
  4. She finds that three months have passed on Earth.
  5. But if the Earth was always in sight, wouldn't she have noticed that the Earth had gone a quarter of the way around the sun?

Sorry, I'm sure this is elementary Einstein physics, but I just don't get it. Is this different than the typical "astronaut-leaves-earth" scenario because my astronaut is not going in a straight line? So is actually in several different inertial frames? Wknight94 talk 01:27, 29 June 2010 (UTC)[reply]

Well, the astronaut isn't in an inertial frame since you have to accelerate to go in a circle, but that isn't an issue since the astronaut in the original version isn't in an inertial frame either since they turn around and come back (that's the solution to the paradox). The time dilation is compensated for by length contraction, so each observer sees everything as being consistent. I'm not exactly sure what would happen in this case (non-inertial frames are complicated), but if you do the maths it will all work out right in the end. I guess the astronaut will see the Earth's orbit as much smaller than the Earth sees it, so it isn't at all odd for it to do a quarter of an orbit in a few hours. The orbit ought to change shape as the astronaut changes direction, though, so I'm not exactly sure how it would work. --Tango (talk) 01:44, 29 June 2010 (UTC)[reply]
If the astronaut went fast enough - in a circle - and waited long enough, billions of years could go by and the sun would burn out, etc., right? Wouldn't the astronaut see all that in the short time she experiences? Wknight94 talk 03:20, 29 June 2010 (UTC)[reply]
The answer is yes, if this thing somehow happened, she would have noticed that the Earth had gone a quarter of the way around the sun. If she got up to a sufficient fraction of the speed of light, she would see the Earth whirling around the sun at a furious pace. I don't see any contradiction anywhere here. Looie496 (talk) 03:31, 29 June 2010 (UTC)[reply]
Just a note though, if you are close enough to actually see earth, then the acceleration from traveling at that speed in a circle is going to kill you. You need to have a radius of 1 ly in order to keep it down around 1g, but of course you can't go around in a circle that big. Googlemeister (talk) 13:51, 29 June 2010 (UTC)[reply]
What if you had a really thick mattress? Heh, thanks. I guess the ridiculous-sounding result stems from my ridiculous scenario. Wknight94 talk 14:20, 29 June 2010 (UTC)[reply]
Actual observations of physical phenomena do indicate that things progress at unseemly accelerated rates. For example, the decay constant for a relativistic muon observed from an observer on Earth is "not correct" as a result of its high velocity - the infamous Muon Experiment - in fact this is a common lab experiment for physics students. See Muon Lifetime or Measurement of Muon Lifetime and Mass Using Cosmic Ray Showers. Decay constants are a little harder to put in "layman's terms" than, say, seasonal changes on Earth - but yes; we actually can see that the rate of passage of time differs because of the relative relativistic speed of the observer and the muon. If you were moving relatively to the Earth with relativistic velocity, you would similarly see an actual change in the rate that things occur - things like how quickly seasons change, how quickly the Earth orbits Sun, and so on. Nimur (talk) 23:19, 29 June 2010 (UTC)[reply]
This can be understood with General relativity. If she orbits the Earth at this rate, she must accelerate towards it very quickly at all times. This causes time to pass faster on the Earth, so she won't see anything unusual about the Earth clocks moving so fast. She spends half her time accelerating away from the sun, but she's further away from it when she's accelerating towards it. As such, it moves forward in time faster when she's accelerating towards it than backwards when she's accelerating away, so she won't see anything unusual about it burning out so quickly. — DanielLC 04:20, 30 June 2010 (UTC)[reply]
For the astronaut to notice the Earth orbiting around the Sun, she would need a frame of reference other than the Earth and Sun, so she would have to notice the positions of other planets and quite possibly the moon to realize how much time has gone by on Earth. ~AH1(TCU) 21:59, 2 July 2010 (UTC)[reply]

Sumo

Why don't sumo wrestlers have heart problems? --75.25.103.109 (talk) 02:24, 29 June 2010 (UTC)[reply]

First, perhaps we should ask whether sumo wrestlers have heart problems at a rate higher than Japanese men at a similar age? If you have statistics at hand, please cite them. -- Scray (talk) 02:43, 29 June 2010 (UTC)[reply]
There is actually some literature on this. Apparently the story is that because they combine high levels of exercise with high levels of calorie intake, they develop a lot of subcutaneous fat but not much of the more harmful visceral fat; see PMID 7859591. Even so they have higher levels of hypertension than the Japanese public as a whole, and after they retire, their levels of visceral fat and heart disease skyrocket. Looie496 (talk) 03:46, 29 June 2010 (UTC)[reply]

"Chronical outfit of disposition" - jargon or gibberish

In the article 78391 Michaeljäger an editor recently added the statement "Its chronical outfit of disposition, defined through the Gödel metric, is 0.4206." Is this jargon or gibberish? If the former, what does it mean? -- Tom N (tcncv) talk/contrib 02:36, 29 June 2010 (UTC)[reply]

Well, since the reference makes no mention of anything remotely connected to it and "outfit of disposition" returns zero ghits, I vote for vandalism. Clarityfiend (talk) 02:51, 29 June 2010 (UTC)[reply]
Well, my google search found a couple more Wikipedia articles where similar statements have been added: HD 171028 b and 1801 Titicaca (reverted, but Google hasn't updated their index yet). Since the Goedel metric is to do with clouds of dust in General Relativity (and asteroids and extrasolar planets aren't made of dust - there is significant EM interaction) and "chronical outfit of disposition" doesn't mean anything that I can find, I'm inclined to agree that it is nonsense. It's good nonsense, though! I've removed it from the two articles I can find it still in. If anyone can provide a reference for it making any sense, feel free to revert me. --Tango (talk) 02:57, 29 June 2010 (UTC)[reply]
This IP editor made about a dozen changes to random articles, and they are all clearly vandalism, and have all been reverted as far as I can tell. End of story. Nothing to see here. Move along. Looie496 (talk) 03:35, 29 June 2010 (UTC)[reply]
Maybe they're hoping to get a job as a Star Trek writer? Nil Einne (talk) 09:28, 29 June 2010 (UTC)[reply]

Blended wing bodies

Are Blended wing bodies stealthy? --The High Fin Sperm Whale 05:50, 29 June 2010 (UTC)[reply]

The article cited mentions the the B-2 Spirit stealth bomber. Cuddlyable3 (talk) 11:29, 29 June 2010 (UTC)[reply]
That still doesn't really answer the question, since the B-2 is only BWB-ish. --The High Fin Sperm Whale 16:52, 29 June 2010 (UTC)[reply]
Along similar lines, it's fair to say that blended wing bodies are stealth-ish. Certainly blended wing concepts can be used in a stealth aircraft (the aforementioned B2), but stealth aircraft can also forego blended wing concepts (the F-117A). Similarly, it would be very easy to make a non-stealthy blended wing aircraft (the exposed engines on the X-48 certainly fit the bill). Throw in that "blended wing" is a term that doesn't appear to be rigorously defined, and I'm not sure that a "yes" or "no" can possibly be provided to the original question. Even "stealth-ish" is fairly useless. By the same arguments, flying wings are also stealth-ish, as are more traditional designs. — Lomn 18:56, 29 June 2010 (UTC)[reply]
Was the YB-49 "stealth-ish"? 67.170.215.166 (talk) 01:10, 30 June 2010 (UTC)[reply]
Yes, it was. --The High Fin Sperm Whale 01:14, 30 June 2010 (UTC)[reply]

Are dogs capable of love?

I have two of the sweetest, most wonderful dogs in the world. Do they love me, or am I just projecting human emotions on them? A Quest For Knowledge (talk) 10:04, 29 June 2010 (UTC)[reply]

Definitely. Love is strong affection and attachment to someone or something. Dogs and many other animals are capable of love. 82.43.90.93 (talk) 10:35, 29 June 2010 (UTC)[reply]
Your opinion, anon. --Chemicalinterest (talk) 10:40, 29 June 2010 (UTC)[reply]
A dog is by ancestry a pack hunter. It's emotions are better viewed in terms of Pair bond and pack dominance and subjugation than human emotions. However it's ok for us to love them. Cuddlyable3 (talk) 11:20, 29 June 2010 (UTC)[reply]
Love is a very vague term. Since you're on the science desk I suggest you read chemical basis for love, and of course puppy love, and Cupboard Love. And how about Greyfriars Bobby?--Shantavira|feed me 11:15, 29 June 2010 (UTC)[reply]
Or Hachikō... Physchim62 (talk) 11:22, 29 June 2010 (UTC)[reply]
Rolls eyes skywards. Reading puppy love tells nothing about dogs. Cuddlyable3 (talk) 11:25, 29 June 2010 (UTC)[reply]
Get a cat. Then you know that that feeling is simple condescension with a bit of pity... --Stephan Schulz (talk) 11:51, 29 June 2010 (UTC)[reply]
There's actually a long debate between people who argue that dogs really like humans, and those who say, "oh, they've just evolved to simulate what humans consider love, in order to get fed." I tend to think that if it looks like love, it probably is love, with full reference to the fact that we don't have a strong scientific definition of "love" that isn't muddled down by centuries of cultural baggage. My dog seems to genuinely enjoy my company even when the food supply is not an issue. --Mr.98 (talk) 11:50, 29 June 2010 (UTC)[reply]
I have a good pet named Peacy. My Peacy will do anything in its power for me and will never get tired. It can sing for hours straight without a murmur. It never falls asleep when I am playing with it. It is vulnerable to viruses though; an occasional shot of anti-virus should help that. It eats a food known as Electric Power Pet Food. It drives hard for perfection by means of an internal hard drive. (WARNING: Do not take this content seriously). --Chemicalinterest (talk) 12:47, 29 June 2010 (UTC)[reply]
Right, but we understand exactly how a machine works in this respect. We don't really understand an animal quite so well. And at some level, inquiring into the "does this other highly evolved mammal really love/think/whatever" gets into a question of whether we "really" do any of those things either, and what one really means by that. I haven't seen any good evidence to suggest that dogs are incapable of such things. Dogs do seem to go fairly "above and beyond" what would be required for a minimal level of opportunism, but there's no way to quantify that. --Mr.98 (talk) 13:55, 29 June 2010 (UTC)[reply]
Its mostly a matter of opinion. We can't understand dogs' feelings (whether they have them or not). --Chemicalinterest (talk) 14:07, 29 June 2010 (UTC)[reply]
But there seems little reason to assume they don't have feelings, which would seem to be the harder case to argue. We know that we have feelings, whatever that means, and we see fairly similar behaviors in dogs (excitement, eagerness, friendliness, moping, shame), though we of course don't know if these are "real" feelings or if they are either conditioned reactions to our expectations or just projections. The "they just fake it" argument seems rather unproven to me, the kind of cynical response that people give because they don't like the fact that people seem to get enjoyment out of pets. The "projections" argument seems highly possible and indeed there must be a lot of projection going on, but considering that some of these behaviors seem extremely ingrained to both dogs and wolves ("play" behavior, apparent "joy" at meeting a sibling, "sulking" behavior), it seems unlikely to me that they are totally manufactured by humans. It seems to me to be a larger stretch to posit that dogs lack feelings than it does to say that they have them in some form or another (and obviously there has got to be a lot of variation in the "some form or another"). And again, at some point, depending on how we define these things, it gets tricky to even say whether humans act the way we idealize as (is there really "love" between humans that is not just rooted in some kind of evolutionary drive?). --Mr.98 (talk) 15:01, 29 June 2010 (UTC)[reply]
Since evolution selects for a reasult whether or not the subject has real feelings or not, this suggest that the functional AI hypothesis is correct. The moment you have a system that acts as if it has feelings, it has real feelings. Otherwise it would be huge accident that 4 billion years of natural selection happens to have led to people with real feelings instead of zombies that act as if they have real feelings. Count Iblis (talk) 15:12, 29 June 2010 (UTC)[reply]
Yes, dogs are absolutely capable of love, because they are fuzzy, cute, playful and have oxytocin in their brains. TheGoodLocust (talk) 23:04, 29 June 2010 (UTC)[reply]

free radicals

Do free radicals actually cause cancer, or is this just a myth that I've heard perpetuated enough times to sound plausible? If so, what kinds?

Thanks - flagitious (talk) 10:55, 29 June 2010 (UTC)[reply]

I accidentally double posted, can someone delete one of these for me - I don't know how. flagitious (talk) 10:58, 29 June 2010 (UTC)[reply]
Done. See Radical (chemistry)#Free radicals in biology. The bits about the harmful effects are all unsourced...which is unhelpful. Vimescarrot (talk) 11:07, 29 June 2010 (UTC)[reply]
Thanks. Yeah, I read that and I was really wondering because the section seems to be screaming [citation needed]. To the google I go! flagitious (talk) 11:58, 29 June 2010 (UTC)[reply]
There's also Reactive oxygen species which does contain some sources. Essentially free radicals will react randomly with DNA in a cell which could potentially damage important genes (like p53) which normally kill cells with damaged DNA through apoptosis. If the damaged cell can't be killed then they can potentially grow into a tumour. 86.7.19.159 (talk) 12:08, 29 June 2010 (UTC)[reply]
Free radicals produce reversible damage to body cells. Normally your body can heal the damage. If there is too many, it can form tumors and other malfunctions of the cell. They contribute to cancer risk, but they probably don't cause it. Antioxidants or reducing agents absorb the shock of the free radicals themselves, protecting your body. --Chemicalinterest (talk) 13:06, 29 June 2010 (UTC)[reply]

However, if some random skin or health food product promises to fight free radicals, be highly skeptical. The best way to fight free radicals is to eat tons of Vitamin C and Vitamin E(particularly a la Linus Pauling). There are very few edible and bioavailable radical scavengers. John Riemann Soong (talk) 16:36, 29 June 2010 (UTC)[reply]

Be skeptical. Ironic that you would then recommend to "eat tons of Vitamin C and Vitamin E". Where is the evidence that taking excessive amounts of vitamins would be any more helpful than "some random skin or health food product"? In fact, the evidence for C and E supplementation is pretty ambiguous. For example, a large randomized, double-blind, placebo-controlled factorial trial of vitamin E and vitamin C showed no benefit for cardiovascular disease and no benefit for preventing cancer. --- Medical geneticist (talk) 17:48, 29 June 2010 (UTC)[reply]
Because it's actually backed up by science? Those studies do not supplement Vitamin C at even the same magnitude of a level that other mammals without our enzyme deficiency would make. Besides, the benefit is lifelong. Naturally if you take it at age 50, where the genetic mutation that will trigger cancer down the road has already occurred, then it is often too late.
People who study Vitamin C megadosage to disprove it don't seem to get the idea: the idea is to make up for our enzyme deficiency. Among other things, the dose could be administered inefficiently -- perhaps direct delivery to the cells is most efficient, seeing as Vitamin C is quite a sensitive reducing agent. It's not just about taking vitamins at the store. John Riemann Soong (talk) 03:24, 30 June 2010 (UTC)[reply]
Could you cite a few references, please? Also, what do you mean by "direct delivery to the cells"? --- Medical geneticist (talk) 19:26, 30 June 2010 (UTC)[reply]

Manganese

Is the punny name a coincidence? --76.77.139.243 (talk) 12:59, 29 June 2010 (UTC)[reply]

??? Don't under stand what you mean. --Chemicalinterest (talk) 13:03, 29 June 2010 (UTC)[reply]
Is it a WP:Troll? --Chemicalinterest (talk) 13:03, 29 June 2010 (UTC)[reply]
If you want data, look at Manganese#History. --Chemicalinterest (talk) 13:10, 29 June 2010 (UTC)[reply]

No,manganese has nothing to do with the language of mangas... Physchim62 (talk) 13:14, 29 June 2010 (UTC)[reply]

If it's unrelated to manga, where does it come from? --76.77.139.243 (talk) 14:19, 29 June 2010 (UTC)[reply]
You have already been pointed to Manganese#History. Why you think that an ages old metal should bear the name of a relatively new art form is puzzling. --Tagishsimon (talk) 14:31, 29 June 2010 (UTC)[reply]
Not even the first time someone has asked that here. 76.77, you ask a *lot* of questions, at least some of which are easily answered by reading the very obvious articles and/or searching the ref-desk archives or wikipedia/google--please help conserve the limited time of ref-desk participants. DMacks (talk) 15:41, 29 June 2010 (UTC)[reply]
Or ask me on my talk page. My talk page is the haven for all chemistry questions. --Chemicalinterest (talk) 17:04, 29 June 2010 (UTC)[reply]

Making Fatty Acid Methyl Esters

Hi all. Recently I've been doing fatty acid extractions and I've been wondering how my method works. In short the method involves putting a biological sample + methanol + acetyl chloride + hexane into a test tube and cooking it for a while. As far as I can tell I'm breaking existing ester bonds (triglycerides) and making new ones (methyl esters). Why is the methyl ester the prefered form? Why do the existing ester bonds break? I thought you needed a basic enviroment for that. —Preceding unsigned comment added by 137.224.252.10 (talk) 14:12, 29 June 2010 (UTC)[reply]

"As far as I can tell" how do you know (or at least suspect) that's what's happening?--are you following some published procedure and seeing predicted positive results, or just seeing some physical change that you are trying to rationalize? (I'm not trying to be rude here, just trying to to figure out what your level of experience/background is here.) Our ester article has a section about reactions, including notes relevant to your "Why do the existing ester bonds break? I thought you needed a basic enviroment for that." concern. DMacks (talk) 15:34, 29 June 2010 (UTC)[reply]
The method is published and relatively old (oldest paper I can find that fully describes it is from 1984). The trick is to extract fatty acids (actually triglycerides) and subsequently methylate them. The methylated fatty acids are then analysed using GC. It's a method that's been used in my lab for ages however when I ask the researcher who first started using it how the method actually worked (the chemistry behind it) he just smiled and said " it just works ". My background is not in chemistry as I'm more on the biology side of things. I did have some introductionary organic chemistry courses. Enough to understand a reaction mechanism, just not enough to figure this out. —Preceding unsigned comment added by 137.224.252.10 (talk) 16:00, 29 June 2010 (UTC)[reply]
OK, you're going to need a bit of thermodynamics to understand why the reaction goes in the direction it does... Imagine a triglyceride, something like a letter E with three long dangly bits hanging off. Now those "long dangly bits" (the fatty acid chains) want to move around in solution, and they do, but they are tied together at one end. If you methanolyze the glyceride linkage to form a methyl ester, the chains can move around even more, because they're free at both ends. In technical terms, they have more conformational degrees of freedom. And more degrees of freedom means a higher entropy (positive ΔS). ΔH is as near to zero as makes no difference, because the bonds being broken and the bonds being formed have about the same strength, so the positive ΔS leads to a negative ΔG and the reaction goes towards the methyl esters.
As for the mechanism, it is an acid-catalyzed nucleophilic acyl substitution: the acid comes from the acetyl chloride you add, which reacts with a bit of the methanol to release HCl. Physchim62 (talk) 18:11, 29 June 2010 (UTC)[reply]

Can I ask why the acetyl chloride is so necessary? It seems pretty hazardous for someone who doesn't understand the chemistry to handle. Why not just add up straight up HCl? I don't think nucleophilic or Lewis acid catalysis is involved.

Also, it's enough to think about kinetics without even thinking about thermodynamics. A heavy polyol that already has partially been esterified will find it difficult to nucleophilically attack your methyl ester, while methanol being light and numerous will find it easy to attack triglycerides. It can be a simple case of numbers: if you have enough methanol, you will end up with mostly methyl esters. I bet a lot of tri/di/mono glyceride is left behind though.

Your lab's method is pretty outdated and hazardous and I think needs to be changed. The trend in vogue today is to add some peptide coupling reagents. You just stir in powder and that's it. Far more selective, less harsh and has a much higher yield. You don't have to deal with HCl gas. Ugh.

Btw, base is used when you want saponification. When you use base, carboxylates are not electrophiles so they don't react further so there is usually no equilibrium reaction. However ions don't undergo GC very well... being well, ions that will probably stay in the liquid phase. John Riemann Soong (talk) 18:56, 29 June 2010 (UTC)[reply]

Ooof! If you can't handle acetyl chloride, you shouldn't be in a chemical lab. full stop end of sentence. The method is a standard method for determining fatty acids in triglycerides by gas chromatography, in use for at least forty years now.
And the reason you add acetyl chloride is exactly to avoid using (expensive, fiddly) hydrogen chloride gas. You can't add hydrochloric acid, because you don't want to add any water to your system of fats and hexane, that doesn't really go very well, now, does it. On the other hand, if you add a bit of acetyl chloride (cheap, available in most decent chemical labs, and a substance that any chemistry student worth their salt will already have handled), it will both scavange any stray water in your methanol and it will produce enough HCl in solution (and a controllable quantity, much easier than working with a cylinder) to provide the acid catalyst to protonate the ester carbonyl group and promote the transesterification. Physchim62 (talk) 19:08, 29 June 2010 (UTC)[reply]
Umm I'm sure acetyl chloride is an even more dangerous reagent than HCl. Most biological labs I know do not have it. And the reaction is just so UGLY though! There's so much opportunity to end up with a mix of products. Why not a mild reagent, like DMAP, and a Lewis acid catalyst you can recycle? John Riemann Soong (talk) 19:33, 29 June 2010 (UTC)[reply]
Btw, I'm curious why you don't think acyl chlorides are intimidating reagents to work with. Plus inventory-keeping and storage are such a pain in the ass. The fat-solubility of acetyl chloride is also another issue, making it more hazardous than even concentrated HCl or sulfuric acid. John Riemann Soong (talk) 19:38, 29 June 2010 (UTC)[reply]
Please tell me you joking? or just trying to wind me up? Physchim62 (talk) 19:46, 29 June 2010 (UTC)[reply]
Um no? I don't like reagents that react whenever you add some random nucleophile to it...and God forbid if your fatty acid has a hydroxyl group somewhere like in a lot of natural plant products. John Riemann Soong (talk) 20:19, 29 June 2010 (UTC)[reply]
Come on, acetyl chloride isn't THAT reactive. Sure, it will give you a burn if you get it on your skin, so don't get it on your skin and, if you do, wash it off straight away. It forms very irritating fumes, so use it in a fume hood. End of story. DMAP, on the other hand, is a central nervous system toxin by skin absorption, with a LD50 of just 13 mg/kg by skin absorption [4]: now, for me, that's a reagent to be treated with respect! And you didn't say what Lewis acid you'd be using (nor how you'd stop the Lewis acid being gobbled up by the DMAP): you have plenty of toxicity and corrosivity problems with common Lewis acids.
Inventory-keeping and storage for acetyl chloride are no worse than for any other chemical – just keep the bottle tightly closed, and lock the lab door when there's no one around. Oh, and don't order 50 litres of the stuff when you've just come back from holiday in central Asia, or the DEA might start getting suspicious! In fact, don't order 50 litres of the stuff, full stop: 500 mL bottles are usually more than adequate even if you're using it fairly routinely.
Acetylation of chain hydroxyls shouldn't be a problem, given the large excess of methanol present (methanol is much more nucleophilic than a long chain secondary alcohol). The acetyl chloride is completely consumed, and your only side product is a small amount of methyl acetate, which will fly through your GC column. And therein is the great beauty of methanolysis: it's simple, it's cheap, it's relatively safe and doesn't produce products which need special treatment for disposal AND you can take your reaction mixture and inject it straight onto your GC column to analyse it, no need for work-up. That's why methanolysis is still a routine technique after forty-odd years. Physchim62 (talk) 21:23, 29 June 2010 (UTC)[reply]
comment - if any 'spare' OH groups are exist, then acetylating them will almost certainly smooth the subsequent chromatography.83.100.252.42 (talk) 11:52, 30 June 2010 (UTC)[reply]
The reaction is Transesterification, the driving force is almost certainly an excess of methanol. (though Physchim's comments about entropy of fats are also valid)83.100.252.42 (talk) 11:52, 30 June 2010 (UTC)[reply]

General anesthetic

What's the fastest-acting general anesthetic? --76.77.139.243 (talk) 14:44, 29 June 2010 (UTC)[reply]

Try Googling fastest general anesthetic. --Chemicalinterest (talk) 14:52, 29 June 2010 (UTC)[reply]
I tried already, and I couldn't find anything. --76.77.139.243 (talk) 15:01, 29 June 2010 (UTC)[reply]
Blunt impact to the head? —ShadowRanger (talk|stalk) 14:58, 29 June 2010 (UTC)[reply]
I'm no expert on anesthesia, from reading General anaesthesia I gather that there are different stages of anesthesia so the answer to your question could be complicated by this. After reading General_anaesthesia#Induction_of_anaesthesia and the links from there I'd have a stab at a volatile anaesthetic such as sevoflurane being pretty fast acting. This paper says it takes between 37 and 70 seconds for "loss of eyelash reflex" to occur depending on the age of a patient. This paper (not free) found it was faster acting than thiopental (an intravenous anesthetic), taking on average 42 seconds to cause loss of eyelash reflex compared to 45 seconds. 86.7.19.159 (talk) 16:40, 29 June 2010 (UTC)[reply]

Tin(II) sulfide formation

Why does it list a reaction between tin(II) chloride and hydrogen sulfide to form tin(II) sulfide and hydrogen chloride? I thought that the reaction went the other way. --Chemicalinterest (talk) 15:05, 29 June 2010 (UTC)[reply]

Depends on the concentrations, as with many chemical reactions. Tin(II) sulfide is very insoluble in soluble in water, which means that H2S will precipitate it even from mildly acidic solutions of tin(II): this sort of reaction used to be very important in qualitative inorganic analysis (tin is in group II, cations that form sulfides in acid solution). It takes concentrated hydrochloric acid to push the equibrium back in favour of SnCl2 and H2S. Physchim62 (talk) 16:34, 29 June 2010 (UTC)[reply]
So only concentrated HCl will reverse the reaction? --Chemicalinterest (talk) 16:53, 29 June 2010 (UTC)[reply]
Actually, looking at the figures, bench dilute hydrochloric acid would probably do it. But don't forget that, when you are precipitating tin sulfide, you are only making a stoichiometric amount of HCl, that's rather different from adding hydrochloric acid onto tin sulfide precipitate (when acid will be in large excess). Physchim62 (talk) 19:25, 29 June 2010 (UTC)[reply]

Why doesn't it have an article? --Chemicalinterest (talk) 16:53, 29 June 2010 (UTC)[reply]

What makes you think it exists? Physchim62 (talk) 17:56, 29 June 2010 (UTC)[reply]
Any other metal oxide exists. --Chemicalinterest (talk) 20:24, 29 June 2010 (UTC)[reply]
Hg(0) + Hg(II) is more stable than 2 Hg(I), as can be seen with the reactivity of mercury(I) chloride. As I imagine, it quickly decomposes even shielded from catalytic agents. Any Hg(I)-O-Hg(I) is probably covalent in nature; the oxygen pulls electrons from one mercury(I) atom and returns another pair of electrons to the other. John Riemann Soong (talk) 20:43, 29 June 2010 (UTC)[reply]
Second question: What makes certain substances more stable while others disproportionate easier? --Chemicalinterest (talk) 20:54, 29 June 2010 (UTC)[reply]
Are you asking why Hg(I) chloride doesn't disproportionate spontaneously? I'm guessing it's because the oxygen is actually rather "unhappy" with the whole Hg-O-Hg arrangement and can get much better electron density by proceeding with disproportionation. Whereas chlorides are relatively 'content'. John Riemann Soong (talk) 21:04, 29 June 2010 (UTC)[reply]
This is complete speculation, but with Hg2O, the oxygen does the reducing and oxidising, which it seems apt to do (oxygen can be both electron-donating and electron-withdrawing at times). With Hg(I) Cl, the mercury atoms have to reduce/oxidise each other directly ... which is difficult without the help of say some UV radiation. John Riemann Soong (talk) 21:06, 29 June 2010 (UTC)[reply]
As an aside, Chemicalinterest, you might want to have a look at Frost diagrams, which graphically represent the stability of an element's oxidation states and makes a link between stability and electrode potentials. Our article is pretty scanty, but Google throws up some good examples. If you're after a textbook, check an undergraduate-level textbook, such as Shriver and Atkins' Inorganic Chemistry, for a more in-depth look at them. Brammers (talk/c) 22:44, 29 June 2010 (UTC)[reply]
But why do some substances disproportionate, eg solid NaClO, while others, eg O2, don't disproportionate into O2+ and O2-? Thank you. --Chemicalinterest (talk) 00:01, 30 June 2010 (UTC)[reply]
The favourability of disproportionation is a result of the stabilities of the disproportionation products relative to the disproportionating species (try saying that after a few beers!). O2+ has a stronger bond (check out the O2 MO diagram but with one antibonding electron removed) but bears a positive charge, and O2-, while having a less unfavourable negative charge also happens to have a weaker bond due to the presence of an additional antibonding electron. Hand-wavey, I know, but that's the best I can do. As for your NaClO question, I'm going to use an example with electrode potentinals from Shriver and Atkins:
5 HClO → 2 Cl2 + ClO3- + 2 H2O + H+ is the overall disproportionation
4 HClO + 4 H+ + 4 e- → 2 Cl2 + 4 H2O Eo = +1.63 V
ClO3- + 5 H+ + 4 e- → HClO + 2 H2O Eo = +1.43 V
so Etotalo = 1.63 - 1.43 = +0.20 V. ΔG = -nFEo so ΔG is negative, and so HClO's disproportionation is thermodynamically favourable (note that this doesn't deal with kinetics, which is tougher to predict). On a Frost diagram, you'd see that HClO's point would lie above a line drawn between its two neighbours: this indicates that it disproportionates. Similarly, if a species lies in a 'trough' on a Frost diagram, the adjacent species will (assuming kinetics are OK) comproportionate to give that species. Brammers (talk/c) 08:57, 30 June 2010 (UTC)[reply]
Wrong reaction there buddy. See potassium chlorate; it is formed by the reaction of sodium chlorate with potassium chloride. The sodium chlorate is caused by the disproportionation of the hypochlorite into chloride and chlorate in the high electrolysis temperature. Thank you. --Chemicalinterest (talk) 10:42, 30 June 2010 (UTC)[reply]
And? That's exactly the reaction that Brammers described. You get sodium chlorate at high electrolysis temperatures because the higher temperature increases the rate of disproportionation of the hypochlorite which is initially formed. The thermodynamic products are chloride and perchlorate, but the production of perchlorate by disproportionation is negligeably slow so, in practice, the system stops at chlorate. Potassium chlorate is less soluble than sodium chlorate, which is why it can be prepared by the reaction you describe. Physchim62 (talk) 10:49, 30 June 2010 (UTC)[reply]
It shows the reduction of hypochlorite to chlorine, not chloride. I never can find a standard potential for reduction to chloride. --Chemicalinterest (talk) 14:54, 30 June 2010 (UTC)[reply]
Generally electrode tables only show a reduced set of potentials from which you can work out most others = for ClO3-/Cl- you probably need to combine ClO-/Cl2 (A) and Cl2/Cl- electrode potentials (B) - the value in this case will be A+B/2 - it's related to the Gibbs energy, if you don't know how to combine electrode potentials that would be a separate question...83.100.252.42 (talk) 21:23, 30 June 2010 (UTC)[reply]
Hang on a minute. Hg2O possibly does exist , see http://antoine.frostburg.edu/chem/senese/101/inorganic/faq/missing-mercurous-compounds.shtml
One important thing to note is that "Hg+" is in fact Hg22+, there's a covalent bond. There's probably not an article because it's not stable and not useful.. Search for it in google books, there's some info on it there... It appears it decomposes in light, and even when ground in a mortar.83.100.252.42 (talk) 21:23, 30 June 2010 (UTC)[reply]

what are these strange holes in the ground in afghanistan?

Here are coordinates,31.63779,65.050017. The town is "Pir Zadeh". Look it up on google maps. There are lines of deep looking holes crossing the terrain, it looks like perforated paper. These lines of holes continue up that river all the way into the mountains. I don't think they are for storing water because they occur in the mountains where there are no fields or orchards. What are these holes? —Preceding unsigned comment added by 75.164.144.81 (talk) 18:45, 29 June 2010 (UTC)[reply]

They're Quanats, an ancient and effective method of channeling water from hills to areas where it is needed for irrigation etc. Mikenorton (talk) 19:22, 29 June 2010 (UTC)[reply]
31°38′16″N 65°03′00″E / 31.63779°N 65.050017°E / 31.63779; 65.050017, fwiw. --Tagishsimon (talk) 21:04, 29 June 2010 (UTC)[reply]

Thank you, thats cool. —Preceding unsigned comment added by 75.164.144.81 (talk) 23:34, 29 June 2010 (UTC)[reply]

Actually it's spelt qanat (as the redirect shows), one of the few words in Scrabble you can spell using a Q but no U. Zunaid 08:27, 30 June 2010 (UTC)[reply]

Soreness from a Shot

What is the origin of residual pain in your arm after a vaccine injection? Is the soreness from damage to the tissue by the needle or is it something to do with whatever is being injected into your arm? Not a request for medical advice, just idle curiosity. —Preceding unsigned comment added by 72.85.199.192 (talk) 19:40, 29 June 2010 (UTC)[reply]

It likely depends on what vaccine and what soreness you're referring to. The Bacillus Calmette-Guérin vaccine for example is noted for the swelling and scar that results (and I can say from personal experience swelling can be fairly sensitive for a fair while). As noted in our artice, this arises from the body's reaction to the vaccine. Nil Einne (talk) 21:14, 29 June 2010 (UTC)[reply]

Where did this stupid idea that investment is a trade-off between risk and returns come from?

where did this stupid idea that investment is a trade-off between risk and returns come from? I mean, I see how it applies to someone with a very poor understanding of the market, industry, and the factors, such as reputations and technology involved, but this adage (risk/return tradeoff) seems to be stated as though it applies to everyone, even people who understand, correctly, exactly what they are doing and why it will get a high return, in that case, without any particular risk. Thank you. 92.230.66.154 (talk) 20:58, 29 June 2010 (UTC)[reply]

Lots of people thought that they had a good understanding of the market, but got their fingers burnt! Perhaps Warren Buffet is one of the very few who got it right by getting out! Dbfirs 21:15, 29 June 2010 (UTC)[reply]
CAPM 92.29.119.46 (talk) 21:16, 29 June 2010 (UTC)[reply]
warren buffet is a good example. obviously he doesn't trade off between risk and return (for example he doesn't ever invest in technology at all - he doesn't know for sure what will happen). so, now that we know that it is false to say, in general, that there is a trade-off between risk and return, can we get to my question: where does this stupid idea come from? 92.230.66.154 (talk) 21:21, 29 June 2010 (UTC)[reply]
Sure, he trades off between risk and return, but the key thing he does is make better estimates of return than other people do (which is why he doesn't invest in technology - he isn't a technology expert so doesn't feel his estimates would be better than other people's). He compares his estimates of risk and return to work out a fair price and then buys if the market price is lower than what he has calculated as a fair price. This is called value investing. --Tango (talk) 02:25, 30 June 2010 (UTC)[reply]
This isn't exactly my field and I would question if it's a science question anyway but my impression is even those with great experience and knowledge of the market generally agree that there is connection between risk and return, and particularly you should considered whether your expected return is worth the risk you are taking (and whether you can accept that risk). Note that when it comes to the more informed discussion, I've never heard it claimed that all risky investments have a high return or that all low return investments have a low risk, in fact it's very commonly acknowledged that there are plenty of cases when the risk doesn't correlate well with the return (always in the wrong direction of course meaning the return isn't worth the risk or you can get a better return for about the same risk) and that unfortunately the more clueless investors are often fooled by such thing. People who understand the market well can generally better understand the risk, and importantly may be able to reduce it (meaning low risk investments may not be of great interest to them). Perhaps most importantly (even more so after recent events), I don't think many of those people who do have such knowledge would agree that it's possible to get a high return (all being relative of course for them a high return may be higher then for an 'average' person) without any risk (actually to be blunt it sounds like the sort of thing those inexperienced with the market seem to think usually to their peril). Nil Einne (talk) 21:23, 29 June 2010 (UTC)[reply]
It is just your opinion. Mine is that it is true. --Chemicalinterest (talk) 22:43, 29 June 2010 (UTC)[reply]
Perhaps that stupid idea originated with the fact that some small startup companies have a product that may or not catch on or a small research firm thinks they maybe onto a possible cure for cancer and if they are correct their stock will quickly multiply in value, but if they are wrong their investors may lose everything - such investments are inherently more risky than those in established companies and investments, even in large firms, are generally, more risky than investing in government bonds, which, in turn are less "risky" than investing in material goods like gold. Of course, a lack of growth may be considered a risk as well and I'm also speaking in general terms. TheGoodLocust (talk) 01:53, 30 June 2010 (UTC)[reply]
It isn't a stupid idea. It's true. Consider the following choice: A) I give you £1000, B) I toss a coin and if it's heads I give you £2000, if it's tails I give you nothing. Both those options have the same expected return, but B is much riskier than A. If you present that choice to a lot of people, most will choose A (it doesn't always work for small amounts of money because people have different priorities when it comes to amounts they consider trivial). In order to get the same number of people choose A as B, I'd have to increase the expected return of B. --Tango (talk) 02:25, 30 June 2010 (UTC)[reply]
Oooh careful...if I remember game-theory studies, for the identical average payoff over many trials, many will pick the option of a larger prize (with the risk of none, vs sure-thing smaller prize) but many will pick sure-thing smaller cost (vs chance of none or larger cost). Humans aren't rational! See risk aversion. As to the original question, if there's a greater chance (less likelihood of winning) of failure (loss of investment), there needs to be a greater prize (return on investment) if you do happen to win in order to reach the break-even point to bother investing at all. To "win" in the long run (i.e.m over many investments in a portfolio) you need to find opportunities whose average payout is higher than the cost (as I understand how someone else proposed Buffett analysis) or else you need to have enough information to (on average, correctly) think that the likelihood of success is much higher than others do (so I can buy their futures that they think are doomed). DMacks (talk) 08:49, 30 June 2010 (UTC)[reply]
It sounds to me like a rhetorical question. Wikipedia will be able to supply lots of good information about the merits and limitations of this idea, but I seriously doubt anyone will be able to identify where it came from. (Also, I don't think the questioner really wants to know where it came from.) This page is not the place for rhetorical questions. Dolphin (t) 02:57, 30 June 2010 (UTC)[reply]
Investments are scarce. People will invest in the ones that are the most appealing. People like investments with low risk and high returns. As such, all the ones with low risk high return, medium risk and very high return, and very low risk and medium return will tend to already be taken. People are unlikely to even consider the really bad investments such as high risk low return. This leaves you with the trade-off of risk and return. — DanielLC 03:02, 30 June 2010 (UTC)[reply]
May be because many people know that there is correlation between risk and returns, but not many are aware that Correlation does not imply causation. - manya (talk) 07:03, 30 June 2010 (UTC)[reply]
This "stupid idea" comes from the capital asset pricing model, developed by Nobel prize winner William Sharpe and others. Like all models, CAPM is built on certain assumptions. One of the assumptions of CAPM is that all investors will act rationally. So one method of "beating the market" is to exploit irrationalities of other investors. Another assumption of CAPM is that all information is available at the same time to all investors. So another way to "beat the market" is to use information that is not available to other investors - but there are often laws against that. Gandalf61 (talk) 09:21, 30 June 2010 (UTC)[reply]
CAPM was the second reply given above. 92.29.114.87 (talk) 11:48, 30 June 2010 (UTC)[reply]
As for Buffett, he buys at a significant discount to the value he calculates the investment has -- he has stated that this "margin of safety" is his form of "risk/return." As for the concept in general, there's a jargon term: "upside down ratio." What is your "upside"? What is your "downside"? The ratio is the upside down ratio. If you think the investment could conceivably go to zero during your time horizon, then your ratio is infinite, so you better be able to conceive of a very high return in the same period. But if you think the worst-case scenario is e.g. a loss of half, but the best-case is tripling, then the ratio is 6 (3 over 1/2). The OP's question is predicated on the idea that a great investor would know that the risk is minimal and the potential return is great, but investors have diversified portfolios and it is never possible to know what sort of "black swan" event might happen to any (or all?) of the positions. Risk/return is best exemplified in the 90-day note, which is considered the proxy for a "risk-free" return: right now, it returns something like 0.15%. You have a 99.99999% chance of getting the return OF investment, in exchange for a tiny return ON investment. But if the OP is so certain that great investors needn't consider risk/return, he/she should try to find an example of ONE investor who has not made many losing investments even after decades of experience and intensive study of the positions prior to investing. 63.17.87.23 (talk) 04:28, 3 July 2010 (UTC)[reply]

Name these plants?

I've seen the this plant, which was about four feet high, in many places in the southern UK, both gardens and wild, and have always wondered what it was. It reminds me of a mallow, but its a lot bigger than a mallow. http://img134.imagevenue.com/img.php?image=45121_DSCF0002_122_247lo.JPG http://img186.imagevenue.com/img.php?image=45114_DSCF0001_122_462lo.JPG

This one is interesting too, about a foot high: http://img232.imagevenue.com/img.php?image=45106_DSCF0003_122_185lo.JPG http://img134.imagevenue.com Thanks 92.29.119.46 (talk) 21:11, 29 June 2010 (UTC)[reply]

General Alert. On the first linked picture, an apparent advert for wind turbines pops up which, when closed, opens a soft porn link. 87.81.230.195 (talk) 21:27, 29 June 2010 (UTC)[reply]

Really? All I see is plants. 92.29.119.46 (talk) 21:30, 29 June 2010 (UTC)[reply]

No really. I got the lovely Jasmine, who might want to go see a dermatologist to get some of those blemishes checked out. --Tagishsimon (talk) 22:00, 29 June 2010 (UTC)[reply]

The image counter indicates people are seeing the plant photos, unless its fake. 92.24.183.236 (talk) 22:57, 29 June 2010 (UTC)[reply]

They doubtless are were seeing the plant photos as well (and as I did), The LiveJasmine porn stream opens opened in a new window which is was initially minimized on to the lower task bar (this is a common ploy of this particular site, which I've encountered before in, ahem, other circumstances ;-) ). Note that all this refers only to the first linked picture; I didn't bother clicking on the second one. Your particular setup, 92, might be automatically blocking this uninvited link.
[Update, as of this posting, the UFO (Unidentified Flirting Object) seems to have desisted.] 87.81.230.195 (talk) 00:34, 30 June 2010 (UTC)[reply]
The foot-high plant is a sedum, I think sedum Autumn joy. Robinh (talk) 07:17, 30 June 2010 (UTC)[reply]
The plant in the first series is a hollyhock, no sign of any 'Jasmine'! Richard Avery (talk) 07:43, 30 June 2010 (UTC)[reply]
I got the same thing as 87, livejasmine popup after I closed the Flash advert for wind power in the main window Nil Einne (talk) 13:55, 30 June 2010 (UTC)[reply]

Stopping time in a fall on concrete

I want to demonstrate the importance of stopping time to my physics students. I can calculate how much time it takes to slow a high diver to zero when he lands in a pool of water, by calculating how fast he is when he reaches water level and using the pool's depth as an upper limit to the distance the diver covers in stopping. I want to contrast this with what would happen to the diver if he landed on concrete instead of water. I figure the stopping time on concrete is very very small, but I'd like to have at least an order of magnitude. Inkan1969 (talk) 21:36, 29 June 2010 (UTC)[reply]

I doubt anyone will venture a guess as to stopping time, but it is easy to make a guess as to stopping distance - assume the diver lands flat on his back and his center of mass stops in a distance of a centimetre or half an inch. I suggest a more satisfactory illustration can be achieved by considering a mass falling onto springs, and calculating the deceleration during the stopping phase. With a small spring constant k the stopping distance and time are greater, and the deceleration is smaller, than with a large spring constant, illustrating that it is better for the diver to find the water than the concrete! Dolphin (t) 22:54, 29 June 2010 (UTC)[reply]
It is so easy to reading "stopping time" as a referring to a time machine of some sort. But you mean how long it takes the object to decelerate. I don't think it's possible to calculate it on cement, a person is flexible, so the skin might decelerate much faster than other body parts. And if you set the number too low you can easily achieve insanely high numbers. The elasticity of the object plays a very large role in how much force it experiences. I agree with Dolphins suggestion - assume a rigid person, and let a spring act as the elasticity. Ariel. (talk) 23:10, 29 June 2010 (UTC)[reply]
If you have the final velocity before impact v and the stopping distance d, you can estimate the stopping time t by assuming linear deceleration: t = 2d/v. Physchim62 (talk) 23:25, 29 June 2010 (UTC)[reply]
Dolphin51 suggests stopping distances of 1 cm to 0.5 in. Does that sound reasonable to others? If so, I can figure out the stopping time from there, like how Physchim62 notes. I also found out on the Diving discussion board that if you bellyflop from 10 m the water will stop you in only 1 ft distance. That's another good illustration of the importance of stopping time.Inkan1969 (talk) 19:17, 30 June 2010 (UTC)[reply]
You might find research on automotive collisions helpful here. There was a question some time back that I remember finding a reference for about the speeds of collision and its effects on the human physiology. Ah here it is - Impact acceleration stress, mentioned during this unfortunately grisly discussion of motorcycle accidents from last year. This book has some very useful quantitative graphs of experimentally measured impact decelerations (literally, from slamming into concrete), and their effects on humans. "We can say that mechanically the lumbar vertebrae can stand a static stress of 800 kp or a dynamic stress during 0.006 sec of 1,300 kp." (Page 26, discussion of maximum durations of impact stresses before mechanical failure of ... human). Beyond this level of impact, humans no longer behave as intact rigid bodies and will deform as a fluid; you will have to model individual stopping-distances and stopping times for individual parts. Nimur (talk) 23:47, 29 June 2010 (UTC)[reply]
Thanks for the suggestion, Nimur. I'll look up the reference. Inkan1969 (talk) 19:17, 30 June 2010 (UTC)[reply]

What are the worms that create this red moss formation

The red moss formations are actually worms. They are in about 10cm of water and are about 1mm by 2-4cms. The red "fuz" will dissapear if touched. If some of the red formation is turned over, the mud will boil for a while and then turn red if undisturbed. What kind of worms are they? Is this a larval stage? File:C:\temp\red moss worms.jpg —Preceding unsigned comment added by 205.175.250.27 (talk) 23:19, 29 June 2010 (UTC)[reply]

Sorry, you've linked to a file on your local hard disk, and we don't have access to that. Could you try uploading to a service like Flickr and then point us to that photo? Comet Tuttle (talk) 23:27, 29 June 2010 (UTC)[reply]
Or you can upload the image to Wikipedia's servers, if it complies with our requirements and you are willing to license it for free re-use. Nimur (talk) 23:58, 29 June 2010 (UTC)[reply]
It sounds like you are talking about tubifex worms? eg http://www.google.co.uk/images?hl=en&q=tubifex&um=1&ie=UTF-8&source=og&sa=N&tab=wi 83.100.252.42 (talk) 11:38, 30 June 2010 (UTC)[reply]
Tubifex worms typically move in a "wavy" motion, but often recoil if disturbed or eaten by fish. I've seen a video of tubifex worms in a sewer moving as if they were a single muscle undergoing contractions (it was simply disgusting). ~AH1(TCU) 21:42, 2 July 2010 (UTC)[reply]

June 30

Are sodium chloride crystals always square?

Recently I've been making homemade potato salad. Never even knew how much I loved it until I made it fresh. Anyway, you put a ton of salt in the water where you boil the potatoes. So I left the salt laden water and went away for two days and when I got back I had salt crystals all over the bottom of the pan, big fat squares and smaller ones, all square crystals. I thought this was pretty neat. I just headed over to the articles and see a picture of a square crystal at the top as well as the structure, which looks conducive to squares, but I'm curious if they're always square like this?--141.155.157.4 (talk) 00:04, 30 June 2010 (UTC)[reply]

Like other crystals, salt has preferred cleavage planes. In the case of NaCl, the preferred planes are aligned at 90-degree angles (in all three axes). As such, there is a preference for square and rectangular shapes. In practice, impurities and the mechanics of forming and breaking crystal faces means that there will be imperfections. You can see such imperfections in the many images on our sodium chloride article. The atomic structure means that the alignment of faces will prefer 90-degree angles; it doesn't directly affect the aspect ratio of the crystals that form, so it is equally easy to form rectangles as squares; but because the crystal probably grew homogeneously and isotropically, a cubic structure is very common. Nimur (talk) 00:38, 30 June 2010 (UTC)[reply]

Drugs

Why are drugs so costly. I think heroin costs as much as Rs 1 crore (that's 10 million) per kg  Jon Ascton  (talk) 03:16, 30 June 2010 (UTC)[reply]

I don't imagine that the heroin industry has a substantial R&D budget. TheGoodLocust (talk) 03:28, 30 June 2010 (UTC)[reply]
Heroin is highly addictive, which means people are willing to pay such large amounts for it. The costs and risks involved with smuggling and dealing illegal drugs (if you risk many years in prison, you're only going to want to take that risk if you can expect massive profits - it's the risk/return trade-off that is discussed a few sections above) mean it isn't viable to sell it any cheaper. The combination of those two facts means that it is sold at very high prices. (If the first weren't true, it wouldn't be sold. If the second weren't true, it would be sold much cheaper.) --Tango (talk) 03:54, 30 June 2010 (UTC)[reply]
Also, since drug dealing is already illegal, dealers tend to have no qualms about using illegal business practices to get rid of competition and charge monopoly rates. Since it's so hard to quit drugs, they can get away with making these rates very, very high. — DanielLC 05:12, 30 June 2010 (UTC)[reply]
Hardly. Drugs are an incredibly competitive market. Certainly they have a low elasticity, but that balances against an incredibly fluid market, unregulated (notwithstanding, of course, the massive resources spent prohibiting them). Insofar as dealers have monopoly rates it's a function of how able they are to repel competitive dealers, which largely requires illegal activity that attracts much more attention than the actual drug selling does. In short, drug prices are high because interdiction and law enforcement efforts make them that way. That doesn't mean though that they're not a competitive market. Shadowjams (talk) 08:04, 30 June 2010 (UTC)[reply]
The book Freakonomics has a chapter devoted to the economics of crack. Its pretty good read on the subject. --Jayron32 03:45, 2 July 2010 (UTC)[reply]

why does my sink solution turn cloudy when I add baking powder to it?

I mean dissolved sodium bicarbonate should be transparent... right? Or does some of it end up not dissolving? John Riemann Soong (talk) 03:42, 30 June 2010 (UTC)[reply]

P.S. I had HUGE chunk of dishwater. Surely that's enough water to dissolve a tablespoon or two of sodium bicarbonate? Would the alkaline conditions precipitate any ions commonly found in food? John Riemann Soong (talk) 03:46, 30 June 2010 (UTC)[reply]

Do you live in a hard-water area? If your tap water has a high level of calcium or magnesium ions, then those ions could react with the baking soda to form insoluble carbonates, which could make the water cloudy. FWIW 67.170.215.166 (talk) 09:22, 30 June 2010 (UTC)[reply]

(edit conflict)

In ionic solutions, the ions are fully disassociated, i.e. if you dissolve in water the salts sodium chloride and calcium iodide, you also get sodium iodide and calcium chloride. What else was in the water? If there was a soluble calcium salt (say calcium chloride), then the insoluble calcium carbonate will be produced, which will precipitate out of solution. (Calcium bicarbonate decomposes to calcium carbonate). Do you live in a hard water area? Hard water contains calcium salts. CS Miller (talk) 09:43, 30 June 2010 (UTC)[reply]

reason of raining

What is the reason of raining ? —Preceding unsigned comment added by 203.194.98.218 (talk) 04:02, 30 June 2010 (UTC)[reply]

See rain for an explanation. Looie496 (talk) 04:58, 30 June 2010 (UTC)[reply]

V-tails

Why are V-tails so stealthy, since when top of the aircraft faces the radar it forms a right angle or corner reflector? --The High Fin Sperm Whale 05:05, 30 June 2010 (UTC)[reply]

Are they terribly stealthy? Anyway, how often do you think the top of an aircraft is going to be facing radar? AFAIK we don't have satellite based radar. TheGoodLocust (talk) 05:38, 30 June 2010 (UTC)[reply]
AFAIK V-tails reduce RCS when viewed from the side by avoiding the right-angle reflector formed by the conventional rudder/tailplane combination. BTW, in most V-tailed stealth aircraft (e.g. F-117, F-22, YF-23) the ruddervators are mounted at an angle other than 90 degrees. FWiW 67.170.215.166 (talk) 09:28, 30 June 2010 (UTC)[reply]
Yes, the critical point here is that from certain directions, the reflections are attenuated. Designers of these aircraft "estimate" which view angles are most likely during a standard flight / sortie during the high-level system-design, when things like "preferred combat sortie flight altitude," "cruising speed," and "maximum allowable proximity to hostile RADAR" are still subject to re-design. As these parameters solidify, flight profiles emerge that indicate that, for example, the aircraft might always be viewed from the side at an angle of, say, 15 to 45 degrees elevation; then the designers then attempt to make such a viewing angle the most stealthy (attenuated reflection). Pilots and mission planners are instructed to design flight profiles to keep the aircraft within its "best-performance" spec. In practice, because aircraft specs are hard to change once they are built, and combat flight profiles are subject to operational needs, the aircraft may be used during combat in a "sub-optimal" stealth regime; but it's still probably more stealthy than an aircraft that wasn't designed carefully with RADAR reflectivity in mind. What this means, on average, is that combat flight profiles that get the aircraft closer to hostile zones will result in statistically fewer detections; and thus, statistically fewer downed aircraft. Historically, especially after the fall of the USSR, the development of stealth aircraft went hand-in-hand with complete and total air superiority during most combat operations; so these kinds of design considerations are subject to fierce debate in terms of how economically effective they are. But if we were fighting an air-war on the scale of, say, World War II, when anti-aircraft fire could down 10% of the planes on any given mission, we would be fighting tooth-and-nail to improve those statistics. Every fraction of a dB of attenuation corresponds to a few extra moments of non-detection, and thus a few extra moments of tactical advantage. I could ruminate extensively on the futility of these sorts of optimizations with respect to modern warfare, but I won't... Nimur (talk) 17:20, 30 June 2010 (UTC)[reply]

Chromosome Location

Which chromosome isresponsible for the DNA that codes for our brain and memory, whether someone is naturally better at Math or better at art? Is it all encoded on one chromosome or is spread out among multiple chromosomes? Also, how is it posible to know which part of a chromosome or the genetic material encoded on say chromosome 6 for example has the DNA for brown eyes, how do scientists know this? —Preceding unsigned comment added by 71.143.241.161 (talk) 07:24, 30 June 2010 (UTC)[reply]

No one knows what codes for brain and memory, and they don't know what codes for brown eyes. At best all we can say is that a bunch of people with a similar trait all had this piece of DNA in common. But often you'll have people with that trait that don't have the gene, and sometimes people with the gene still don't have the trait. It's rare to actually be able to "read" the DNA and understand what it does. Usually all you can say is that this piece seems to be common to these traits, without understanding any of the details - and without even understanding what other DNA might have an effect too. Ariel. (talk) 11:54, 30 June 2010 (UTC)[reply]
No DNA directly codes for traits, features, aptitudes, or any such things. Bits of DNA (usually called genes) generally code (directly or via the intermediary of RNA) for the production or expression of proteins, which then interact in often extremely complicated ways (including promoting or blocking the actions and/or effects of other DNA or proteins). The end results of all these RNA and protein expressions (including, crucially, their timing) and interactions may usually, often or sometimes (depending on the context of all the other interactions going on) result in the physical manifestation of certain particular traits, features, etc of the human body and brain: saying that "gene x codes for feature y" is a frequently used shorthand expression, but glosses over the complexities and can be misleading.
Sometimes a functioning gene may not be contiguous: that is, it may be split into two (or more?) physically unconnected parts. Sometimes a part of a gene "for x" may also interact with some or all of another gene to result in "y", so the same short stretch of DNA may be expressing two (or more?) different proteins at the same or different times; also, the same protein may do two (or more?) different things at different times, or at the same time in different places, depending on the other proteins present there.
Things like higher mental properties and aptitudes are very difficult to correlate with specific bits, or combinations of bits, of DNA - the concept of emergent properties may be useful to consider in this regard.
Chromosomes are merely a way of packaging all the genes into manageable 'chunks' that are less fragile, less unwieldy, and easier to copy than if they were all in one 'chunk' - a single chromosome, which the earliest life forms with fewer genes presumably started out with. It's possible for a single chromosome to split into two, or two to merge into one, without this preventing successful reproduction by the individual concerned with an 'unmodified' mate. For example, Chimpanzees and Bonobos have 48 chromosomes instead of Humans' 46 because around 1 million years ago (after the split some 4-8 million years ago between the Human and C/B populations) two chromosomes in a human individual joined up to form one larger one which, after spreading through the human population, is now our Chromosome 2. Similarly, a human family has recently been identified in which 2 more chromosomes have merged giving a complement of only 44, though all (or nearly all) of the genes involved are still present, just differently packaged. [Sorry, can't re-find the reference for this. Anyone?] Migration of individual genes from one chromosome to another is, I believe, not at all uncommon, and minor losses (or duplications) of one or several genes in a chromosome are pretty common.
The approximate position or near-exact identity of a particular gene "for" x has in the past been worked out by looking at people with and without x, and with and without various bits of chromosomes, and finding the correlations. Now that gene sequencing has, very recently, become much easier and cheaper and the entire human genome (that is, those of a handful of individual humans) has been sequenced, this field of knowledge is moving more rapidly than laypersons (like myself) can keep up with. 87.81.230.195 (talk) 14:05, 30 June 2010 (UTC)[reply]
There are a couple of misconceptions in the original question. First, memory is not encoded in the DNA, it is encoded in the connections (synapses) between neurons within various parts of the brain. The field of neurophysiology is all about the chemical and electrical signals that mediate memory formation. There might be a role for epigenetic mechanisms in memory formation, but this is still an active area of research and not definitive. Formation of the brain itself is an extraordinarily complicated process that is guided by thousands of genes scattered across all chromosomes, acting in different cell types at different times during development. The field of developmental neuroscience aims to understand this process from anatomic, cellular, and molecular points of view. Whether a person is skilled at math or art (or both!) is a result of multifactorial inheritance and cannot be pinned to a single gene, and is equally dependent on complex interactions between their genetic makeup and their environment. See nature versus nurture for a general discussion, although I think this is a false dichotomy -- everything is about genes AND environment. The methods used for identifying genes associated with human characteristics (eye color, disease risk, complex traits) are quite diverse and depend on the specific question being asked. The OP should start with human genetics as a basic introduction to the topic. --- Medical geneticist (talk) 14:53, 30 June 2010 (UTC)[reply]


Putting aside environment for a moment, what are some of the methods used now or in the past that scientists and researchers use in order to identify genes associated with human characteristics? Does one of those methods include comparing the genes of people with a similiar characteristic or trait? And also, when they identify those genes are they able to trace them back to where and which chromosome they came from? I ask because, I once read that the location on the chromosome and which specific chromosome determines what that DNA will turn into or become, though I am not sure how true that is? —Preceding unsigned comment added by 71.143.241.161 (talk) 17:29, 30 June 2010 (UTC)[reply]

I've tried to answer part of this question below, but I'm having trouble understanding your last question. Can you clarify what you mean by "the location on the chromosome and which specific chromosome determines what the DNA will turn into or become" ? --- Medical geneticist (talk) 19:18, 30 June 2010 (UTC) [reply]
Concerning the original question: All of the chromosomes are involved in generating the structure of the brain. There are 23 pairs of chromosomes, each containing several thousand protein-coding genes. The great majority of genes are expressed in the brain, and a large fraction are expressed only in the brain. In quantitative terms, most of the information in our genome is directed toward shaping our brains. (This is not to deny the contribution of experience and environment -- I am only asserting that the contribution of genes is very large, not that other contributions are small.) Looie496 (talk) 17:38, 30 June 2010 (UTC)[reply]
As I said above, there are lots of methods that can be used to determine the function of genes. A classic method is called linkage analysis where one basically tries to identify markers that segregate with a particular trait following a particular inheritance pattern in a family. For the study of complex traits, the genome-wide association study or GWAS (unfortunately, that article could be much better) has become the dominant method of identifying genes associated with human characteristics. There have been hundreds or thousands of GWAS studies performed in the last few years, all building on the completion of the human genome and Hap map projects. One way to perform a GWAS is to compare people with a certain characteristic (disease, trait, etc) to people without that characteristic by looking at the presence of single nucleotide polymorphism (SNP) markers, which are genetic differences that are present in the general population at a certain frequency. Using fancy statistics you can determine which markers are more often present in people with the trait versus those without, thus generating an "association" between a marker and the trait of interest. By definition, we know exactly where these markers are located along each chromosome (otherwise they wouldn't be that useful) and what genes are nearby. However, association does not equate to causality and there are lots of challenges to interpreting the biological significance of any given association. --- Medical geneticist (talk) 19:15, 30 June 2010 (UTC)[reply]
Even if talents have a genetic basis, I don't think it's possible to determine that any single gene is responsible for what the individual is good at. An example of environmental factors taking precedence over genetic predispositions is music making a child more intelligent, as it has been shown that playing a musical instrument increases can make a person smarter in subjects such as mathematics, while limiting a child's exposure to music has been demostrated to have the opposite effect. ~AH1(TCU) 21:36, 2 July 2010 (UTC)[reply]

Telescope Eyepiece

In old German astronomy texts the ocular is described as : mit 20 Linien Oeffnung ( 20 lines opening) . Is there a possibility to translate this into a unit used today? Thanks.--Stone (talk) 07:28, 30 June 2010 (UTC)[reply]

The length unit line (German: Linie) seems to be referenced here. Best see both of these articles. HTH. --Ouro (blah blah) 08:47, 30 June 2010 (UTC)[reply]
The line was a unit of length of about 1.9–2.2 mm throughout most of Europe. I've not been able to find a specific reference to the German Linie, but it's a fair assumption that it was about the same length. That makes your occular 38–44 mm: does that sound reasonable? Physchim62 (talk) 09:14, 30 June 2010 (UTC)[reply]
Perfect thanks! There was no German line. The French line was in use for most of the technical stuff like telescopes. The best point to look for is the w:de:Pariser_Linie given with 2,256 mm. Paris inch is another place to look for. Why do we have no article on the Paris line? --Stone (talk) 09:22, 30 June 2010 (UTC)[reply]
Ligne. Although I think I might merge it into Line (length) along with the other translations. Physchim62 (talk) 09:48, 30 June 2010 (UTC)[reply]
Be bold! --Ouro (blah blah) 10:56, 30 June 2010 (UTC)[reply]
I understand from the german text, that these two units are similar, but not the same. --Eingangskontrolle (talk) 19:11, 1 July 2010 (UTC)[reply]

Make more time

I've been asked at work to make more time for a project, but I'm not sure I have the necessary supplies. What's time made of? 67.188.234.85 (talk) 08:36, 30 June 2010 (UTC)[reply]

Rocks and buckets. --TammyMoet (talk) 08:41, 30 June 2010 (UTC)[reply]
A song text says: time is made from honey slow and sweet. Worth a try.--Stone (talk) 08:41, 30 June 2010 (UTC)[reply]
Yeah, that's what the song says. But only the fools know what it means. --Trovatore (talk) 08:54, 30 June 2010 (UTC)[reply]
Next you'll be sent down to the supply depot for a "long weight". CS Miller (talk) 09:47, 30 June 2010 (UTC)[reply]
Don't forget to pick up some "elbow grease" while you're there. And don't come back without it! Physchim62 (talk) 12:56, 30 June 2010 (UTC)[reply]
Making more time is easy. What you do is you find some midnight oil and ignite it. I suppose that the additional time is a waste product of this process. Googlemeister (talk) 12:59, 30 June 2010 (UTC) [reply]
I had some midnight oil, but it solidified when it got cold out. So I burned it at both ends and that worked out just fine. Matt Deres (talk) 13:40, 30 June 2010 (UTC) [reply]
If you keep it up, the <small> tag will break sooner or later. --Chemicalinterest (talk) 14:46, 30 June 2010 (UTC)[reply]
Did you intend that to be a reply to 67.188? 86.164.57.20 (talk) 15:51, 30 June 2010 (UTC)[reply]
Or your face will get stuck that way.... Paul (Stansifer) 17:41, 30 June 2010 (UTC)[reply]
Gibberish
The following discussion has been closed. Please do not modify it.
yjr nrdy esu yp ,slr ,ptr yo,r gpt s nifhry od yp ,slr yjr pggovr ,ptr ;olr s jp,r/ d[rmf snpiy yjtrr yjpidsmf fp;;std pm pggovr gocyitrd yjsy ,slr oy ,ptr ;olr s jp,r. smf upi eo;; drr yjsy rbrtupmr pm yjr [tpkrvy jsd ,ptr yo,r yp eptl pm oy \'- 85.181.144.74 (talk) 18:25, 30 June 2010 (UTC) my hands were on the wrong key, can you fix that?[reply]
/ayewkt rgR QIYKS Gcw vwwb niew qiergqgukw ud tiy gS Nsw HIJW/. /KUJWM /U/;N deTWS JBIR, bs nTVW QIEJ IB RGW WZOKbRUIBM UD TIY;EW YAUBF RGW AOxwvE, 86.164.57.20 (talk) 19:32, 30 June 2010 (UTC)[reply]
the best way to make more time for a budget is to make the office more like a home. spend about three thousand dollars on office fixtures that make it more like a home. and you will see that everyone on the project has more time to work on it ] Translation. --Chemicalinterest (talk) 19:32, 30 June 2010 (UTC)[reply]
In the June 2010 Scientific American article "Is Time an Illusion?", we learn that, "The universe may be timeless, but if you can imagine breaking it into pieces, some of the pieces can serve as clocks for the others." Hopefully that will help. Bus stop (talk) 19:42, 30 June 2010 (UTC)[reply]
Simple: you spend less time doing other things!--RampantHomo (talk) 01:17, 1 July 2010 (UTC)[reply]

I've also heard that you can spend all your time making money, or you can spend all your money making time. --Trovatore (talk) 01:24, 1 July 2010 (UTC)[reply]

Sheesh. Time is made of paper, ink, and staples. What else? --Anonymous, 03:36 UTC, July 1, 2010.

You can extend the time you have using time dilation. Either move slower than everyone else, move further from a gravity well, or some combination thereof. Also, make sure not to spend more than a few milliseconds doing this, or you'll waste more time than you save. — DanielLC 07:56, 2 July 2010 (UTC)[reply]

Time may be partially or completely made of Paul Simon. He wrote "Time, time, time, see what's become of me." If there's not enough Paul left (he was getting kind of small the last time I looked), he may be able to suggest other aging musicians who could help. (I don't think Crosby has much to do these days.) Twang (talk) 18:23, 2 July 2010 (UTC)[reply]

You don't actually have to make more time (ie. in a schedule, or physically creating time which is impossible) to make it feel like you have more time. See time perception and perception of time. I'm sure there are techniques to make it feel like you had more time than you did considering all the stuff you've gotten done and I've done that, however I don't have the capacity to elucidate them, but compare apathy and flow. ~AH1(TCU) 18:35, 2 July 2010 (UTC)[reply]

Fabry-Perot filter

What's a MEMS-based tunable Fabry-Perot filter? There isn't an article on it. --76.77.139.243 (talk) 13:15, 30 June 2010 (UTC)[reply]

There is, it's just called a Fabry–Pérot interferometer! Physchim62 (talk) 13:21, 30 June 2010 (UTC)[reply]
And you may want to read up on MEMS as well. "Tunable" suggests that the wavelength that the filter passes can be adjusted, probably because the MEMS is capable of adjusting the spacing of the components on demand. -- Coneslayer (talk) 13:25, 30 June 2010 (UTC)[reply]

Why do cars go slower in reverse than they do going forward?

^Topic says it all. 148.168.127.10 (talk) 14:13, 30 June 2010 (UTC)[reply]

In general, because it is anticipated that you'll only travel for very short distances in a backwards direction; because visibility is restricted, and because steering works differently, both of which make longer distances a more unlikely proposition. And then because having only a single reverse gear calls for less machinery in the gearbox department, equalling lower cost. --Tagishsimon (talk) 14:17, 30 June 2010 (UTC)[reply]
Taking a slightly different tack: as Tagishsimon notes, cars have only a single reverse gear. It's comparable, roughly, to first gear going forward, so a car goes about as fast in reverse as it does going forward in first gear. The car doesn't go faster in reverse because there are no additional gears as there are for driving forwards. Now apply the rest of Tagishsimon's answer -- there are no further reverse gears because of cost, weight, lack of need, safety, etc. But the fundamental mechanical limitation is the single gear ratio. — Lomn 14:49, 30 June 2010 (UTC)[reply]
Actually most cars can go pretty fast in reverse gear, much faster than is safe anyway. I've always thought they should have a speed limiter in reverse gear. Maybe they do these days, it's a while since I tried accelerating hard in reverse.--Shantavira|feed me 15:04, 30 June 2010 (UTC)[reply]
Yes, you can go quite fast, in the same way that you can go quite fast forwards in 1st gear, but the car doesn't like it! (And neither does your fuel consumption.) --Tango (talk) 16:33, 30 June 2010 (UTC)[reply]
Some cars do go as fast in reverse as forwards...the Bond Minicar, for example doesn't have a proper reverse gear - so to drive backwards, you stop the engine - flip a switch that makes the starter motor run backwards and start the engine running in the opposite direction! There is one such family of cars, the DAF Daffodil and it's successors that used an early continuously variable transmission called "Variomatic" - fans of the Daffodil who restore these cars have races where everyone drives in reverse! The Dutch have a regular series of races where everyone drives in reverse - and the Daffodils had to go into a class of their own because they can go 100+ mph backwards! Here is a YouTube video of one race...but I don't think it's the Daffodil class because they typically race at '-60 mph'!
Also, there are indeed cars with speed limiters in reverse. The older BMW MINI ONE's (which also had continuously variable transmissions) have to be speed-limited in reverse in order to avoid shredding their wierd rubber drive belts. It's in the nature of any normally-gearless transmission system that in order to go backwards, you have to have a way to use auxilliary gears to get into reverse (often built into the differential casing) - and that naturally results in a car that goes as fast in reverse as forwards. Since that's obviously dangerous, using a speed limiter is a smart idea.
My (stick-shift) MINI Cooper'S can do close to 45mph in 1st gear - and the 1st and reverse gear ratios are almost identical - so it clearly could go 45mph in reverse. I don't intend to find out though!
SteveBaker (talk) 19:42, 30 June 2010 (UTC)[reply]
Steering while driving in reverse at high speed is not nearly as stable as when going forward at the same speed so experimentation as to how fast your car can go in reverse is probably not a fantastic idea. Googlemeister (talk) 20:09, 30 June 2010 (UTC)[reply]
See Steering#Rear_wheel_steering. Acroterion (talk) 20:30, 30 June 2010 (UTC)[reply]

I recall seeing an English film in which a couple were out driving in the early days of the automobile and their car did not have sufficient power to make it up a steep hill -- it kept stalling, even in 1st. They switched places and the woman backed it up the hill successfully because the reverse gear was geared lower than 1st gear. Does anyone here know (1) what film that was from and (2) how common it is (or was) to have the reverse gear geared considerable lower than 1st? -- 60.49.38.251 (talk) 11:05, 1 July 2010 (UTC)[reply]

In winter it's common to drive a Front-wheel drive car in reverse up a slippery slope to maximise the traction of the engine weight over the front wheels. Cuddlyable3 (talk) 11:24, 1 July 2010 (UTC)[reply]
No idea about the film, but lots of old (and newer) cars have reverse gears at higher ratios compared to the rest, resulting in more torque and less top speed for the gear. It only stands to reason, for all the explanations mentioned, that you would want the reverse gear to allow only slow, careful motion. Our own article on gear ratios proclaims this, at least for the 2004 Chevy Corvette. --144.191.148.3 (talk) 19:07, 1 July 2010 (UTC)[reply]

Acidity of warm water

since

  1. Is warm water more or less acidic than cold water?
  2. What happens when a carbonated beverage becomes warm?

--478jjjz (talk) 16:40, 30 June 2010 (UTC)[reply]

Pure water is neutral at any temperature, as far as I know (I can't think of any reason for it not to be). A carbonated beverage contains carbonic acid, so is slightly acidic. I don't know what happens to it when it changes temperature, though. --Tango (talk) 16:44, 30 June 2010 (UTC)[reply]
In practicality, water will have the same pH, whatever the temperature is. If there is a lot of carbon dioxide (such as you blowing breath through the water), then it will be more acidic when cold since more carbon dioxide dissolves in cold water.
Gases are less soluble in high temperatures than in low temperatures. So the equilibrium below would go toward the gas side. --Chemicalinterest (talk) 18:06, 30 June 2010 (UTC)[reply]
A solution is neutral when pH = pOH. This may or may not be 7, depending on the temperature. John Riemann Soong (talk) 19:03, 30 June 2010 (UTC)[reply]
I believe the pH of completely neutral boiling liquid water (no carbon dioxide) is 6.15. Now if you turn the pressure up and it boils at a higher temperature, you can increase the self-dissociation constant considerably. John Riemann Soong (talk) 19:14, 30 June 2010 (UTC)[reply]

CO2 (g) + H2O (l) ⇌ H2CO3 (aq)

What is the role of temperature in the above reaction?--478jjjz (talk) 16:49, 30 June 2010 (UTC)[reply]

Primarily because of entropy, when you warm a solution, otherwise solid solutes become more soluble, whereas gaseous solutes become less soluble. Warm water can hold less dissolved gasses (such as CO2) than cold water. When you boil water, this is the origin of the initial bubbles you see when the water is still below the boiling point. It's not the water turning into gas, it's the dissolved gasses coming out of solution as the water warms. -- 140.142.20.229 (talk) 17:16, 30 June 2010 (UTC)[reply]
Several things I should add: While absolutely pure water contains equal concentrations of hydronium and hydroxide ions, the absolute concentrations of them varies slightly with temperature (see autoionization constant for water). Also water, even pure water, rapidly picks up a small amount of carbon dioxide from the air, pushing the pH into the acid range. Finally, in addition to the temperature dependence on solubility, there is a temperature dependence on acid dissociation constants. I'm not sure what it is for the carbonic acid <==> bicarbonate transition, but for the bicarbonate <==> carbonate transition, it's enough to cause a 0.1 pH units or so difference per 10 °C change near room temperature. -- 140.142.20.229 (talk) 17:30, 30 June 2010 (UTC)[reply]
This one smacks of homework. We won't do homework, as a rule of thumb, unless you're stuck. The second one is an unusual question though, so I'll bite: gases are less soluble at higher temperatures, so the drink will become fizzier as you warm it. This Elmhurst College page has a bit more on it. Brammers (talk/c) 17:42, 30 June 2010 (UTC)[reply]
Gases are less soluble in hot liquids, so the equilibrium would go to the gas side. In cold temperatures, it will go away from the gas side. See Le Chatelier's principle for an explanation of equilibrium. --Chemicalinterest (talk) 18:06, 30 June 2010 (UTC)[reply]
The above is true, and is true for all gases. It's an entropy effect. Physchim62 (talk) 12:40, 1 July 2010 (UTC)[reply]

The pH of water varies with temperature, and this can occasionally have important chemical effects (I personally once spent three months studying one of them, but it never got to the point where it could be published). We say that water is neutral if it has equal concentrations of H+ and OH, but these are different chemical species with different chemistry. If you heat pure water, you will increase the concentration of H+ (and of OH at the same time), so you will increase the rate of any reaction which depends on H+ for its rate-determining step. Physchim62 (talk) 12:40, 1 July 2010 (UTC)[reply]

For a related topic on a larger scale, see ocean acidification. The oceans release more carbon dioxide from carbonic acid and are able to absorb less carbon dioxide from the atmosphere at a warmer temperature, but high atmospheric carbon dioxide concentrations that would likely be responsible for the warming result in more carbon dioxide going to the oceans and therefore more carbonic acid production and increased acidification. ~AH1(TCU) 18:28, 2 July 2010 (UTC)[reply]

I tried to sanitise my potato peeler by sonicating it in acetone

When I dipped it in acetone it seemed fine...then I turned the sonicator on and the solution turned all black! When I removed the peeler from solution half of the lacquer/varnish had been dissolved.

question: is the peeler still safe to use for food? (I brought it from home to peel plant/vegetable tissue for the lab, but still...)

Also, why would acetone do this, as opposed to methanol, ethanol or isopropanol? Or heavy-duty glassware detergent?(Nothing happened with those agents when I used them as sonicating solvents.) John Riemann Soong (talk) 19:21, 30 June 2010 (UTC)[reply]

Acetone is a more powerful solvent. I don't think that it is unsafe. Wash away the dissolved residue and let the acetone evaporate. --Chemicalinterest (talk) 19:23, 30 June 2010 (UTC)[reply]
Because acetone is a strong solvent for paints and plastics and is used to strip some kinds of paints and lacquers? It's commonly used as nail polish remover and to remove uncured urethane, and to my knowledge, it's not a particularly good disinfectant (or at least not better than), compared to, say, ethanol or isoproanol, and more toxic to people than either, at least until it's all evaporated. Acroterion (talk) 19:27, 30 June 2010 (UTC)[reply]
Actually acetone is not very toxic at all. It's a great solvent, but not a particularly good disinfectant (possibly because it's not very toxic). Ariel. (talk) 05:41, 1 July 2010 (UTC)[reply]
Yes, for short-term exposure, even to high concentrations, it's not too bad (and I rather like the odor), but occupational exposure to acetone-based paints and finishes can do harm. Urethane floor finishes once exclusively employed an acetone solvent/carrier, and I was acquainted with a number of floor finishers who were pretty well addled. Acroterion (talk) 12:22, 1 July 2010 (UTC)[reply]
Acetone does it because it lacks OH groups, which means that dissolution of non-OH containing solids such as lacquer is not unfavourable as it is in MeOH , EtOH , water/detergent etc.. but it is polar, which usually helps.
Other non hydrogen bonding solvents will do the same eg DCM, CCl4, MeOAc 83.100.252.42 (talk) 21:37, 30 June 2010 (UTC)[reply]
Is carbon tetrachloride isn't polar though....isn't it exceptionally oily? How it dissolve lacquer when hexanes wouldn't?
Also lacquer I think is composed of solvent-like polymers with lots of esters or carbonyl functionalities...do esters and nonprotic carbonyls generally prefer to bond to carbonyl carbons electrostatically rather than through hydrogen bonding? I suppose it's because carbonyl oxygens are weaker hydrogen bond acceptors? John Riemann Soong (talk) 22:02, 30 June 2010 (UTC)[reply]
1. CCl4 is much more polarisable. (and a fundamentally bigger molecule that hexane)
2. a. Yes your description of lacquers sounds right.
2. b. No. The idea related to the hydrogen bonds is that MeOH etc can form 2 hydrogen bonds with MeOH, but only one with C=O , that's why hydrogen bonding solvents show reduced dissolving power compared to other polar solvents for molecules that are not hydrogen bond donor/acceptors.
Non-protic polar molecules tend to dissolve ... generally .. the polar factor is usually small compared to the entropy effect. eg acetone and hexane mix. The energy of dipole dipole interactions is going to be fairly limited compared to the thermal energy in a liquid at r.t - any interactions may be frozen out in the solid though. A hydrogen bond to C=O is stronger than a dipole dipole interaction. 83.100.252.42 (talk) 22:35, 30 June 2010 (UTC)[reply]
Next time sonicate the peeler in water with detergent! Graeme Bartlett (talk) 12:10, 1 July 2010 (UTC)[reply]
Or use a brush.. I remember your question about cleaning a knife with a gas flame.. what next?! . "I used laser ablation to remove grease spots from a tea towel and I'm worried it's affected the fabric..." Sf5xeplus (talk) 12:52, 1 July 2010 (UTC)[reply]
Ewwww brushes. I was trying to kill microbes and remove any biofilms, not generate them. 15:50, 1 July 2010 (UTC)

Parasite?

Does anybody happen to be able to identify this (I suppose) parasite? (Sorry for the out-of-focus image.) Some context: several of this white pods appeared, together with whitish spiderweb-like filaments, on a lemon plant kept in a vase on a balcony in an apartment building in Rome (Italy). The largest pod is about 1 centimeter long. Thanks! [[::User:Goochelaar|Goochelaar]] ([[::User talk:Goochelaar|talk]]) 19:44, 30 June 2010 (UTC)

It is a kind of a scale insect, probably Margarodidae family. --Dr Dima (talk) 20:44, 30 June 2010 (UTC)[reply]
It really looks like the cottony cushion scale.--Stone (talk) 21:38, 30 June 2010 (UTC)[reply]
Thanks to both! It looks like it indeed. My lemon is not happy, but at least I have learnt something today. [[::User:Goochelaar|Goochelaar]] ([[::User talk:Goochelaar|talk]]) 21:55, 30 June 2010 (UTC)
You are welcome. The easy way to deal with those guys is simply to remove them using a cotton ball soaked in vodka or dilute ethanol. Don't soak the cotton ball too much, and don't let the alcohol drip on the soil or roots. You will need to repeat this several times, every other week or so. There are species of scale insects that would color the cotton ball bright red, this is normal (it's not blood, it's carmine). --Dr Dima (talk) 22:51, 30 June 2010 (UTC)[reply]
A lemon tree? Carmine? The OP might be able to make a Campari and Lemonade --Frumpo (talk) 08:05, 1 July 2010 (UTC)[reply]

how to remove cuticle from a leaf?

Is there any way of removing the cuticle of a leaf while keeping most of the cells underneath it alive? It's okay if some of them are injured. A surfactant + sonication, maybe? John Riemann Soong (talk) 19:50, 30 June 2010 (UTC)[reply]

this paper says that the plants they chose allow the leaf epidermis to be easily (?) removed with a dissecting needle and forceps. You will probably need a good stereo microscope and a steady pair of hands :) --Dr Dima (talk) 21:01, 30 June 2010 (UTC)[reply]
Interesting..... uhh I'll try it out tomorrow. Maybe there's a way to "weaken" the cuticle? Btw, if I dip the leaves in alcohol or something it won't kill the cells inside, should it? Maybe a few seconds of sonication then attempt to pry off the epidermis? (Btw, I assume the bottom of the leaf is covered with a water-impermeable substance too ... just less waxy...) John Riemann Soong (talk) 21:13, 30 June 2010 (UTC)[reply]

Extinct Volcanoes in North America and Asia.

Please give me the names and information about extinct Volcanoes in Canada, Alaska, The United States, Australia, and New Zealand. Also, any other outside Reference sources of Information I can consult. --24.193.198.15 (talk) 20:58, 30 June 2010 (UTC)[reply]

You can find the ones we have articles on in Category:Extinct volcanoes 83.100.252.42 (talk) 21:05, 30 June 2010 (UTC)[reply]
You may also be interested in Large igneous province, supervolcano and extinct volcano. ~AH1(TCU) 18:17, 2 July 2010 (UTC)[reply]

which type of plant cells (and species) tends to have the biggest / most permissive plasmodesmata

Are onion root cells a good choice, or are there other types I should look at? What are "root tip cells" btw? Could I get them from store-bought onions? John Riemann Soong (talk) 21:11, 30 June 2010 (UTC)[reply]

Do any nonhuman animals possess a conscience?

Or is it impossible to determine with certainty, as with love, mentioned above? --95.148.107.189 (talk) 21:52, 30 June 2010 (UTC)[reply]

As with love, how do we define it? There are definitely experiments that show that many other animals understand what we might call "fairness"—getting angry when they don't get equal treatment for other animals they consider their equals. See this story from NPR from a couple years back. Is that a "conscience"? Can we speak with any certainty about human emotions in non-humans? Do we trust Koko when she signs "mad" at the three kittens who have lost their mittens? --Mr.98 (talk) 22:34, 30 June 2010 (UTC)[reply]
(ec) You must be more specific in what you ask. Conscience means (1) ability to know good and evil, that is, to tell a moral right from moral wrong; and (2) an ability to feel remorse. AFAIK this is not directly related to the ability to feel affection or love. There exist definitions of conscience, consciousness, sentience, sapience, and self-awareness; knowing them will allow your question to be formulated more precisely (but not necessarily to be answered more fully, as the answers to most questions of that kind are not yet known). Too many a flame war has been fought due to inconsistent terminology on this subject; you would not want to be on the receiving end of most of it. Please restate your question more specifically. --Dr Dima (talk) 22:37, 30 June 2010 (UTC)[reply]
I'm not sure any of those terms have "precise definitions". In fact their articles are largely about the endless philosophical debates about what they might mean ("sapience" is the exception, because it foists the definition off onto "wisdom" and "sound judgment"). The plain fact of it is that these terms mean a lot to us as emotional/intellectual beings but applying them in a precise, "scientific" manner seems fairly impossible, because nobody can really agree on what is "precisely" meant by them (and any one "precise" definition ends up excluding other meanings that are seen as important).--Mr.98 (talk) 00:08, 1 July 2010 (UTC)[reply]
I said "definitions", I never said "precise definitions". By putting "precise definitions" in quotes you incorrectly cite the previous post. --Dr Dima (talk) 02:13, 1 July 2010 (UTC)[reply]
If you are very interested in animal intelligence, can I point you towards the books of Temple Grandin, especially Animals in Translation? They are a delight to read and extremely enlightening. If I can hazard a personal opinion, I think she does a good job of asking the "right" kind of questions about animal intelligence, and avoiding the pointless ones (which are really just proxies for "do animals have souls?" and other metaphysical dead-ends). --Mr.98 (talk) 00:11, 1 July 2010 (UTC)[reply]
If animals have any sense of morality, it must be very different from the human sense, and even that has a lot of intervariability. However, we cannot "test" to see if animals have a self-awareness, as brain scans fail to "detect" any material consciousness and humans assume that we (the self) have a consciousness but this delves into the realm of qualia. You might also like to try the thought experiment involving robots, as we cannot currently by definition create a "consciousness" in a robot just as we cannot test for a consciousness in either robots, other humans or animals. ~AH1(TCU) 18:13, 2 July 2010 (UTC)[reply]

1) According to the article neutron star, the star doesn't collapse further because of the Pauli exclusion principle. But the article also clearly indicates that a quark star or black hole will collapse further. Does this mean that the Pauli exclusion principle only applies up to the point at which the strong repulsive force that keeps the nucleus from imploding applies?

2) I'm still miffed by the whole black hole paradox. As person A approaches the black hole, his clock will continue to go slower and slower until it appears to the outside world that it stops (i.e., it's asymptotic). But as person A approaches the black hole, time doesn't stop for person A. What the heck is going on here? Magog the Ogre (talk) 22:31, 30 June 2010 (UTC)[reply]

For (1), yes. And for (2), that's just what happens. It's like in special relativity, where a clock moving relative to your frame of reference will go slower than one that is stationary, except that it's caused by the gravitational field rather than the relative motion of reference frames. Confusing Manifestation(Say hi!) 23:44, 30 June 2010 (UTC)[reply]

Are we to assume that guy A just gets torn apart right at the event horizon then? Because right at the edge, time will be going infinitely fast. So we're assuming the black hole falls apart via Hawking radiation in person B's time, which would be infinitely quick by person A's time? If so, that means that person A would get ripped apart before falling within the event horizon... I think... Magog the Ogre (talk) 01:11, 1 July 2010 (UTC)[reply]

No, the person A (Shall we call her Alice?) does not get torn apart at the horizon (assuming that the black hole is big enough that the horizon is not too close to the center of the black hole where alice will be torn apart by the singularity. See spaghetification). Also even though from the point of view of B (Shall we call him Bob?) Alice does halt to a stop as she approaches the horizon, from the point of view of Alice nothing special happens at the horizon and she does not see Bob spedding up to infinite speed. Dauto (talk) 03:31, 1 July 2010 (UTC)[reply]
I think you meant spaghettification. CS Miller (talk) 06:41, 1 July 2010 (UTC)[reply]
Yes, indeed. Thank you. Dauto (talk) 17:07, 1 July 2010 (UTC)[reply]

Yes, but this is a contradiction. Bob stops seeing Alice move, her time stops. So, um, when she sees Bob, doesn't she see him quicker and quicker? That's what Stephen Hawking maintains anyway. It violates the principle that one person's reality does not conflict with another's (I can't remember the name). Grr, this is the second or third time I've brought this up on here, is there some internet forum where physicists hang out? Magog the Ogre (talk) 00:43, 2 July 2010 (UTC)[reply]

You're thinking of Leonard Susskind and his book "The Black Hole War". But it's not really a paradox (even though it seems like one); as long as the two can never get together and "compare clocks", there's no problem. From Alice's point of view, she passes through the event horizon just fine, but gets spaghettified eventually. From Bob's point of view, Alice sits near the Event horizon forever. But Alice can't come back and ask Bob what happens after what happens after (to him) an "infinite" amount of time because she's stuck in the gravity well of a black hole. Bob sees one thing, and Alice sees another, but as paradoxical as it sounds, it ends up being OK. Buddy431 (talk) 02:37, 2 July 2010 (UTC)[reply]
Alice and Bob see different things but without contradiction (That's why the theory is called relativity).For instance, from the point of view of Alice there is no Hawking radiation, while from the point of view of Bob there is a surface just above the horizon (called the extended horizon) of incredibaly high temperature from which the Hawking radiation comes from. Which one is right? Both are. To reiterate what I said earlier, Alice doesn't see Bob speeding up to infinite speed. The idea that Bob and Alice have clocks that tick at different rates and that if one sees the other one faster than the latter must see the former slower is too simplistic and plainly wrong. It is wrong even within special relativity where when two observers pass each other both of them observe the other's clock to tick more slowly than their own. Dauto (talk) 04:34, 2 July 2010 (UTC)[reply]
A related concept is the Relativity of simultaneity, which is a function of these same "paradoxes" (being paradoxes only in the classical physics or intuitive sense). It is actually impossible to determine which of two events can be said to occur "absolutely" first. For any observable pair of events which are seperated by any arbritrary space, there will always exist some frame of reference where one occurs first, and a different frame of reference where the other occurs first. This is very similar to the idea that from Alice's frame of reference, she passes through the event horizon at a steady rate; and notices no observable change to herself at the event horizon in terms of her motion, but Bob observes her stop moving entirely. That's the core principle of the term "relativity"; there is no universal reference frame from which measurements can be made in an absolute manner. All reference frames are relative to each other. --Jayron32 04:51, 2 July 2010 (UTC)[reply]

Homogeneous Continuous-time Markov chains - analytical expressions for the finite time transition probabilities?

This looks like a question much better suited to the Wikipedia:Reference desk/Mathematics. Dolphin (t) 00:15, 1 July 2010 (UTC)[reply]

I have transferred this question to the Mathematics Reference Desk. See HERE. Dolphin (t) 00:30, 1 July 2010 (UTC)[reply]

July 1

Iron

Does the body use iron(II) or iron(III)? --75.25.103.109 (talk) 00:18, 1 July 2010 (UTC)[reply]

Both..oops Fe2+ probably = See Hemoglobin - especiallly Hemoglobin#Iron's_oxidation_state_in_oxyhemoglobin . Sf5xeplus (talk) 00:21, 1 July 2010 (UTC)[reply]
Both. Most iron containing proteins are (nominally) iron(II), but iron is transported around the body (by transferrin) and stored (by ferritin) as iron(III). Physchim62 (talk) 10:24, 1 July 2010 (UTC)[reply]

Expiration of medicine

What does it mean for medicine to expire? Assuming we are dealing with a pharmaceutical that doesn't become harmful as it ages, how much of the active ingredients can be lost before it is considered expired? —Preceding unsigned comment added by 173.49.77.61 (talk) 00:22, 1 July 2010 (UTC)[reply]

Usually drugs just become less effective as they get older. Some could become harmful though - don't risk it! --Tango (talk) 00:27, 1 July 2010 (UTC)[reply]
Why do they get less effective? --RampantHomo (talk) 01:14, 1 July 2010 (UTC)[reply]
Various reasons. Oxidation, decay due to exposure to light (that's why they are often in dark brown bottles), etc.. --Tango (talk) 02:23, 1 July 2010 (UTC)[reply]
How about a reference here at the Reference Desk? The first google hit on why does medicine expire yields this Medscape article, in which a medical doctor concludes that almost all drugs are fine to take after the expiration date, which he says is merely the last date at which the pharmaceutical company asserts the drug is still effective. On the other hand, he cites one example in which a guy may have damaged his kidneys by taking expired tetracycline. Comet Tuttle (talk) 03:01, 1 July 2010 (UTC)[reply]
As an example, I recall from my chemistry class that Hydrogen peroxide molecules (H202) will drop the extra Oxygen atom at somepoint, and as this happens to collectively more Hydrogen Peroxide molecules, the contents of the bottle become more and more water, and less and less hydrogen peroxide, thereby significantly reducing the effectiveness over time. Falconusp t c 03:34, 1 July 2010 (UTC)[reply]
Hydrogen peroxide is a special case - it's very unstable. Most (pretty much all) medicines are far far more stable. Concentrated hydrogen peroxide is unstable enough to be used as rocket fuel. Ariel. (talk) 10:32, 1 July 2010 (UTC)[reply]
Important distinction: it's important that a rocket fuel is "high energy" (thermodynamically unstable) not that it rapidly/spontaneously decomposes (somewhat unrelated kinetic issue, which is sometimes even a negative quality to have). Lots of excellent rocket fuels are easy to handle and store until they are made to react with a large release of energy. Some fairly low-energy molecules are labile enough that they can change slightly under mild conditions, leading to loss of intended activity. DMacks (talk) 14:17, 1 July 2010 (UTC)[reply]
I should have said "unstable enough to be used as a monopropellant rocket fuel". Ariel. (talk) 14:52, 1 July 2010 (UTC)[reply]
If the drug is not harmful after it expires, it may even still work due to the placebo effect. ~AH1(TCU) 18:06, 2 July 2010 (UTC)[reply]

Help identifying this flower

I would like to know the species of this flower so I can add the image to the correct article, but I haven't got a clue! The background info for this photo is:

  • In the UK in Oxfordshire
  • Taken in March
  • Found in open cut grass

Thanks in advance - Zephyris Talk 00:47, 1 July 2010 (UTC)[reply]

Is it not speedwell (Veronica (plant) of some variety)? It looks like the thing I've always called speedwell, and if I had to guess from the articles I'd say it was Veronica persica. But hopefully a botanist will help you with the more specific species. 86.164.57.20 (talk) 01:40, 1 July 2010 (UTC)[reply]
Looking it up in Complete British Wild Flowers by Paul Sterry, which has photos of many kinds of Speedwell, then it could be Veronica persica, polita, or agrestis. V. persica - reddish stems, "white on the lower lip of the corolla". Cannot see a reddish stem, but the other part matches. The photo shown in that Wikipedia article is perhaps the wrong species therefore. 92.28.244.45 (talk) 09:57, 1 July 2010 (UTC)[reply]
Thanks guys, thats brilliant. From those descriptions it must be V. polita, I will replace the image on that page with this one. - Zephyris Talk 13:13, 1 July 2010 (UTC)[reply]
I thought I had typed V. persica above, not polita. Anyway I have now corrected it, and the desrciption corresponds to V. persica, and the V. polita article photo needs to be changed back. 92.28.247.183 (talk) 19:15, 2 July 2010 (UTC)[reply]

Neodymium magnets

Do the neodymium and iron in neodymium magnets (Nd2Fe14B) oxidize over time? --75.25.103.109 (talk) 01:02, 1 July 2010 (UTC)[reply]

Not sure but they are almost always coated (ie Ni plated)...
" Neo magnets or rare earth magnets have poor resistance to corrosion and should have a coating or plating applied" [5] . Not sure of the details.Sf5xeplus (talk) 01:31, 1 July 2010 (UTC)[reply]
"Sintered Nd2Fe14B tends to be vulnerable to corrosion. In particular, corrosion along grain boundaries may cause deterioration of a sintered magnet. This problem is addressed in many commercial products by providing a protective coating. Nickel plating or two layered copper nickel plating is used as a standard method, although plating with other metals or polymer and lacquer protective coatings are also in use" from Neodymium magnet .. link from that article http://www.journalamme.org/papers_vol20/1369S.pdf Sf5xeplus (talk) 01:33, 1 July 2010 (UTC)[reply]
From first hand experience: Yes they rust! I have several of those magnets ripped out of hard disks. The glue they use to fasten it to the holder is very very good so I torn of the nickel layer and now the magnet is not protected at that point. After several weeks this area get redish coloured and if you touch it you have rust on your fingers. The material is sintered and has a large surface area making it vulnerable for corrosion along the former grains.--Stone (talk) 05:05, 1 July 2010 (UTC)[reply]

Weight loss

I was watching a comedy/sitcom TV series and there is a woman who is a bit...*ahem*... "un-thin" but she is not at all overweight or fat. She came up with this insane regimen to lose weight, eg eating nothing but 1 salad and coffee 3 days of the week and eating normally the other 4 but running 3 miles and biking 10. This made me think: how would a person who is of a somewhat higher but not immediately health-threatening weight lose weight? Like what kind of everyday changes should they make. This is on a hypothetical level of course, not for medical advice, since I noticed every diet seems to be geared only to the REALLY overweight people.—Preceding unsigned comment added by 68.248.225.254 (talk) 01:57, 1 July 2010

There is one and only one way of losing weight and it applies to everyone, regardless of their starting weight: you consume fewer calories than you burn. That is it (well, you could get liposuction, I guess). You can do that by eating less, exercising more or a bit of both. The important thing is to make sure you still get enough nutrition despite reduced calories - you still need plenty of protein, fibre, vitamins and minerals and a little fat. If you are interested in losing weight yourself then a nutritionist will be able to work out a diet for you that will have all the nutrients you need while having a certain number of calories based on the amount of exercise you will be doing. We can't give specific dietary advice here - there are too many factors involved. --Tango (talk) 02:31, 1 July 2010 (UTC)[reply]
The real trick is finding a method to ensure you actually do eat less than you expend. The diet mentioned above might work for that for that or another person, it might not. I've been fat since high school and tried several diets and exercise regimes and found the following: Exercise 3 times a week works a little for me, and at least makes me feel better. Low calorie diets don't work for me (I can't handle being always hungry). Low fat diets don't work for me (I can easily get or stay fat on bread alone!). Low carbohydrate diets (specifically strictly following the Atkins diet) worked (120kg to 98kg over 3 months) for me on my third attempt - it took me that long to learn that the diet doesn't work if I cheat or if I don't eat often enough. But everyone is different.
Every day changes that I've seen help people: Exercising more - cycling or walking to work or school; not snacking; eating 5 times a day rather than 3; eating only and exactly 3 times a day; not eating high fat food; not eating high calorie food; not eating processed food; cutting out a particular food (crisps, chocolate, bread, etc). --Psud (talk) 08:53, 1 July 2010 (UTC)[reply]
The general view is that diets don't work lifestyle changes do. There seem to be three main areas, actual nutrition, exercise and the psychology of nutrition and exercise.
The main thrust of most of the work I've seen has been that the most enduring weight loss is slow, over several months.
As with Tangos comments, it's a question of eating a balanced, healthy, diet and adding in some exercise. The exercise needn't be formal, but increasing exertion in daily life on a regular basis.
One of the main concerns about the psychology is that published diets are seen as some kind of magic bullet, rather than a stimulus to actually change behaviours. Hence the type of club based weight loss system where there is both mutual support, and tacit peer pressure.
As with Psud there are little changes, smaller more regular meals, although some of those are little more than formalised snacks. Snacking on fruit, nuts etc rather than processed products etc.
ALR (talk) 09:47, 1 July 2010 (UTC)[reply]
Ah, yes, thank you for making that point - I forgot to. Your new diet needs to be something you can happily stick to for the rest of your life, or you will almost certainly put the weight back on again. --Tango (talk) 15:59, 1 July 2010 (UTC)[reply]
I've recently started to think about a simple rule for health and weight loss: never eat processed foods. Only eat food in the same form it was alive, apart from cutting, and avoid meat also. Following this rule you would avoid the salt, fats, and sugars that routinely tempt us to eat too much. Something I do is to eat a lot of vegetables, as this fills you up, and fruits also. 92.28.244.45 (talk) 10:10, 1 July 2010 (UTC)[reply]
While there is merit to the above, you need to be careful that you are still getting B vitamins and protein if you are going to cut out meat completely. While I agree that vegie diets are easier to reduce calories, you do need to be aware of the nutrition provided by meat and ensure that you are including it. Googlemeister (talk) 13:32, 1 July 2010 (UTC)[reply]
You are probably thinking of Vit B12, which is abundant in fish and eggs. People also greatly overestimate the amount and quality of protein that adults need (according to these pages), and excess protein is now thought to be bad for you. Sardines are extremely rich in Vitamin B12, more than anything else you would eat I think. I suggested avoiding meat because of the saturated fat it contains, particularly with modern farming methods. The B vitamin folic acid is mostly obtained from vegetables I believe, as well as other B vitamins, although I am not an expert on individual B vitamins. If you are eating a diverse diet, then eating some red meat is going to be more harmful than good for you. 92.28.247.183 (talk) 19:22, 2 July 2010 (UTC)[reply]

I agree with ALR that diets don't work. I've read that this is because the logic of "eating less than you burn" does not apply. As long as you're not on a diet that severly restricts calory intake (say eating less than 1500 Kcal per day), the body will regulate its metabolic rate such that the fat cells stay filled to some fixed level. This also means that if you do lose weight as a result of sticking to a diet of, say, 1500 Kcal, you will eventually return to approximately your old weight if you start to eat a normal amount of kcals again, even if that is still less than what you were used to eating.

What does seem to work is eating healthier, sleeping better and getting more exercise. It could be that if you do this, the body will decide to keep the fat cells filled at a lower level. Count Iblis (talk) 15:39, 1 July 2010 (UTC)[reply]

You make a good point. I forgot to say that you need to consume only slightly fewer calories than your burn - your intake should stay at this new level for the rest of your life. Rapid weight loss doesn't work, you will almost certainly put the weight back on again. Your weight is a key factor in determining how many calories you burn, so as you lose weight the calories you are burning will reduce until you reach a new equilibrium and then you'll stay at your new weight. If you reduce your calories to 1500kcal/day then you won't reach an equilibrium before you starve, which is why you'll end up increasing your intake again. For a typical lifestyle, most people need around 2000-2500kcal/day. If you are overweight, chances are you are consuming more than that (and burning more than that due to your weight - most overweight people aren't constantly gaining weight). You shouldn't try to reduce your intake to less than 2000-2500, you should reduce it to 2000-2500. --Tango (talk) 15:59, 1 July 2010 (UTC)[reply]
See somatotype and constitutional psychology. For some people it will be more difficult to "lose weight". ~AH1(TCU) 18:03, 2 July 2010 (UTC)[reply]

Suntan

I was playing tennis but I was wearing sandals and did not think to put sunscreen on my feet. Now they have some weird tanned stripes where the holes elt sun in. How long does it take to get "un-tan"? Is there any way I can speed up the process of getting un-tan, whih I assume is my body metabolizing melanin? THNX 68.248.225.254 (talk) 01:57, 1 July 2010

It's going to depend on your exposure to sun and varies from person to person, but it can easily be weeks. It would be quicker to just tan the rest of your feet, but I wouldn't recommend that - intentionally exposing yourself to sunlight is generally not to be recommended. You could try a fake tan, of course. --Tango (talk) 02:38, 1 July 2010 (UTC)[reply]
"Intentionally exposing yourself to sunlight is generally not to be recommended." Not recommend by who? Vampires? I am going to go out on a limb here and say that moderate exposure to the sun is generally not a major health concern. And I will base that statement off of the 6,000,000,000 or so people on this planet who encounter sunlight on a regular basis without keeling over. Granted if you are going to overdo it that results in a slight increase in your chance of skin cancer, but that doesn't mean you should not go out during the day. In fact, if you are not exposed to sunlight, you can have vitamin D deficiencies, so telling people not to go out in the sun can be detrimental to their health. Googlemeister (talk) 13:27, 1 July 2010 (UTC)[reply]
I said "intentionally". That means going outside for the purpose of exposing yourself to sunlight. Going outside for other reasons and just happening to be exposed to sunlight is a different matter entirely - you have to weigh up the benefit from whatever you are doing with the harm from the sunlight, and as long as you don't overdo it it is often worth going outside. The amount of sunlight required to get enough vitamin D is minimal - in fact, with a decent diet, it can be none at all, at most it's about 10 minutes with only your face and arms uncovered on an overcast day at temperate latitudes (for someone with reasonably fair skin). --Tango (talk) 16:06, 1 July 2010 (UTC)[reply]
The cells in the epidermis (the outer layer of skin) don't "untan" - they just die and eventually slough off. That process takes 27 days after the cell is first formed. In effect, you grow an entire new skin every four weeks! So if an epidermis cell was 'tanned' right after it was formed then the tan might not completely fade for 27 days. I think that's the worst case - but in practice, I doubt the suns' rays are able to tan those deepest layers of skin, so probably only the outer layer of 'older' cells got tanned - and they'll be around for less than 27 days. SteveBaker (talk) 03:50, 1 July 2010 (UTC)[reply]
Wash your feet a lot - it helps get rid of the surface of the skin. 92.28.244.45 (talk) 10:16, 1 July 2010 (UTC)[reply]

wth? How do you tan that quick? John Riemann Soong (talk) 16:02, 1 July 2010 (UTC)[reply]

A few hours of strong sunlight is more than enough for someone with reasonably fair skin to tan. Remember, the OP wasn't necessarily tanned enough for someone to notice them as being significantly tanned, just enough for there to be a noticeable difference between neighbouring tanned and untanned areas of skin. --Tango (talk) 16:08, 1 July 2010 (UTC)[reply]
Consistently wearing sandals in the summer will give you a "sandal tan". I find it usually fades by the wintertime. ~AH1(TCU) 18:01, 2 July 2010 (UTC)[reply]

Reptiles + amphibians

I currently have a 29 gallon fishtank that I'd like to transform into a aquaterrarium. I'm looking to drop the water level to about a fourth (and maintain the 7 fish I have now), add some newts and then put in 2-3 thick branches and have anoles in there as well. I'm under the impression that newts possess special toxic slime that they exude upon attack that will protect them from the anoles. Would that be a plausible set up? DRosenbach (Talk | Contribs) 02:31, 1 July 2010 (UTC)[reply]

Well many have poisonous skin eg Rough-skinned newt (also Newt#Toxicity), I suppose this would be easier to answer if you told us the type of newt.
Even so it's not clear that the anole would attack the newt (relative sizes?), or indeed that it would be aware of a poison, or that the poison actually acts as a deterrent (eg bitter) rather than species attrition.
Additionally I would wonder if the anoles and newts would be compatible in terms of their ecosystem - eg dry/damp??
87.102.17.114 (talk) 11:47, 1 July 2010 (UTC)[reply]

I would say fire-bellied newts, but I may get another type -- unsure. DRosenbach (Talk | Contribs) 03:35, 2 July 2010 (UTC)[reply]

Surface tension

On the surface of a liquid, there aren't any molecules above the surface molecules, hence they experience an attractive force towards the interior of the liquid. But what causes the increases force tangential to the surface? 70.52.45.181 (talk) 05:34, 1 July 2010 (UTC) Further questions: I'm looking at the following link, http://www3.interscience.wiley.com:8100/legacy/college/cutnell/0471713988/ste/ste.pdf. When considering how to define surface tension, the article makes reference to a C-shaped apparatus. Why is γ = F/2l and not F/l? It says something about there being two surfaces, but I only see one. Second question: In example one, the article considers the surface tension as applying a force outwards. But I thought surface tension was only inwards. I can see why compressing the liquid will produce some outward force against the needle, but why would that equal γL? It seems like those are two different phenomena. Thanks a lot guys! 70.52.45.181 (talk) 06:36, 1 July 2010 (UTC)[reply]

ok First question - there isn't really an attractive force towards the centre of the liquid (yes I know the pdf says that).. The molecules on the surface are in equilibrium, so the net force is 0. Obviously if you try to pull a molecule from the surface of the liquid then there will be a force resisting that. maybe this seems pedantic
Following on, still answering the first question: are you familiar with the Energy=force x distance relationship (or Force=dE/dx) - I think it makes surface tension a lot easier to understand... if so you can assume that the molecules at the surface are at a higher energy (since they are surrounded by less molecules .. and molecules gain energy by interacting with those next to them) .. Then assume that this 'surface energy' is proportional to area ie E=kA .. (k is a constant for air/water interface) - if you can work with that then it is fairly easy to show the answer to the second question without "hand waving" - it also proves that water surfaces generally form the shape of lowest surface area (ie a water droplet is spherical).
if not a different explanation will be necessary - but it gets a bit fiddly explaning just with text
Your third question - is this about a needle in water? (ie Example 1) - again - if you consider the surface area you will see that the surface tension acts in such a way to minimise the surface area (in this case it means flat) - so if the needle pushes the water down, the surface tension will try push the water back up into flatness.
Alternatively (3rd question) - try drawing the 'force arrows' in the water surface around the needle (as in diagram b page 2) - you will see that some of the force arrows are pointing upwards (more that 180 degrees of force arrows) - giving a net upwards force.83.100.252.42 (talk) 11:24, 1 July 2010 (UTC)[reply]
Sorry, if you don't mind I would like things to be kept to forces and such...grasping something in terms of energy and surface area-minimization is fine, but my understanding of this thing from a force perspective leads me askew, and that's something I have trouble living with. Sorry! 65.92.5.151 (talk) 03:26, 2 July 2010 (UTC) (PS my IP seems to be changing; don't worry it's OP, not an imposter :) ). 65.92.5.151 (talk) 03:27, 2 July 2010 (UTC)[reply]

Plaiting and fluffiness

When my hair has been plaited for more than a couple of hours, untwisting the plait and brushing my hair makes it go very fluffy. It eventually "calms down" hours later but can be pretty wild directly after this sequence.

My hair is not normally fluffy and it doesn't "fluff" after brushing any other time.

It's very wavy in thick sections when it comes out of the plait and I'm guessing this has something to do with the subsequent fluffiness, but I'm missing the connection or reason. Does anyone know why plaiting produces the effect? I tried searching for an answer, but it seems plaiting is a common answer to fluffiness and I therefore get a lot of false positives, :) Maedin\talk 06:57, 1 July 2010 (UTC)[reply]

Does a penis have other functions besides urination and copulation?

Does penis have any other functions besides urination and copulation? For example, helping maintain balance as a man ealks (nude), or being a primary way to get off access heat during a heat wave, or being a sensory organ regarding thr environment, like a nose, or - anything at all? 92.224.206.50 (talk) 07:21, 1 July 2010 (UTC)[reply]

Re "sensory organ": Two men were peeing off the end of a pier at night when one remarked, "Gee, the water is pretty cold." The other replied, "And deep, too." -- 60.49.38.251 (talk) 09:58, 1 July 2010 (UTC)[reply]
Writing one's name in the snow. Cuddlyable3 (talk) 10:36, 1 July 2010 (UTC)[reply]
Masturbation? Which is used by some as "stress reliever"? --Enric Naval (talk) 10:40, 1 July 2010 (UTC)[reply]
The penis provides the site for visible circumcision that is performed for ritualistic or religious purposes. Any usefulness for maintaining balance is disproven by the considerable numbers of women and men with tight trousers who do not fall over. Cuddlyable3 (talk) 10:43, 1 July 2010 (UTC)[reply]
As far as balance goes, if it's long enough, you can use it as a tripod. --Enric Naval (talk) 10:56, 1 July 2010 (UTC)[reply]
Its dysfunctions and complications can be useful indicators of health problems; hypertension, for example, or a hormonal imbalance. More indicative, of course, if the onset of said side effect is sudden and/or the age isn't elderly. Though clearly not its primary application, it's still a gauge that females don't necessarily have (or at least not as obviously). Maedin\talk 12:33, 1 July 2010 (UTC)[reply]
It functions as a differential analyzer of the attributes "surprise" and "panic". Surprise is the first time it won't the second time and panic is the second time it won't the first time. Cuddlyable3 (talk) 13:13, 1 July 2010 (UTC) [reply]
It's also a good MacGuffin for troll-like questions. --Tagishsimon (talk) 09:45, 2 July 2010 (UTC)[reply]

Credibility of hypotheses

What is it about a particular scientific hypothesis that makes it likely to reflect reality, or unlikely to reflect reality?--220.253.100.166 (talk) 08:49, 1 July 2010 (UTC)[reply]

Experiment :) --Dr Dima (talk) 10:12, 1 July 2010 (UTC)[reply]
See the article Falsifiability. Cuddlyable3 (talk) 10:34, 1 July 2010 (UTC)[reply]
A hypothesis is just an idea - until it's tested in some way, we don't know whether it's true or false. However, some hypotheses are more likely to be true than others. For example: You might hypothesize that I'll type the letter "e" more than once in the next sentence. Sadly, that turns out to be wrong. It was a good hypothesis - but until it was tested, we didn't know whether it was true or false. In general, a hypothesis that requires a violation of the known laws of physics/chemistry/whatever is much less likely to be false than one that doesn't. Overturning some piece of well-established science is much less likely than proving something that's already known to be possible. If you hypothesize that T.Rex was able to run at 20mph, that's not an impossible thing - and by measuring fossil bones and looking for T.Rex footprints - you might be able to prove it...but if you hypothesize that the jet of gasses from a pulsar shoot out at twice the speed of light - then we're very nearly certain that this is false - just from general physics principles. SteveBaker (talk) 19:09, 1 July 2010 (UTC)[reply]
Only an omniscient being could answer this question, since the rest of us have no sure knowledge of the underlying reality. The hypotheses are the only scientific guide we humans have to reality. Occam's razor is helpful in keeping things simple. William Avery (talk) 19:54, 1 July 2010 (UTC)[reply]

Career in nanotechnology?

[I'm asking this question on behalf of a friend] I'm almost done my undergraduate studies in physics and I have started, for the first time really, to consider my career options. I've always enjoyed physics, and I would absolutely love to continue doing research in it, but I have been cautioned about the dangers of entering a overly theoretical discipline of physics. But there are plenty of fields with "real-word applications" that are certainly very interesting. In particular, something like condensed matter physics or theoretical chemistry seem worthwhile to pursue. However, a few weeks ago an old teacher of mine suggested that I look into nanotech, nanoelectronics in particular. It seems fascinating, and the technology and lab equipment are very impressive. My sole caveat is this: will I lose the physics I enjoy if I go into this? I want to work with the equations and study the mechanics of things...will that be jeopordized in something like nanotech? Thanks. 70.52.45.181 (talk) 10:52, 1 July 2010 (UTC)[reply]

Nanoelectronics involves or can involve a lot of condensed matter physics, and some theoretical chemistry (of the right sort) is also applicable - so your friend can carry on as normal :)
83.100.252.42 (talk) 11:31, 1 July 2010 (UTC)[reply]
Nanotech is a wide range of different things - from the very practical stuff like improved water filtration for use in the third world - to the highly theoretical stuff like Molecular assemblers. See List of nanotechnology applications. These days, saying that you want to work in Nanotechnology is like saying you want to do Math...it's used all over the place in a vast range of industries. I think this means that you should be able to find a niche somewhere between the so-practical-it's-just-engineering to the so-unlikely-that-research-funding-is-unobtainable. Somewhere in that spectrum, you should be able to find a job that pays well and lets you do the science you love. In case you didn't already check it out, we have lots of articles on the subject, including Nanoelectronics itself - and also Molecular electronics, Molecular logic gate, Molecular wires, Nanocircuitry, Nanowires, Nanolithography. NEMS, Nanosensor, Nanoionics, Nanophotonics and Nanomechanics. SteveBaker (talk) 18:57, 1 July 2010 (UTC)[reply]

Thanks to both of you for the informative and detailed responses. 65.92.5.151 (talk) 03:23, 2 July 2010 (UTC)[reply]

Wasps

A wasp (either a yellowjacket or common wasp, I think) somehow got into my bed this morning, with predictably hilarious consequences. While I treated the stings with hydrocortisone cream, they were still extremely painful, even after the swelling had gone down. So, my question: Bees use apitoxin and wasps apparently use "a chemically different venom designed to paralyze prey, so it can be stored alive in the food chambers of their young". What is the mechanism which causes the prolonged pain in bee and wasp stings? Apitoxin says that bee venom contains proteins which cause inflammation, but that some of the components are anti-inflammatory agents. Why this apparent contradiction? --Kateshortforbob talk 11:10, 1 July 2010 (UTC)[reply]

I'm no doctor but from the sound of the article, apitoxin is multi-faceted. Some ingredients are anti-inflammatory while others are inflammatory, but not all work at the same time or on the same tissue. I suspect this dichotomy is part of why it hurts so much, the venom does different things at different stages ensuring that the wound will continue to cause pain and elude the body's self defense mechanisms. --144.191.148.3 (talk) 18:52, 1 July 2010 (UTC)[reply]

A hydrophobic cup?

What if a drinking cup has its inner surface lined with a hydrophobic material and water is poured into it? 67.243.7.245 (talk) 12:34, 1 July 2010 (UTC)[reply]

You've never drunk out of a polystyrene cup? Physchim62 (talk) 12:41, 1 July 2010 (UTC)[reply]
Cup holds water then. This happens a lot - paper cups with water proof coatings for instance. Was there some particular detail you where interested in? Sf5xeplus (talk) 12:54, 1 July 2010 (UTC)[reply]
Paper cup describes all the hellish details in full measure.Sf5xeplus (talk) 12:55, 1 July 2010 (UTC)[reply]
But water remains in a PS cup. It's not completely hydrophobic then? 67.243.7.245 (talk) 20:53, 1 July 2010 (UTC)[reply]
It is completely hydrophobic. Hydrophobic materials do not counteract the force of gravity; they merely are not miscible with water. That's all hydrophobic means. It just means that the material does not dissolve in or mix with water. They do not repel each other with a force in the way that, say, two north poles of a magnet will. The simply don't mix. When oil (a hydrophobic substance) and water are shaken, the seperate, but not from any force more complex than gravity; the less dense oil floats to the top because it is less dense, and thus lighter per unit volume, than the water. They aren't "forced apart" by any force. Likewise, water placed in a hydrophobic cup will simply sit in the cup and not soak through it. If you made a cup of hydrophilic material, like say salt, the water would dissolve the cup, making it not very useful as a cup. --Jayron32 05:54, 2 July 2010 (UTC)[reply]
You will get a convex Meniscus. Conversely, in a hydrophilic cup the meniscus will be concave. Ariel. (talk) 14:50, 1 July 2010 (UTC)[reply]

the cup shivers uncontrollably and cowers away from the water, if you persist in pouring water onto it, it "shuts down", curling into the fetal position and crying silently. If it is a prolonged experience in which the cup cannot get away, it can be permanently deformed psychology, perhaps hardly speaking a word for the rest of its life (or until treated). medications can help, but "water therapy", the naive idea that you should fight fire with fire, and just immerse the cup completely until it overcomes its fear, has disastrous results and is totally discredited; don't even think about it. 92.230.234.237 (talk) 13:08, 1 July 2010 (UTC)[reply]

be VERY careful around hydrophobic cups, in case they bite you.... Physchim62 (talk) 16:15, 1 July 2010 (UTC)[reply]

Help identifying this fruit

Unknown fruit.

I took this picture of a fruit on a tree in the botanic gardens in Oxford in February and have absolutely no idea what it is! Can anyone help? The fruit itself was around 4-5 cm diameter. Unfortunately as it was in the botanic gardens I have no idea of where in the world it is native to and the tree itself was fairly nondescript; ~7-10 metres tall with a trunk diameter of around 40-50 cm. The time of the year also meant there were no leaves for reference although the fruit was happy to stay on the tree over the winter - it was absolutely covered in these fruits. - Zephyris Talk 13:21, 1 July 2010 (UTC)[reply]

Umm... not to be glib but if you were at the Oxford Botanical Gardens then why did not you just ask a botanist? 76.199.154.122 (talk) 17:46, 1 July 2010 (UTC)[reply]
Well I took a picture of what I thought was the species label for the tree... Turned out to be the species label for the miniature daffodils under the tree! - Zephyris Talk 19:30, 1 July 2010 (UTC)[reply]
Do you remember if the tree had thorns? If it did, this might be a Bael. Googlemeister (talk) 18:26, 1 July 2010 (UTC)[reply]
I don't think it did, see the additional image. - Zephyris Talk 19:30, 1 July 2010 (UTC)[reply]
Here's the number: 01865 286 690 you can ask them what's the tree above the daffodils. 71.161.46.51 (talk) 21:03, 1 July 2010 (UTC)[reply]
You might also find their website useful. It includes an interactive map (which regrettably does not identify individual plants) and an e-mail address for queries. 87.81.230.195 (talk) 05:43, 2 July 2010 (UTC)[reply]
They haven't been much help have they? My money is on the Dove-Tree or Ghost-Tree, Davidia Involucrata[6]. "Fruit: 3x2.8cm, obovoid (ie egg-shaped with the big end at the opposite end to the stalk), much ribbed, deep green and slightly glossy, ripening dark purple. Pedicel (ie stalk) 10-14cm, much swollen at the fruit end." (A Field guide to the Trees of Britain & Northern Europe, Alan Mitchell, Collins 1974). The English names come from the spectacular large white oval bracts in late May. "Also regrettably known as the 'Handkerchief Tree'" says Mr Mitchell. Discovered in W. China in 1904; the remarkable story of it's introduction to Western science can be found here[7], pages 40-41. A photo of the Oxford Botanical Dove Tree in full foliage is here[8]. Alansplodge (talk) 18:48, 2 July 2010 (UTC)[reply]
The dove-tree looks very plausible! I think I would put my money on that... - Zephyris Talk 22:20, 2 July 2010 (UTC)[reply]
Found another picture[9]; scroll down to page 14. Is this the one? Alansplodge (talk) 22:44, 2 July 2010 (UTC)[reply]
Could it be a variety of walnut? Walnut shells are hidden inside a covering that looks like a fruit when growing. 92.28.247.183 (talk) 20:03, 2 July 2010 (UTC)[reply]
I thought of that but walnut trees have big chunky twigs[10], the nuts have short stalks and no ribs. Alansplodge (talk) 22:39, 2 July 2010 (UTC)[reply]

Would the car explode?

If something happened and a car's intake valve didn't shut all the way (or the valve suddenly developed a serious fissure while the car was in use) and the compression stroke came back up and the spark plug went off, is there anything stopping the combustion flame from streaking back through the lines to the fuel tank and causing a catastrophe?20.137.18.50 (talk) 14:04, 1 July 2010 (UTC)[reply]

The fuel line is full of liquid fuel. You need air in order to support combustion, so it would be hard to propagate the flame back very far prior to where air is mixed with the fuel vapors. DMacks (talk) 14:09, 1 July 2010 (UTC)[reply]
A Carburetor if there is one, also makes a barrier since the fuel must go through a narrow nozzle.87.102.17.114 (talk) 14:26, 1 July 2010 (UTC)[reply]
If a valve gets stuck open, you don't get compression and the fuel doesn't ignite properly. Aside from the fact of there being no air in the fuel lines, there isn't likely to be any combustion either. What happens is that the car sputters along, sounding really rough and developing very little power, because it's only running on three cylinders (assuming it's a four cylinder car that is!). Gasoline really doesn't burn very easily unless it's under hot or under pressure and in vapor form mixed with air. SteveBaker (talk) 18:39, 1 July 2010 (UTC)[reply]
A backfire in the intake is not unheard of on poorly maintained cars for a variety of reasons (such as timing, like you mentioned), and while it can easily damage intake manifold parts like rubber seals or sensors it is not likely to cause the fuel lines to catch fire for the aforementioned reasons. --144.191.148.3 (talk) 18:42, 1 July 2010 (UTC)[reply]
Yep. Remember - the car is a four-stroke machine. On the first down-stroke, gas/air is sucked into the cylinder. On the up-stroke, it's compressed...only the valve isn't closed so this just pushes most of the gas/air mixture back out again. Then the spark goes off - but there's hardly any gas/air mixture to ignite - and it's not hot and under pressure like it should be - so it probably won't ignite. The lack of a fuel burn means that there is nothing to push the piston back down again - aside from the inertia of the car and flywheel...but it'll go back down - sucking in whatever fuel and air is left in the cylinder. Then the exhaust valve opens and the piston goes back up - pushing most of whatever was in there out to the exhaust. But not all of the exhaust gasses will go out - lots of them will be pushed back to the carb. SteveBaker (talk) 18:45, 1 July 2010 (UTC)[reply]

How much energy required to reach infinite speed?

I can't remember the equation for how much energy is required to reach a given speed. What is it again? And what do you get when you feed infinity through it? Presumably no answer, since reaching c takes infinite energy.--92.251.129.172 (talk) 15:04, 1 July 2010 (UTC)[reply]

E=1/2 mv2 where E=kinetic energy, m = mass and v= velocity. So, yes, at v= infinity, E = infinity. This is the non-relativistic definition of energy and velocity, so I am sure there are adjustments to be made for speeds close to light speed, however these adjustments actually make it worse; you hit infinite energy at the speed of light (3.00 x 108 meters per second). So, you cannot ever reach the speed of light. It's a pain in the ass for science fiction authors, but the speed of light is a hard limit. --Jayron32 15:13, 1 July 2010 (UTC)[reply]
I thought they usually invented some warp drive or hyperdrive that somehow let them past lightspeed?--92.251.129.172 (talk) 15:25, 1 July 2010 (UTC)[reply]
Such things are utterly impossible given the laws of physics as we know them. But science fiction is fiction (meaning "it ain't all true"!)...and they are free to invent anything they want in order to make the plot work...and hand-waving away the speed of light limit is something you pretty much have to start off doing if you want people to go to other stars. A few science fiction authors don't do that...but they are certainly in the minority. SteveBaker (talk) 18:27, 1 July 2010 (UTC)[reply]
Hence "invented".--92.251.158.103 (talk) 19:59, 1 July 2010 (UTC)[reply]
Yes. The one E.E. Doc Smith invented for Lensmen just got rid of the pesky m in the above equation, so you only have to overcome the resistance of the interstellar medium. Unfortunately, they answer still is infinite (and the original question is physically meaningless - you cannot go faster than the speed of light without violating causality, no matter what your trick is). --Stephan Schulz (talk) 15:29, 1 July 2010 (UTC)[reply]
Well, there's a hidden assumption there that your "trick" is frame-independent. It's imaginable that there is some as-yet-undiscovered physical process that is not frame-independent, and that defines a preferred frame of reference which we have not yet been able to detect. Then you might be able to use that process to send information at arbitrary speeds in that preferred frame, but unlike in the case of the tachyonic antitelephone, you might not be able to "close the loop" to send information back to where it started from (which would allow the grandfather paradox).
In this scenario (for which, certainly, I am not claiming there is any evidence at all; this is a thought experiment), there would still be some observer, in motion relative to the preferred frame, who could observe an effect occurring before its cause in his time coordinate. But so what? That's just a coordinate. As far as I can see, there is no danger of paradox unless you can create a causal loop.
Another possible way out is, keep frame invariance, but throw out isotropy — the magic FTL drive works only when you're headed towards cosmic right; the usual speed limit applies to cosmic left. --Trovatore (talk) 03:44, 2 July 2010 (UTC)[reply]
The relativistic formula for kinetic energy is:
.
Formally, setting v to infinity gives you a negative kinetic energy of -mc2 ! But this is meaningless, because Ek tends to infinity as v approaches c, so you can't go through the "light barrier". Gandalf61 (talk) 15:30, 1 July 2010 (UTC)[reply]
so we can create a perpetual motion machine! All we need to do is get something to infinite velocity, and it will generate tons of free energy! Or maybe we need to use negative energy to get something going that fast. Googlemeister (talk) 15:43, 1 July 2010 (UTC) [reply]
No - none of those things work. The answer isn't infinity - it's the square root of some negative quantity. If (v > c) then (v/c)2 is greater than one - so 1-(v/c)2 is negative. If you take the square root of a negative number on your pocket calculator, it probably says "E" or "Error". You get a complex number - and those can't exist as actual quantities in the physical world - whenever you find a complex number as the final answer to a calculation, you know you've screwed up somewhere. That's WHY this equation says you can't travel faster than light. If the answer was merely infinity, we could possibly imagine some kind of meaning to the answer...but a complex number means "your math is broken somewhere" - and the reason it's broken is because you put a disallowed number into the equation. SteveBaker (talk) 18:27, 1 July 2010 (UTC)[reply]
I understood the answer would be meaningless I was jsut curious about what it would be.--92.251.129.172 (talk) 18:41, 1 July 2010 (UTC)[reply]
As v approaches infinity, the imaginary part decreases without limit. Thus, the energy would be real if it's moving at infinite velocity. Of course, from any other point of reference, it's going at a finite, but faster than light, velocity, and it comes out imaginary, so it doesn't really help.

You can question why you would even want to move your rest mass. Arguably "you" are an algorithm that your brain is computing and that can be sent using photons from one machine to another machine at lightspeed. So, travel at lightspeed is possible. Count Iblis (talk) 15:47, 1 July 2010 (UTC)[reply]

Well what they mean is matter cannot move at lightspeed. Although you could claim that since we are moving away from a certain galaxy at 0.6 c, and that same galaxy is moving the other way at 0.6 c, we are moving apart faster than the speed of light. ALthough that still isn't going the speed of light.-92.251.129.172 (talk) 15:53, 1 July 2010 (UTC)[reply]
You could claim that, but you'd simply be wrong, because relativistic velocities (i.e. ones that are a substantial fraction of the speed of light) don't obey simple arithmetic where 0.6 + 0.6 = 1.2; they obey more complex arithmetic where the answer can never be greater than 1.0: this seems counter-intuitive to us because our everyday logic is based on everyday experience which never includes relativistic velocities, but it has been extensively proved by observations and experiments. 87.81.230.195 (talk) 18:09, 1 July 2010 (UTC)[reply]
As I understand it, though, there is a sense in which a galaxy beyond the event horizon can be said to be moving away from us faster than the speed of light. It gets tricky to state exactly what this means. That general sort of question belongs to general rather than special relativity, and the math is hairy, and there's no longer always as clear a choice of coordinate systems as you might expect. --Trovatore (talk) 01:41, 2 July 2010 (UTC)[reply]
I think you mean a galaxy outside of our light cone. The event horizon is something different entirely. --Jayron32 01:53, 2 July 2010 (UTC)[reply]
What I mean is outside the observable universe in a certain sense. I have never entirely gotten straight what is the accepted terminology on this, but our observable universe article appears to be talking about all objects that could potentially have been affected by an event in the past, by which we could also have been affected. I mean the time-reversed notion — all objects that could potentially influence a future event that we could also influence. The language at particle horizon suggests that event horizon is used in this sense. --Trovatore (talk) 01:57, 2 July 2010 (UTC)[reply]


There is a connection with light cones, but no, "outside our light cone" is not the point exactly. The point is that our forward-facing light cone, and the forward-facing light cone of the galaxy in question, have empty intersection. --Trovatore (talk) 01:59, 2 July 2010 (UTC)[reply]
They're not moving, or at least not that fast. It's just that the intervening space is expanding. — DanielLC 07:42, 2 July 2010 (UTC)[reply]
Well, I was careful to say "there is a sense" in which it's moving away from us faster than light. --Trovatore (talk) 07:49, 2 July 2010 (UTC)[reply]

question about new planet

Time is talking about a new planet that someone got a photo of that is 500ly away. This planet is described as orbiting it's star at 330 AU, and has a surface temp of 2700K. To me this sounds more like a red dwarf star then a planet. Could someone familiar with the methodolgy give me a rough idea of how they can differentiate between a large planet and a very small star in orbit around another star (like Proxima Centauri)? Googlemeister (talk) 15:41, 1 July 2010 (UTC)[reply]

Make life easy for us and give us a link to the article, please!
This other article (from National Geographic) has a bit more detail about the planet. Its mass has been calculated at roughly eight times the mass of Jupiter. That's certainly big, but it's not 'star' big. Our own article on stars discusses the minimum stellar mass, which is something like eighty times the mass of Jupiter. The boundary is governed by the minimum mass required to sustain stable nuclear fusion in the stellar core; the gravitational attraction holding the star together has to be high enough to balance the pressure generated by core hydrogen fusion. There is a gray area as you go to objects with masses lower than that, but bigger than gas giants: the brown dwarfs. The intro to that article discusses the definitions (and the challenges associated with assigning objects to one category or the other). In general, the boundary between large gas giants and small brown dwarfs is taken to be at around 13 Jupiter masses.
Finally, the high surface temperature of the planet in the story is a bit of an anomaly. It's not being heated (significantly) by internal fusion, nor is its heat drawn from the parent star. Instead, this is a very young planet (just a few million years), and its very high temperature comes from gravitational collapse. (The gravitational potential energy freed up as matter fell in to form the planet appears as heat.) The young Earth went through a similar period as it was forming billions of years ago, it took millions of years before the crust cooled and solidified. TenOfAllTrades(talk) 16:00, 1 July 2010 (UTC)[reply]
I do kind of have to agree, though, that this is not what I think of as a referent for the word planet. Really I think the IAU got the definition wrong in a number of ways — rather than distinguish between "planets" and "dwarf planets" on the basis of this fairly silly clearing the neighborhood concept, it would have been more revelatory to draw a line between the "real" planets (that is, the rocky planets), and the gas giants, which are just another sort of cat entirely. --Trovatore (talk) 02:09, 2 July 2010 (UTC)[reply]
Trovatore and his pet peeve. Dauto (talk) 04:07, 2 July 2010 (UTC)[reply]
1RXS J160929.1-210524, btw. --Sean 16:20, 1 July 2010 (UTC)[reply]

Trench binocular

The german de:Scherenfernrohr has no interwikis - but I think this instrument is well known. See the picture on the right. Can you help? --Eingangskontrolle (talk) 16:56, 1 July 2010 (UTC)[reply]

If you are asking what the instrument is, it is a periscope.--Shantavira|feed me 17:08, 1 July 2010 (UTC)[reply]
Periscope seems closest, or else binoculars. The first talks about use in trench warfare but does not mention the design that has separate periscope for each eye of the binocular. The second, in binoculars#Military, specifically mentions this design and application, but is only a small part of a larger and wider-ranging article mostly about binoculars not the key periscope idea. DMacks (talk) 17:11, 1 July 2010 (UTC)[reply]
...and it's not even mentioned in trench warfare. DMacks (talk) 17:17, 1 July 2010 (UTC)[reply]
There doesn't seem to be a specific article in the EN Wikipedia. They're mentioned in passing, as "trench binoculars", in the binoculars article. Searching Google, there doesn't seem to be a general English language term for them, beyond "trench binoculars" or "rabbit ear scope". -- Finlay McWalterTalk 17:13, 1 July 2010 (UTC)[reply]
Can someone who can read German more fluently than I check if de:Scherenfernrohr and its cited ref seem notable enough to make a Trench binoculars article? Although File:British trench periscope Cape Helles 1915.jpg looks like a simple (monocular) design not binoculars...hard to see clearly, so maybe better to do trench periscope instead of specifically binocular. DMacks (talk) 17:15, 1 July 2010 (UTC)[reply]
The link is available in english http://home.arcor.de/thuernagel/sf14-e.htm same person has a whole set of articles on "military optics" (or 'tactical optics') III Taktische Optiken http://home.arcor.de/thuernagel/katalog.htm . There's a mixture of reliable info and some small amount personal reflection/speculation in their (which is all probably completely right) website.
Also known as "donkey ears" in the UK.
Maybe a redirect to a new extended section in periscope? or a complete new article on 'tactical optics'?
I can't see enough there as it is.. though the article would probably survive on it's own as a stub on the assumption that there is more info out there, and the obvious fact that such items are of social-historical significance.87.102.17.114 (talk) 18:03, 1 July 2010 (UTC)[reply]
It's my understanding that they're a bit more than just binoculars with a periscopic lightpath. I believe (but can't find a reliable source to adequately support, otherwise I'd have started on the article) that the rabbit-eared variety (like this) are intended for field artillery spotters. My understanding is that the two lenses independently targettable; the spotter picks his target and aligns both lenses on it - he can then read off the angle from a little gauge, look up the corresponding distance in a little trig table, and report that to the gunners. It's obviously not as accurate as a proper survey with two theodolites separated by a measured baseline, but it's going to be better (and quicker) than just guessing and having to walk trial shots up and down. -- Finlay McWalterTalk 17:41, 1 July 2010 (UTC)[reply]
I think it's just for observers - the wide position gives better stereoscopic view (ie less foreshortening) (Is this correct?)
You're thinking it's a rangefinder - the person says Scherenfernrohre are the predecessors of the stereoscopical rangefinders - the same site has info on these - they can look very similar - but are more meaty - eg http://home.arcor.de/thuernagel/em61.htm .. they could use the thing as a rough rangefinder though,as the article says.87.102.17.114 (talk) 18:03, 1 July 2010 (UTC)[reply]
Some info with pictures through in english here http://www.paulstiger1.co.uk/WWII-Optics-Collection.htm and here http://www.fieldgear.org/optics.htm 87.102.17.114 (talk) 18:16, 1 July 2010 (UTC)[reply]
The internet eh http://www37.atwiki.jp/strike_witches/m/plugin/ref/?guid=on&serial=921&w=500 ... 87.102.17.114 (talk) 18:24, 1 July 2010 (UTC)[reply]
If anyone is interested in writing the article on these particular type of thing then de:Artillerietruppe_(Wehrmacht) is useful - it says that they were used by forward observers for artillery.
Also searching "scissors telescope" shows that they were also used attached to tanks (with reliable sources, not forums).87.102.17.114 (talk) 19:04, 1 July 2010 (UTC)[reply]
It's a http://en.wiktionary.org/wiki/telestereoscope too.87.102.17.114 (talk) 19:04, 1 July 2010 (UTC)[reply]

Do the positions of stars change much relative to each other?

Will Alpha Centauri always be "about" the same distance from Earth? Suppose one picked any two stars at opposite sides of the galaxy. Does their position change much relative to each other? —Preceding unsigned comment added by 92.251.129.172 (talk) 18:28, 1 July 2010 (UTC)[reply]

The Wikipedia articles for many stars, such as Alpha Centauri gives their proper motion in the infobox at the right. -- Finlay McWalterTalk 18:32, 1 July 2010 (UTC)[reply]
So the sky will be very different in a million years time?--92.251.129.172 (talk) 18:34, 1 July 2010 (UTC)[reply]
Yes; this page has movies of the Big Dipper over 200,000 years, and it changes substantially in that time. -- Coneslayer (talk) 18:49, 1 July 2010 (UTC)[reply]
See radial velocity for a sense of how the distances of stars change over time. Stars have even swung very close to the solar system in the past, and will likely do so again in the future—see Gliese 710 (negative radial velocity) for example. ~AH1(TCU) 17:42, 2 July 2010 (UTC)[reply]

Tori Vienneau

was the Tori Vienneau murder case a suicide  ?


http://www.youtube.com/watch?v=LM6bBVd0f7s —Preceding unsigned comment added by Alexsmith44 (talkcontribs) 18:30, 1 July 2010 (UTC)[reply]

Why are you asking us here? We're scientists, not Columbo. 87.102.17.114 (talk) 18:35, 1 July 2010 (UTC)[reply]


law —Preceding unsigned comment added by Alexsmith44 (talkcontribs) 21:08, 1 July 2010 (UTC)[reply]

World building.

I'm sure questions like this have been asked many many times here, but I think mine is a little different.

Let's say I'm making a fictional planet and I want to make it as scientifically realistic as possible. I think I can handle most of the biological and geological topics, but mostly what I wanted to know about was meteorological and climatological. Things like ocean currents, jet streams, air masses and weather patterns, etc. What would the reference desk suggest as guidelines for creating the weather patterns on a fictional planet?

Also, I think I can handle most of the geology, but what confuses me most is plate tectonics. I can grasp the basic concepts of rifting and subduction and so on, but I don't think I know enough to create a fictional tectonic system on a fictional planet. This is important because drawing the maps of my world requires being consistent with tectonics.

63.245.168.34 (talk) 19:42, 1 July 2010 (UTC)[reply]

Make it pretty similar to Earth and you should be fine. If you stray from that, then really it's anybody's guess. We've only studied one planet in any great detail, so we don't know what else is possible. --Tango (talk) 19:52, 1 July 2010 (UTC)[reply]
Are you sure about that? We know that weather patterns are defined by some pretty basic physical laws like the Coriolis Effect and so on, and we're familiar enough with these rules and how they effect the atmosphere and the oceans that we can predict the weather with reasonable certainty. Also, don't you think that making it too similar to Earth would be very unlikely? 63.245.168.34 (talk) 20:05, 1 July 2010 (UTC)[reply]
Not necessarily, perhaps scientifically, a planet can only be inhabitable if it is similar to earth. I mean how likely is it for life to develop on a planet with almost 0 tectonic activity? Of course if the book is not about humanss (or similar to human) colonizing a planet, but about a chlorine breathing insect race, then of course all bets are off. Googlemeister (talk) 20:57, 1 July 2010 (UTC)[reply]
There is a whole book on this, created just for you: World-Building: A writer's guide to constructing star systems and life-supporting planets. There are other books in the series with titles like Aliens and Alien Societies. (I haven't read World-Building but have read Aliens. The latter covers a lot of bases and made me consider aspects I hadn't thought about.) Comet Tuttle (talk) 21:16, 1 July 2010 (UTC)[reply]
This is actually very difficult to deal with in a comprehensive way -- just consider how much the Earth's climate has varied over the past billion years, mainly as a consequence of the continents moving around. It is widely believed, for example, that the Earth was in a snowball state from 800-600 million years ago, with the oceans frozen over nearly down to the equator, due to all of the continental landmass being concentrated near the equator. In short, you can get anything from rainforests at the poles to glaciers at sea level at the equator, just by shifting the land around. Looie496 (talk) 22:41, 1 July 2010 (UTC)[reply]
That's mostly what I'm curious about. How does the shape of the land affect the climate?

Also, as for the planet being inhabited or inhabitable, in the context of my fictional universe, it's a terraformed planet with a young, engineered biosphere. Only inhabited by species that were brought to it by humans. There are no aliens in my fiction.

63.245.168.34 (talk) 00:12, 2 July 2010 (UTC)[reply]

Land has profound effect on climate. Generally speaking, the direction of prevailing winds over landmasses determines things like where deserts and forests will appear. If prevailing winds have a long Fetch over water, then the land downwind tends to be much rainier than if the winds have a long fetch over land. Also, there are effects like rain shadow, which makes land upwind of a mountain range rainier than land downwind of the mountain range; which is why Seattle is much rainier than say Spokane. --Jayron32 01:06, 2 July 2010 (UTC)[reply]
Also, land tends to heat up (and cool down) far quicker than water: you're better off swimming in the Mediterranean in October than in April, even if the land temperatures are similar. Physchim62 (talk) 01:58, 2 July 2010 (UTC)[reply]
From observations on exoplanets, astronomers can deduce a thermal "cimate" map of some planets, as they are hotter in some areas and cooler in others. However land and ocean have not yet been confirmed on any exoplanets, not even Super-Earths, so the hypothetical planet should be like Earth in the sense that it has mountains and oceans. As for the effects of land patterns on the overall climate of regions, here is a question I posted here way back, when I was proposing a way to model the climate that is probably much less complicated than supercomputer simulations but do not involve mathematical formulae and calculations. As I never got an answer, I'm not sure how accurate my method is, but it should be a good general approximation of the climate of a planet given its land masses, but its functioning is reduced when less is known about the planet. Regular weather patterns can be inferred from the location of quasi-stationary highs and lows, but there will always be great variations. We know the general climate of Mars, yet we still don't have the ability to predict global dust storms that often occur on the Red Planet or what exactly triggers them. Also, if the climate changes on your planet, the effects of that change are likely going to be harder to predict than just the climate map in a static climate. ~AH1(TCU) 17:24, 2 July 2010 (UTC)[reply]

Micro rockets

Suppose we want to send a miniscule payload from the surface of the Moon to the Space station. Then it seems to me that at least in theory, the rocket could be made very small. So, what is the minimum mass of a rocket that is able to let a payload of mass m (assume m < 1 microgram) escape from the Moon's gravity, as a function of m? Count Iblis (talk) 22:56, 1 July 2010 (UTC)[reply]

What kind of fuel do you plan to use? By mass, do you include everything involved or just the rocket (projectile) that goes up into the air? I ask because different fuels have different masses. Also, assisted blastoff reduces fuel required. Further, the length of thrust is important. Do you want an explosion or a controlled thrust? -- kainaw 23:11, 1 July 2010 (UTC)[reply]
For a payload that small, the payload is irrelevant. What matters is how light an engine you can make. The smallest rocket able to lift a payload of a microgram will be essentially the same size as one able to lift a payload of a few hundred grams. That said, I can't think of anything with a mass of less than a microgram that I would want to send from the Moon to a space station, certainly not on its own - just put it in with something else that is going. --Tango (talk) 23:15, 1 July 2010 (UTC)[reply]
I see, so it is not possible to make microscopic engines? Count Iblis (talk) 00:59, 2 July 2010 (UTC)[reply]
I wouldn't say it's impossible, but I don't know of anyone ever doing so. There just isn't any reason to try. --Tango (talk) 01:19, 2 July 2010 (UTC)[reply]
Just fire it from a rifle? (I don't know how the space station would catch the bullet!) Dbfirs 07:13, 2 July 2010 (UTC)[reply]
See Space gun. It's a lot harder to get things into space than people on here seem to realize. --Mr.98 (talk) 11:37, 2 July 2010 (UTC)[reply]
This question is about launching from the Moon into the Moon's orbit. Gravity is 5/6 that on Earth and orbital distance is about 1/4 that of Earth. So, there are two things making it much easier. Gravity is much less. Orbit is much closer. Further, there is almost no atmospheric drag. So, I do not see why shooting a small projectile into orbit would be a big deal. -- kainaw 12:24, 2 July 2010 (UTC)[reply]
I've read that there's a limit to how small you can make a rocket. Unfortunately, they didn't actually say what the limit was. That said, getting it off the moon is much less difficult then getting it off the Earth. — DanielLC 07:32, 2 July 2010 (UTC)[reply]
Bottle rockets are pretty small... Googlemeister (talk) 18:17, 2 July 2010 (UTC)[reply]

Why does the ocean appear so high?

I took this picture of the Atlantic Ocean from an elevated train.[11] Am I unfamiliar with nature or does the ocean appear elevated especially towards the horizon? 67.243.7.245 (talk) 23:31, 1 July 2010 (UTC)[reply]

We do not know how the camera was oriented. The technical term for this parameter is the elevation of the camera - the angle it makes with respect to the horizon - and not to be confused with its elevation above sea-level. Obviously, by angling a camera, you can make the horizon appear to be at any vertical position inside the image. In addition, there may be an optical illusion of distorted perspective if the train's window has a weird aspect ratio or orientation, or if the train was actually tilted when you photographed the scene. The train was high above the ground, (the other kind of elevation), then you may see a lot more land in the foreground than usual, contributing to the unusual perspective. Nimur (talk) 23:51, 1 July 2010 (UTC)[reply]
Well, that's all valid, but also it looks like the foreground may be sloping downward a bit, which is a frequent cause of illusions of this sort. Looie496 (talk) 01:22, 2 July 2010 (UTC)[reply]
Actually, we do know how the camera was oriented -- if you're looking toward a horizon where the sea seems to form a sharp boundary against the sky (rather than fading into haze), a line from you to that horizon must be horizontal. (Horizon. That's why we call it "horizontal".) But as Looie says, the foreground is sloping, so this may contribute to the shoreline looking lower than you expect. --Anonymous, 04:14 UTC, July 2, 2010.
I think the reason is rather perception (or the lack thereof) of depth (in the sense of distance). On the structured foreground one has a pretty good feel for how the distance increases as one looks from the bottom of the picture to the land-sea boundary. The unstructured sea does not offer any depth indicator (increasing haziness doesn't help on this fairly clear day) so one does not perceive any distance difference between the land-sea boundary and the horizon. That's why the sea appears like a vertical wall, the entire blue surface at the same distance. I hope I understood the question correctly. --Wrongfilter (talk) 15:25, 2 July 2010 (UTC)[reply]
I've noticed the same effect while standing in an airport close to sea level—the ocean in the distance appeared somewhat distant as well as a bit elevated. Other than perspective could some kind of mirage effect explain the phenomenon? ~AH1(TCU) 17:10, 2 July 2010 (UTC)[reply]

July 2

Phantom question (nothing to do with the paranormal)

Just a couple of days ago I've read a funny story on the Internet about an F-4 Phantom taking off from a combination civilian/military airfield. Supposedly, the Phantom pilot was in a hurry to take off but the tower told him to wait because of heavy civilian traffic; then, after some back-and-forth between the tower and the pilot, the controller said, "OK, if you can reach 14,000 feet within half the runway length, then you are cleared for takeoff; otherwise, continue to hold." Allegedly, what happened then was that the Phantom taxied into position at the approach end of the runway, engaged full afterburner, lifted off within half the runway length, and then climbed vertically to 14,000 feet, thus abiding by the tower's conditional clearance. (Here's the link to the site in question: http://www.businessballs.com/airtrafficcontrollersfunnyquotes.htm ) My question is, does the Phantom have a sufficient thrust/weight ratio to do that? I've never flown the Phantom, so I can't be sure that this is possible. Thanks in advance! 67.170.215.166 (talk) 01:48, 2 July 2010 (UTC)[reply]

Based on the numbers in our article, the plane could climb vertically if it was empty, but the thrust/weight ratio was only 0.86 in a fully loaded configuration. Looie496 (talk) 03:23, 2 July 2010 (UTC)[reply]
So in other words, the plane must've had no bombs, no missiles, and only a light load of fuel on board. (Which means that the pilot (1) wasn't flying far, and (2) must've been a real hotshot to climb in afterburner with such a light fuel load.) Thanks for the info, and clear skies to you! 67.170.215.166 (talk) 03:49, 2 July 2010 (UTC)[reply]
Although not, I think, literally possible, such a feat would be much closer to attainable by an English Electric Lightning. 87.81.230.195 (talk) 05:56, 2 July 2010 (UTC)[reply]
Chatting back and forth with ATC, especially during heavy traffic would be seriously unprofessional and violate FAA regs. I hope this story is not based on fact. Googlemeister (talk) 18:16, 2 July 2010 (UTC)[reply]

Wormholes

Does a wormhole exert its own gravitational pull assuming they exist? Based off diagrams I have seen of wormholes they seem to have a gravitational well similar to that of a black hole, so wouldn't they have an event horizon around each of the mouths and trap any traveller going through them inside? Or does the negative energy density required inorder to keep a wormhole open cancel out the gravitational fields. I know that this is assuming that traversable wormholes exist and they might not, but thanks for the help anyway. --74.67.89.61 (talk) 01:56, 2 July 2010 (UTC)[reply]

Wormholes do not exist. They are hypothetical conceptualizations. Even within the realm of the hypothetical, they introduce irresolvable contradictions. Wormholes are an interesting, mind-bending game for geometrically inclined mathematicians. But we have never seen them; we have never seen any evidence of them; and our best efforts to consistently explain them would require bizarre and unrealistic physics. It would serve the world a lot better if the pop-science physics books would focus on actual, existing strange physics - like the solution to the generalized double pendulum. Have you ever seen one of these crazy contraptions? I can't comprehend the popular fascination with fictional physics, when there is so much unexplained in actual physics. Nimur (talk) 02:22, 2 July 2010 (UTC)[reply]
There is no currently known property that forbids the existence of wormholes (and lets be clear here, I am not talking about traversable wormholes, I am talking about any type of wormhole, stable or not, microscopic or macroscopic) although there are a lot of strange implications. We currently do not know enough to say whether they exist or not for certain. When black holes were found in general relativity, many scientists believe they were impossible even though there was no principle that made them impossible, and look where we are today, finding that black holes are actually common throughout the universe. Wormholes are in a similar state today of what black holes used to be in, being allowed by general relativity with nothing preventing them, but no evidence for them and strange impications. I am not saying that they exist or don't, I am simply saying we do not know enough to give a definite answer, which is why I said ASSUMING THAT THEY EXIST do they exert a gravitational pull, i am simply curious as to whether someone knows the answer to this. —Preceding unsigned comment added by 74.67.89.61 (talk) 13:59, 2 July 2010 (UTC)[reply]
The problem is, we can't just assume they exist. We have to make some assumptions about how they exist - we have to change something in the laws of physics, and there are all kinds of ways we could do that (some more plausible than others). Depending on how we change physics, we'll get different answers to your question. If we talk about a specific theory of wormholes, for example the wormhole described by Matt Visser which is mentioned in Wormhole#Traversable wormholes, then the question is answerable. Unfortunately, I don't know the answer... I can't find the paper mentioned. --Tango (talk) 02:28, 2 July 2010 (UTC)[reply]
Forgive me for soap-boxing, but somebody needs to just come out and say it. Wormholes do not exist. Before anybody tries to contemplate the gravitational behavior of a hypothetical wormhole, they should be required to thoroughly, quantitatively, and correctly describe the behavior of this simple contraption, which is only under the influence of regular, earth gravity. Once you have mastered the mathematical techniques necessary for the generalized description of a system in ordinary geometries, you will have the foundations for the mathematical tools to play with general solutions in arbitrary geometries. Unless you have these techniques completely mastered, any description of wormholes is just gonna be a lot of handwaving and nothing more. That's why it is so frustrating to see so much meaningless and frivolous writing on the subject of wormholes, hidden behind the excuse of advanced science/magic and totally devoid of actual meaning. Nimur (talk) 02:34, 2 July 2010 (UTC)[reply]
In order to actually try to be encouraging, rather than discouraging, here is a list of topics you will need to learn thoroughly in order to understand geometries and topologies for wormholes:
This list is not complete, but by the time you master those concepts, you'll already know what else you need to learn in order to study wormholes. Spend a lot of time on the mathematical foundations. These subjects are very difficult, but they are not inaccessible. They are requisite - these are the languages which are best suited to discussions of the geometry of space. "Simple english" just doesn't have the precision and unambiguity necessary to properly describe these sorts of systems. Nimur (talk) 02:45, 2 July 2010 (UTC)[reply]
I hold an MMath degree in which I studied (and passed) modules in all the fields you mention. I am qualified to discuss the subject of wormholes. We do not know if wormholes can exist or not, but from a mathematical point of view (which is the point of view I take, since I am a mathematician by training) the changes to our physical theories are minimal (basically, you just have to forget the weak energy condition). Just because we cannot demonstrate that the weak energy condition can be violated does not mean that we cannot investigate the consequences of such a violation. --Tango (talk) 03:07, 2 July 2010 (UTC)[reply]
I have found the paper! It is here. It describes a solution in which a traveller would feel no forces on travelling through. In the words of the paper, if you send a beam of light into the wormhole "the throat of the wormhole acts as a “perfect mirror”, except that the “reflected” light is shunted into the other universe." (The paper describes wormholes between two universes, but the idea should work for wormholes within one universe too - at least, I can't see any problems with it.) Away from the throat of the wormhole, the universes are just flat Minkowski space, so there would be no gravitational attraction (or repulsion, for that matter). --Tango (talk) 03:07, 2 July 2010 (UTC)[reply]
And that paper links to Wormholes in spacetime and their use for interstellar travel: A tool for teaching general relativity (1988). (PDF available). That paper lays out some conditions for wormholes: "It turns out that there are very simple, exact solutions of the Einstein field equations, which describe wormholes that have none of the above problems. ... The traversible wormhole solutions to Einstein's field equations are so simple that they can be used as a tool for teaching beginning relativity students how to solve the Einstein equations, how to interpret physically solutions they have obtained, and how to explore the properties of solutions." This is what the OP was really asking, I think. Thanks Tango for the reference above. Nimur (talk) 14:37, 2 July 2010 (UTC)[reply]
I see no reason to believe the OP was interested in the pedagogical properties of wormholes. Those properties are very important (my first rigorous exposure to wormholes was purely as a convenient example of a non-trivial solution to the Einstein equations, although that was the non-traversable Schwarschild wormhole, so not much use to the OP), but I don't think the OP cares about them. --Tango (talk) 15:11, 2 July 2010 (UTC)[reply]
To repeat what I've said in the past about these traversable wormhole "solutions": a solution of general relativity is a solution of the Einstein field equation Gμν/8πG = Tμν, and any spacetime geometry whatsoever "solves" that equation if you don't constrain Tμν. All you have to do is work out Gμν/8πG for your geometry and then declare Tμν to be equal to it by fiat. Thus to say that a traversable wormhole geometry is a "solution" of general relativity is to say nothing at all, unless you have some independent constraints on Tμν (i.e., some non-gravitational laws of physics). Even the simplest physical constraints on Tμν rule out all of these wormhole solutions. They are therefore not solutions in any meaningful sense. The authors of these papers do acknowledge this, but what they don't mention is how easy it is to write down wormhole and warp-drive and time-travel geometries with whatever crazy properties you want, in the absence of any physical constraints. There's nothing interesting or inventive about the particular geometries they wrote down. -- BenRG (talk) 22:51, 2 July 2010 (UTC)[reply]
I would say a "solution" of the Einstein field equation is an explicit description of both the spacetime geometry and the matter/energy distribution that produces it. Just saying that the stress-momentum-energy tensor equals the appropriate value doesn't really count. --Tango (talk) 23:07, 2 July 2010 (UTC)[reply]
But it has been pointed out that these solutions require "exotic matter" and other un-physical assumptions, in some distribution, to produce the wormhole geometry. The paper Tango linked at least did not require exotic matter to be present in the throat of the wormhole (hence the arbitrary label of "traversable"), but exotic/unphysical matter was required elsewhere in some distribution to produce the system and its geometry. Such exotic matter does not exist. (Rather, we have never observed it, nor do we have any reason to believe it exists, but it would be really neat if it existed and it is very hard to prove non-existence, so lots of people latch on to the idea). Nimur (talk) 00:05, 3 July 2010 (UTC)[reply]
Actually, the solution did involve exotic matter at the throat, just not covering the whole throat so a traveller could avoid it. Also, "traversable" usually refers to the wormhole existing long enough to travel through rather than to whether there is exotic matter in the way. While there is far from any conclusive evidence that exotic matter can exist, the Casimir effect does point in the right direction. --Tango (talk) 00:49, 3 July 2010 (UTC)[reply]

MP3 player (well, really amplifier) power consumption

I recently got a new iPod Nano (5G). It seems to have somewhat lower volume than my old (2G) one. It's fine for music, but very many spoken word programs (such as the excellent Open Yale Courses) are recorded with a much lower volume than music. As a consequence, I often listen to them at 100% volume, and even then some are quite quite for outdoor listening. I'm wondering what that does for battery life. Does the power consumption depend only or primarily on the volume level (on the principle that the signal is maximally amplified, even if there is not much of a signal), or does it depend primarily on the amount of actual sound energy coming out of the headphones? The first seems intuitive - if I crank up the volume, I use more energy. Physically, the second is more plausible - after all, where would the energy go otherwise? Does a amplifier amplifying a flat line get hot? If the second is true, how much of an effect is it? Will my mp3 player have a significantly longer battery life playing e.g. the Pastorale than playing Painkiller (and yes, I have both on my player ;-). --Stephan Schulz (talk) 09:30, 2 July 2010 (UTC)[reply]

As a general rule, the power consumption of a noramlly designed amplifier is approximately proportional to the volume it produces, rather than the setting of the volume control. --Phil Holmes (talk) 13:11, 2 July 2010 (UTC)[reply]
Thanks! So I should always use an endless loop of 4′33″ for battery benchmarking ;-). --Stephan Schulz (talk) 13:19, 2 July 2010 (UTC)[reply]
iPod Nano uses a Class D amplifier, and great pains are taken to reduce its quiescent current. (That means that "0 micro-amps" would be spent on audio power amplification if you played 4'33" through it - but of course, there are parasitics and losses, but generally very small). Realistically, you will benefit more in terms of battery life by turning the LCD screen / backlight off - this draws significantly more current than the audio system. Unfortunately the iPod nano internals are proprietary and their amplifier and audio system is integrated into the "anonymously marked" IC packages with Apple part-numbers only; it is impossible to get the exact specs. Nimur (talk) 15:04, 2 July 2010 (UTC)[reply]
Just as an aside, I believe you can set individual mp3 files to play with different volumes in iTunes — File > Get Info > Options > Volume Adjustment. You could boost the predictably quiet files by 100% before putting them on the iPod. (I'm reasonably sure those settings are transferrable to the iPod, but I haven't tried it.) --Mr.98 (talk) 19:11, 2 July 2010 (UTC)[reply]

is there such a thing as a "semi-permanent" or reversible tattoo?

so I really like the idea of getting a tattoo, but who knows if later I need to sell out to the man as most people in corporations do after a while. If I ever need to have Forbes and Fortune's cocks ramming down my throat, or, who knows, maybe even Wall Street's directly, they will not be moved by tattoos on me. So, my question is whether there exists a tattoo that is "permanent" in the sense that it doesn't "just" go away, it is a real tattoo and looks like one, but semipermanent or reversible in the sense that I can get rid of it for a fee later? For example, I can imagine that a certain class of dyes have a specific "antidote", a chemical that will cause them to break up and disappear into the body, despite the fact that they do not by themselves. Is there such a thing? Or any other way to get a semipermanent or reversible tattoo? Thank you. 92.230.232.169 (talk) 09:41, 2 July 2010 (UTC)[reply]

As far as I know, best-practice modern tattoos will respond well to laser treatment (laser light tuned to the dye is used to destroy the colour molecules, your immune systems takes care of the rest). There is a certain risk, though, both that the process will be incomplete, but also of allergic reactions. And it's expensive... --Stephan Schulz (talk) 09:46, 2 July 2010 (UTC)[reply]
so why dont they develop special dyes that respond to, say, a certain frequency of microwave radiation, or a certain "antidote" chemical, or anything else... —Preceding unsigned comment added by 92.229.13.177 (talk) 10:40, 2 July 2010 (UTC)[reply]
Our article Temporary tattoo#Temporary variants of permanent tattoos mentions InfinitInk which is supposed to be easier to remove with laser treatment. While are I'm sure inks which are easier to remove which could be developed, I'm not convinced your first idea would be much easier or cheaper then with lasers, particularly if you don't want the rough area you were tattoed to be cooked and also don't wont your tattoo to fade a lot over time due to natural exposure to EM radiation from the many forms of transmission common nowadays. Your second idea is more promosing but developing something safe and effective is probably not easier or cheap particularly since the antidote will need to penetrate the skin Nil Einne (talk) 14:16, 2 July 2010 (UTC)[reply]
There's also the far-off possibility of nano-tattoos, which could be reprogrammed as you saw fit. (Kind of goes against the point of a tattoo, if you ask me. They're not just an image, they're an image of commitment!) IMO, if you are worried about not looking corporate enough, get it in an area that can be easily hidden, like the shoulder or upper arm, which can be displayed if and when you want it pretty easily, but is also easily hidden under your suit. And please watch your language on here—it's rude to swear for no purpose, and makes you sound neither edgy nor interesting. --Mr.98 (talk) 14:33, 2 July 2010 (UTC)[reply]
I acknowledge your sentiment, though I wonder if it isn't prompted by your regularly ingesting more corporate c**k than is your preference. see? self-censored.92.224.207.197 (talk) 17:49, 2 July 2010 (UTC)[reply]
Henna? Woad? 92.28.247.183 (talk) 21:17, 2 July 2010 (UTC)[reply]

I reduced copper(II) sulfate (blue) with ascorbic acid solution (colorless) to get a yellow solution. Is it copper(I) sulfate? It reacted with hydrogen peroxide to form a green solution again (it was green because of the additional acidity of the ascorbic acid). --Chemicali nterest (talk) 14:16, 2 July 2010 (UTC)[reply]

Sounds more like a copper ascorbate complex to me. Physchim62 (talk) 14:43, 2 July 2010 (UTC)[reply]
I disagree (that example is in anhydrous conditions) - in aqueous solution copper is reduced [12] [13]
Possibly you made nano-copper in the first step [14]. It's also possible that the some of the product at the first step was fine Copper(I) oxide (this paper [15])
Additionally 'Cuprous(I) hydroxide' is yellow and is obtained in alkaline conditions, though this seems very unlikely in acid conditions - see also Benedict's reagent and Fehling's reagent
This paper [16] is available online, and covers a lot of stuff relating to the first reaction. —Preceding unsigned comment added by 77.86.124.131 (talk) 15:56, 2 July 2010 (UTC)[reply]
The second step (green solution) sounds like it might be a copper(II) complex - which are often green with oxy-ligands - I'm not sure what happens to the ascorbic acid through all this - but I guess it will be a complex of whatever the ascorbic acid is oxidised to.Sf5xeplus (talk) 14:55, 2 July 2010 (UTC)[reply]

Well Cu(I) isn't very stable in water. I'm going with the idea of nanocopper -- you reduced Cu(II) to Cu(I) only to have it disproportionate into Cu(II) and Cu(0). (And the Cu(II) gets reduced again into Cu(I)...) And colloidal copper is much different than bulk copper....because of different surface energies and the Mie scattering it will take on a different colour...welcome to nanochemistry! John Riemann Soong (talk) 15:54, 2 July 2010 (UTC)[reply]

Also did you add Vitamin C in excess? Or? John Riemann Soong (talk) 15:57, 2 July 2010 (UTC)[reply]

Neither were in clear excess. When CuCl2 is reacted with ascorbic acid, white CuCl is formed. I was wondering whether there is a similarity in the sulfate reaction. --Chemicalinterest (talk) 20:17, 2 July 2010 (UTC)[reply]
I will try tin(II) chloride as a reducing agent and see if there is a difference. --Chemicalinterest (talk) 20:21, 2 July 2010 (UTC)[reply]
The issue is that Cu(I) sulfate is probably very soluble in water whereas CuCl precipitates in excess and is thus most of the Cu(I) is shielded from disproportionation. Try reducing Cu(II) chloride in the presence of concentrated HCl.... also reduce Cu(II) sulfate with Vitamin C in clear excess, eliminate the chance of any remaining Cu(II).
Btw, do you have any sodium borohydride? John Riemann Soong (talk) 21:04, 2 July 2010 (UTC)[reply]
(I ask because NaBH4 is a little more environmentally friendly than Tin(II) and Tin(IV)...what do you do with your waste?) John Riemann Soong (talk) 21:08, 2 July 2010 (UTC)[reply]
Tin is ok unless you're a mollusc ... In the overall scheme of things using NaBH4 is just as damaging (production of) as a little solder+HCl, but I appreciate your point.77.86.124.131 (talk) 21:25, 2 July 2010 (UTC)[reply]
well the basic products of NaBH4 decomposition can be neutralised with acetic acid or vinegar... then dumped down the sink. You can't do that with Tin(IV). John Riemann Soong (talk) 21:27, 2 July 2010 (UTC)[reply]
Tin in the enviroment (in the quantities here) is unlikely to be a problem .. the main problem from tin is poisonous by products from its production ie smelting —Preceding unsigned comment added by 77.86.124.131 (talk) 21:34, 2 July 2010 (UTC)[reply]
Do you have a centrifuge? If you want to do the waste management yourself, you can recover the colloidal copper by centrifuge. Sodium ions, sulfate ions, phosphate ions and chloride ions don't stress the environment that much. John Riemann Soong (talk) 21:31, 2 July 2010 (UTC)[reply]
I would use tin(II) chloride in quantities in order of .2 grams; that shouldn't hurt the environment. --Chemicalinterest (talk) 00:21, 3 July 2010 (UTC)[reply]
More clues: When reacted with an excess of ascorbic acid, it forms a yellow solution. The yellow solution becomes darker as ammonia is slowly added. The color is similar to a dilute solution of iron(III) chloride in acidic conditions. Wisps of brown precipitate are formed. When much ammonia is added, it creates a very dark brown precipitate that lightens within 1 second to a yellow brown precipitate. The yellow brown precipitate reacts with small amounts of hydrochloric acid to form a colorless precipitate (CuCl), which dissolves in excess HCl to form a clear solution. The precipitate also reacts with hydrogen peroxide to form a greenish solution (Cu(H2O)2+). Hope this helps. --Chemicalinterest (talk) 00:43, 3 July 2010 (UTC)[reply]

Concentration of NaOH

100 mL of 0.125 M NaOH was mixed with 64 mL of H2O. Calculate the final concentration.

Is it incorrect to use the dilution equation in the following way?

(100 mL) (0.125 M) = (64 mL) (C)

Concentration = C = 0.20 M H2O.

--478jjjz (talk) 17:04, 2 July 2010 (UTC)[reply]

Yes that's wrong.
The correct formula is :
  Initial volume
  --------------   x Initial concentration  = Final concentration
   Final volume  
The final volume is 164mL. 77.86.124.131 (talk) 17:16, 2 July 2010 (UTC)[reply]

So, then the final Concentration is

[(100mL)/(164 mL)] * 0.125 M = 0.0762 M

--478jjjz (talk) 17:44, 2 July 2010 (UTC)[reply]

HCl and NaOH

25.0 mL of 0.5 M NaOH was reacted with 45 mL of 0.12 M HCl. Calculate the final concentration of NaOH.

(0.0250 L of NaOH)*(0.5 mol NaOH) = 0.0125 mol NaOH.


(0.045 L) * (0.12 M HCl) = 0.0054 mol HCl


Since NaOH and HCl react in a 1:1 ratio, then HCl is the limiting reagent. The product is 0.0054 mol NaOH.

How do I proceed further?--478jjjz (talk) 18:03, 2 July 2010 (UTC)[reply]

You're starting materials indicate 0.0125 mol NaOH and 0.0054 mol HCl are availible to react. Since there is a 1:1 ratio, answer these questions:
  1. If 0.0054 mol of HCl react, and the SAME NUMBER OF MOLES of NaOH react as well, then how many moles of NaOH actually react?
  2. If you started with 0.0125 mol of NaOH, and you TAKE AWAY the number of moles of NaOH that reacted (#1 above) how many moles of NaOH are left over?
  3. If you mixed two solutions, one of which contained 0.0250 L and the other which contained 0.045 L, what is the TOTAL VOLUME of the two solutions after mixing?
  4. Now, you have some NaOH left (answer to #2) and it is in the combined volume (answer to #3). What is the final concentration, given that the concentration of the NaOH left is the LEFTOVER MOLES OF NaOH DIVIDED BY THE COMBINED LITERS OF SOLUTION.
Every problem you solve that looks very similar to this is solved roughly the same way. --Jayron32 18:14, 2 July 2010 (UTC)[reply]
  1. 0.0054 mol NaOH react
  2. 0.0071 mol NaOH left over
  3. 0.070 L = total volume
  4. (0.0071 mol NaOH)/(0.070 L) = 0.10 M NaOH.--478jjjz (talk) 18:29, 2 July 2010 (UTC)[reply]
Solution concentration equals amount of stuff divided by solution volume--say that like a hundred times or so. That is the formula that always answers any dilution question, because you will always have or can figure out any two of those variables and then use that formula to solve the third. The secret to avoiding mistakes is to always write units with your numbers so you know exactly what the value means (i.e., whether it's concentration, volume, etc.). So when you first say "(0.5 mol NaOH)" BZZT, that's a kind of careless mistake that can lead to all sorts of confusion and more serious understanding mistakes later ("L * mol" is...um...L*mol not mol--that's a pretty fundamental fact). It's not just nit-picky--if you are working through towards an answer and the units are not the right type for the question ("how many moles?" and you have an answer in liters) you immediately know you made a mistake. Say you have figured out "0.0054 mol NaOH", that's an amount of stuff. You can easily figure out the total volume in which it is dissolved (the volume of a mixure of same-solvent solutions is just the sum of the volume of each part). And now you have a formula you've said like a hundred times or so that uses those two to figure out concentration. DMacks (talk) 18:49, 2 July 2010 (UTC)[reply]

Titration

250.0 mL of an unknown HCl solution was titrated with 14.75 mL of 0.0762 M NaOH to the phenolphthalein end point. Calculate the molarity of the HCl solution.--478jjjz (talk) 18:41, 2 July 2010 (UTC)[reply]

The reaction is

HCl + NaOH ==> NaCl + H2O


Total volume= 250.0 mL + 14.75 mL = 264.75 mL = 0.26475 L

Every concentration problem is the same, just different numbers. DMacks (talk) 18:49, 2 July 2010 (UTC)[reply]

We have 0.0012 mol NaOH --478jjjz (talk) 18:47, 2 July 2010 (UTC)[reply]

DMacks, I beg to differ with you. I don't know how many moles of HCl I started with. This is the source of my confusion.--478jjjz (talk) 19:00, 2 July 2010 (UTC)[reply]
If you have the number of moles of NaOH and wish to find the number of moles of HCl, look at the equation. It will tell you the ratio between the two. - Jarry1250 [Humorous? Discuss.] 19:05, 2 July 2010 (UTC)[reply]
0.0012 mol HCl reacted with 0.0012 mol NaOH. I don't know how many moles of HCl I started with; how do I find the moles of leftover HCl?

--478jjjz (talk) 19:11, 2 July 2010 (UTC)[reply]

What does the "phenolphthalein end point" mean? DMacks (talk) 19:26, 2 July 2010 (UTC)[reply]
It means to the point where all the acid (HCl) and base (NaOH) have completely reacted.--478jjjz (talk) 19:30, 2 July 2010 (UTC)[reply]
With none left over? DMacks (talk) 19:32, 2 July 2010 (UTC)[reply]
Since I don't know the starting moles of HCl, I don't know how much of it is left over. I want to know if this problem from one of my chemistry quizzes is even solvable.--478jjjz (talk) 19:34, 2 July 2010 (UTC)[reply]
Maybe the context of my question wasn't clear. Again, for the definition of the phenolphthalein endpoint of a titration, you say the acid and base "have completely reacted". How many moles of base do you need in order that X moles of acid gets "all reacted"? Does the definition here mean you have added just an arbitrary small amount, exactly as much of one as the other, or a huge excess amount? How much un-neutralized acid (or base) is present according to the definition of your endpoint? DMacks (talk) 19:42, 2 July 2010 (UTC)[reply]
  • 0.0012 mol NaOH has reacted with X amount of HCl. I don't know the initial concentration of HCl which I can multiply by the volume to get the # of moles of HCl. If I have, let's say, 0.000005 moles of HCl at the start of the reaction, then HCl would be the limiting reagent & none would be left over.--478jjjz (talk) 19:51, 2 July 2010 (UTC)[reply]
If you have 0.000005 moles of HCl at the start of the reaction, how many moles of NaOH would you add to get that amount of HCl all reacted? DMacks (talk) 19:53, 2 July 2010 (UTC)[reply]

They would both be in a beaker. 0.0012 mol NaOH reacts with with all HCl and (0.0012 -0.000005) mol NaOH is left over in the basic solution in the beaker due to the excess NaOH.--478jjjz (talk) 19:56, 2 July 2010 (UTC)[reply]

There's your problem: the very definition of your endpoint is when the solution reaches a very specific pH (the color changes), which tells you exactly how much H+ is present at that point. The casual wording of the reaction description might be throwing you: if you have >0 of compound X present, it is not "all reacted", it's excess unreacted. All reacted means all of it is reacted, not just "as much as can". If I have 10 H+ and add 5 OH-, my OH- is all reacted, but my H+ is not all reacted. The first drop of base you add to the acid completely reacts, but you (hopefully) recognize that this is not the endpoint of the titration (would be silly because every titration would be "1 drop"). You keep adding base until (for example) neutrality (until the H+ is "all reacted"). DMacks (talk) 20:05, 2 July 2010 (UTC)[reply]
  • The purple words above have been copied verbatim. I have concluded that this problem isn't solvable. I have to speak to the instructor and let him know that it is preposterous to put unsolvable problems on quizzes to make me get a low grade. (If anyone can somehow solve this problem, then please do!)--478jjjz (talk) 20:11, 2 July 2010 (UTC)[reply]
The problem is perfectly solvable. In fact, it is a routine calculation in analytical chemistry! Just just need to figure out the amount of HCl in the initial solution, then divide by the volume of the initial solution to find the concentration. Physchim62 (talk) 20:16, 2 July 2010 (UTC)[reply]
(ec) To be honest, if I were the teacher, I would tell you to go back and relearn what "endpoint" means in the context of a titration experiment. Again, per the technical definition of phenolphthalien endpoint, you know exactly the concentration of unreacted acid in the solution at that point (i.e., excess, compared to the amount of base you added). DMacks (talk) 20:17, 2 July 2010 (UTC)[reply]
This problem was in a lab quiz. The #s in the above purple problem have nothing to do with the experiment that I had performed during the prior class. As it stands, I re-affirm that the purple problem is unsolvable.--478jjjz (talk) 20:22, 2 July 2010 (UTC)[reply]
The first step in pretty much any titration problem is the calculate the amount of reagent you have added from the burette. Now here, you've added 14.75 mL of a 0.0762 M solution: amount is concentration times volume, so you have added n(NaOH) = 0.01475*0.0762 = 0.001124 mol. You know the equation, so you know that that hydroxide has reacted with 0.001124 mol of HCl. So how much HCl is left over? Take a quick look at our article on phenolphthalein, and you will see that it changes colour at pH 8.2… so when the phenolphthalein changes colour, there's no hydrochloric acid left! So your initial solution of HCl contained 0.001124 mol HCl in 250 mL, in other words it was a 0.001124/0.25 = 0.00450 M solution. Physchim62 (talk) 20:41, 2 July 2010 (UTC)[reply]
Let's start again
14.75 mL of 0.0762 M NaOH was used - that's 14.75 x 0.0762 = 1.12395mmol of NaOH (A)
It was titrated to an end point with an acid-base indicator in the reaction NaOH + HCl >>> NaCl + H2O , that's as 1:1 reaction so 1.12395mmol of HCl must have been used (B)
Molar concentration is related to number of moles and volume by the equation:
                      Number of moles
   Concentration =  ------------------  (C)
                         Volume
You've been asked to find the molality (concentration) of the HCl solution, so that's 1.12395mmol ÷ 250ml (D)
Be careful with units, since both the quanities in D are 'milli' in this case the answer is is mol/litre , which is the same of molality, no further work needed.
If you didn't get any part please ask about it. I've labelled each step A,B,C etc .77.86.124.131 (talk) 20:33, 2 July 2010 (UTC)[reply]
Your methodology differs from the section above this problem. Therein, the leftover moles that hadn't reacted were used to find the concentration. On the other hand, in this problem, the number of moles that have reacted have been used to find the concentration.--478jjjz (talk) 20:38, 2 July 2010 (UTC)[reply]
  • The problem should have said " Calculate the initial molarity of the HCl solution." User:Physchim62's explanation has made me believe that 0.001124/0.25 = 0.00450 M HCl is indeed correct for the initial concentration.
The point is that you start out with 250 mL of an HCl solution: the implication is that it is a sample of a larger volume of solution that you have sitting in a bottle somewhere. Physchim62 (talk) 20:58, 2 July 2010 (UTC)[reply]

I have finally reconciles the approach of this problem and the one above it. In both cases, we have used the unreacted substance to find concentration. In this one, we found the unreacted amount prior to the reaction to get the concentration. In the previous problem, we found the unreacted amount after the reaction to get the concentration.--478jjjz (talk) 20:50, 2 July 2010 (UTC)[reply]

The other point with a titration is that you use an indicator (here, phenolphthalein) so that you know exactly when the last of the reactant has been used up. In effect, you already know the final concentrations – they're both zero. Physchim62 (talk) 20:58, 2 July 2010 (UTC)[reply]
I thank you and others who helped me with this problem.--478jjjz (talk) 21:12, 2 July 2010 (UTC)[reply]

Temperature of the ocean

What IS the temperature of the water 1000s of meters down in the worlds oceans? —Preceding unsigned comment added by 86.182.34.28 (talk) 19:20, 2 July 2010 (UTC)[reply]

Pretty close to freezing. 35-40 deg F. Volcanic vents the exception. Googlemeister (talk) 19:25, 2 July 2010 (UTC)[reply]
Abyssal plain#Terrain features has lots of interesting information on this subject. --Tango (talk) 20:05, 2 July 2010 (UTC)[reply]
Four degrees celsius, because that is the temperature at which water is most dense. See below, and the article - sorry, my error. Thermocline has good information for you. Because of salinity-based and thermal-based upwelling, there may be local variations, but in the very deep ocean, there is a reasonably static temperature profile (even if there is mass transfer of the water). Nimur (talk) 20:30, 2 July 2010 (UTC)[reply]
As I learned after giving the same answer here some time ago, the four degree value holds true for fresh water but not for salt water. The link you pointed to explains that the freezing point of seawater is about minus two Celsius, and the density increases right down to the freezing point. Looie496 (talk) 00:20, 3 July 2010 (UTC)[reply]

Audio amplifier power output

Why is the output of a domestic audio amplifier not inversely proportional to the loudspeaker impedance. I was taught that P=V^2/R but amplifiers dont seem to obey this law when you look at their specifications. Why not?--Bellwelder (talk) 20:12, 2 July 2010 (UTC)[reply]

Mostly because the amplifier itself has impedance as well - this makes your equation:
   P=V2/(Rspeaker+Ramplifier)
The other additional factor in old fashioned amplifiers is the power supply capacity of the power supply ie the transformer - there's a limit to how much power a transformer can supply related to its inductance, but I don't know the equation.
77.86.124.131 (talk) 20:41, 2 July 2010 (UTC)[reply]
You will get maximum power transfer from the amplifier to the loadspeaker when their impedences are matched, not when the speaker impedence is minimum: this is known as the maximum power theorem. Physchim62 (talk) 21:47, 2 July 2010 (UTC)[reply]

natural skin oil

What oils most closely mimic natural skin oil: for example, olive oil, peanut oil, canola oil, mineral oil, vaseline, coconut oil, palm oil, etc.? —Preceding unsigned comment added by Alexsmith44 (talkcontribs) 20:34, 2 July 2010 (UTC)[reply]

Human skin oil is called Sebum , there's a simplfied analysis at Sebaceous_gland#Composition - of those components most are not found in vegetable oils, so a comparison between them isn't that viable. (Mineral oil and Vaseline are pure petroleum product and not like skin oil at all).77.86.124.131 (talk) 20:47, 2 July 2010 (UTC)[reply]
Very roughly it's similar to 40% vegetable oil + 20% beeswax + 10% vaseline + 10% soap (though it's not salty like soap) .. hope that helps.77.86.124.131 (talk) 20:59, 2 July 2010 (UTC)[reply]

Name these two plants?

In a park in southern England. http://img186.imagevenue.com/img.php?image=04455_DSCF0003_122_354lo.JPG The one on the right foreground had grape-like berries earlier in the year, on the now dried fish-bone-shaped parts. Thanks 92.28.247.183 (talk) 21:04, 2 July 2010 (UTC)[reply]

Just to give the same warning as before, the image link gives a popup to a NSFW site77.86.124.131 (talk) 21:18, 2 July 2010 (UTC)[reply]
Back left is I think some kind of fig (ficus), but there's an awful lot of them to choose from. Perhaps the one in front is mahonia? 213.122.27.137 (talk) 22:42, 2 July 2010 (UTC)[reply]

I bought some annual seeds to sow in my garden in southern England and was dispointed to discover that they were only 2 or 3 inches high when flowering. Are there any big garden plants that will grow as an annual, particularly in semi-shade? 'Architectural plant' means a large plant at least three or four feet high. Thanks. 92.28.247.183 (talk) 21:12, 2 July 2010 (UTC)[reply]

Rhododendron? I think you mean a small shrub of which there are many - was there anything else you wanted in the plant that could help narrow the search?77.86.124.131 (talk) 21:20, 2 July 2010 (UTC)[reply]
Some varieties of Hollyhock (Althaea aka Alcea) would certainly qualify: they are often used as the 'backdrop' plant (in front of which shorter flowers are placed) in the traditional English Cottage garden. Foxgloves (Digitalis) would fit the size criterion, but are perennial or biennial.
Leafing quickly through Dr D. G. Hessayon's The Bedding Plant Expert, other candidate annuals with some varieties in the 3-4 feet range include: Love-lies-bleeding (Amaranthus); African Daisy (Arctotis); Spider Flower (Cleome); Datura; Sunflower (Helianthus); Burning Bush (Kochia); Larkspur (Delphinium); Annual Mallow (Lavatera); Nasturtium (Tropaeolum); Sweet Pea (Lathyrus); Ornamental Maize (Zea). 87.81.230.195 (talk) 22:41, 2 July 2010 (UTC)[reply]

July 3

Enthalpy-entropy chart

Hi. On this enthalpy-entropy chart, can someone please tell me what 'x' represents (ie x = 10%, x = 20%, x = 30%...).

The person who actually created the graph seems to have 'retired' from Wikipedia, and another user asked for a caption to be added to it.

Thanks in anticipation,  Chzz  ►  01:32, 3 July 2010 (UTC)[reply]

It's either Percent humidity, i.e. the ratio of actual vapor pressure to vapor pressure at the dew point, or some sort of meaure of proximity to the critical point. --Jayron32 01:38, 3 July 2010 (UTC)[reply]

Medroxyprogesterone versus medroxyprogesterone acetate

Hola,

I'm trying to figure out the difference between medroxyprogesterone versus medroxyprogesterone acetate. Anyone know? Anyone have any sources that can be used to distinguish the two? WLU (t) (c) Wikipedia's rules:simple/complex 01:57, 3 July 2010 (UTC)[reply]

Other than the obvious difference in chemical structure, what are you looking for? It appears that the 17-acetate is the only one used medically - both of the references in the MP article actually relate to MPA. The Merck Manual entry indicates that the former is "supplied as the acetate". The article in MedlinePlus uses the terms interchangeably. Many references discuss plasma levels of MPA, so my initial impression that the acetate is readily hydrolyzed is probably false. -- Scray (talk) 02:59, 3 July 2010 (UTC)[reply]